MED SURG HESI V1 Questions and Answers (2022/2023) (Verified Answers)

The nurse is assessing a 48-year-old client with a history of smoking during a
routine clinic visit. The client, who exercises regularly, reports having pain in the calf
during exercise that disappears at rest. Which of the following findings requires further
evaluation?

  1. Heart rate 57 bpm.
  2. SpO2 of 94% on room air.
  3. Blood pressure 134/82.
  4. Ankle-brachial index of 0.65.
    4
    An Ankle-Brachial Index of 0.65 suggests moderate arterial vascular disease in
    a client who is experiencing intermittent claudication. A Doppler ultrasound is
    indicated for further evaluation. The bradycardic heart rate is acceptable in an athletic
    client with a normal blood pressure. The SpO2 is acceptable; the client has a smoking
    history.

An overweight client taking warfarin (Coumadin) has dry skin due to decreased
arterial blood flow. What should the nurse instruct the client to do? Select all that apply.

  1. Apply lanolin or petroleum jelly to intact skin.
  2. Follow a reduced-calorie, reduced-fat diet.
  3. Inspect the involved areas daily for new ulcerations.
  4. Instruct the client to limit activities of daily living (ADLs).
    5.Use an electric razor to shave
    1,2,3,5
    Maintaining skin integrity is important in preventing chronic ulcers and
    infections. The client should be taught to inspect the skin on a daily basis. The client
    should reduce weight to promote circulation; a diet lower in calories and fat is
    appropriate. Because the client is receiving Coumadin, the client is at risk for bleeding
    from cuts. To decrease the risk of cuts, the nurse should suggest that the client use an
    electric razor. The client with decreased arterial blood flow should be encouraged to
    participate in ADLs. In fact, the client should be encouraged to consult an exercise
    physiologist for an exercise program that enhances the aerobic capacity of the body.

A client with peripheral vascular disease has undergone a right femoral-popliteal
bypass graft. The blood pressure has decreased from 124/80 to 94/62. What
should the nurse assess first?

  1. IV fluid solution.
  2. Pedal pulses.
  3. Nasal cannula flow rate.
  4. Capillary refill
    2
    With each set of vital signs, the nurse should assess the dorsalis pedis and
    posterior tibial pulses. The nurse needs to ensure adequate perfusion to the lower
    extremity with the drop in blood pressure. IV fluids, nasal cannula setting, and capillary
    refill are important to assess; however, priority is to determine the cause of drop in
    blood pressure and that adequate perfusion through the new graft is maintained.

The nurse is caring for a client with peripheral artery disease who has recently
been prescribed clopidogrel (Plavix). The nurse understands that more teaching is
necessary when the client states which of the following:

  1. “I should not be surprised if I bruise easier or if my gums bleed a little when
    brushing my teeth.”
  2. “It doesn’t really matter if I take this medicine with or without food, whatever
    works best for my stomach.”
  3. “I should stop taking Plavix if it makes me feel weak and dizzy.”
  4. “The doctor prescribed this medicine to make my platelets less likely to stick together and help prevent clots from forming.”
    3

Weakness, dizziness, and headache are common adverse effects of Plavix and
the client should report these to the physician if they are problematic; in order to
decrease risk of clot formation, Plavix must be taken regularly and should not be
stopped or taken intermittently. The main adverse effect of Plavix is bleeding, which
often occurs as increased bruising or bleeding when brushing teeth. Plavix is well
absorbed, and while food may help decrease potential gastrointestinal upset, Plavix may
be taken with or without food. Plavix is an antiplatelet agent used to prevent clot
formation in clients who have experienced or are at risk for myocardial infarction,
ischemic stroke, peripheral artery disease, or acute coronary syndrome.

A client is receiving Cilostazol (Pletal) for peripheral arterial disease causing
intermittent claudication. The nurse determines this medication is effective when the
client reports which of the following?

  1. “I am having fewer aches and pains.”
  2. “I do not have headaches anymore.”
  3. “I am able to walk further without leg pain.”
  4. “My toes are turning grayish black in color.”
    3

Cilostazol is indicated for management of intermittent claudication. Symptoms
usually improve within 2 to 4 weeks of therapy. Intermittent claudication prevents
clients from walking for long periods of time. Cilostazol inhibits platelet aggregation
induced by various stimuli and improving blood flow to the muscles and allowing the
client to walk long distances without pain. Peripheral arterial disease causes pain
mainly of the leg muscles. “Aches and pains” does not specify exactly where the pain is
occurring. Headaches may occur as a side effect of this drug, and the client should
report this information to the health care provider. Peripheral arterial disease causes
decreased blood supply to the peripheral tissues and may cause gangrene of the toes; the
drug is effective when the toes are warm to the touch and the color of the toes is similar
to the color of the body.

The client admitted with peripheral vascular disease (PVD) asks the nurse why
her legs hurt when she walks. The nurse bases a response on the knowledge that the
main characteristic of PVD is:

  1. Decreased blood flow.
  2. Increased blood flow.
  3. Slow blood flow.
  4. Thrombus formation.
    1

Decreased blood flow is a common characteristic of all PVD. When the
demand for oxygen to the working muscles becomes greater than the supply, pain is the
outcome. Slow blood flow throughout the circulatory system may suggest pump failure.
Thrombus formation can result from stasis or damage to the intima of the vessels.

The nurse is planning care for a client who is diagnosed with peripheral vascular
disease (PVD) and has a history of heart failure. The nurse should develop a plan of
care that is based on the fact that the client may have a low tolerance for exercise
related to:

  1. Decreased blood flow.
  2. Increased blood flow.
  3. Decreased pain.
  4. Increased blood viscosity.
    1
    A client with PVD and heart failure will experience decreased blood flow. In
    this situation, low exercise tolerance (oxygen demand becomes greater than the oxygen
    supply) may be related to less blood being ejected from the left ventricle into the
    systemic circulation. Decreased blood supply to the tissues results in pain. Increased
    blood viscosity may be a component, but it is of much less importance than the disease
    processes.

When assessing the lower extremities of a client with peripheral vascular
disease (PVD), the nurse notes bilateral ankle edema. The edema is related to:

  1. Competent venous valves.
  2. Decreased blood volume.
  3. Increase in muscular activity.
  4. Increased venous pressure.
    4

In PVD, decreased blood flow can result in increased venous pressure. The
increase in venous pressure results in an increase in capillary hydrostatic pressure,
which causes a net filtration of fluid out of the capillaries into the interstitial space,
resulting in edema. Valves often become incompetent with PVD. Blood volume is not
decreased in this condition. Decreased muscular action would contribute to the
formation of edema in the lower extremities.

The nurse is obtaining the pulse of a client who has had a femoral-popliteal
bypass surgery 6 hours ago. (See below) Which assessment provides the most accurate
information about the client’s postoperative status?

  1. radial pulse
  2. femoral pulse
  3. apical pulse
  4. dorsalis pedis pulse
    4
    The presence of a strong dorsalis pedis pulse indicates that there is circulation to
    the extremity distal to the surgery indicating that the graft between the femoral and
    popliteal artery is allowing blood to circulate effectively. Answer 1 shows the nurse
    obtaining the radial pulse; answer 2 shows the femoral pulse, which is proximal to the
    surgery site and will not indicate circulation distal to the surgery site. Answer 3 shows
    the nurse obtaining an apical pulse.

The nurse is teaching a client about risk factors associated with atherosclerosis
and how to reduce the risk. Which of the following is a risk factor that the client is not
able to modify?

  1. Diabetes.
  2. Age.
  3. Exercise level.
  4. Dietary preferences
    2
    Age is a nonmodifiable risk factor for atherosclerosis. The nurse instructs the
    client to manage modifiable risk factors such as comorbid diseases (eg, diabetes),
    activity level, and diet. Controlling serum blood glucose levels, engaging in regular
    aerobic activity, and choosing a diet low in saturated fats can reduce the risk of
    developing atherosclerosis.

The nurse is assessing the lower extremities of the client with peripheral
vascular disease (PVD). During the assessment, the nurse should expect to find which of
the following clinical manifestations of PVD? Select all that apply.

  1. Hairy legs.
  2. Mottled skin.
  3. Pink skin.
  4. Coolness.
  5. Moist skin.
    2,4

Reduction of blood flow to a specific area results in decreased oxygen and
nutrients. As a result, the skin may appear mottled. The skin will also be cool to the
touch. Loss of hair and dry skin are other signs that the nurse may observe in a client
with PVD of the lower extremities.

The nurse is unable to palpate the client’s left pedal pulses. Which of the
following actions should the nurse take next?

  1. Auscultate the pulses with a stethoscope.
  2. Call the physician.
  3. Use a Doppler ultrasound device.
  4. Inspect the lower left extremity
    3

When pedal pulses are not palpable, the nurse should obtain a Doppler
ultrasound device. Auscultation is not likely to be helpful if the pulse isn’t palpable.
Inspection of the lower extremity can be done simultaneously when palpating, but the
nurse should first try to locate a pulse by Doppler. Calling the physician may be
necessary if there is a change in the client’s condition.

Which of the following lipid abnormalities is a risk factor for the development
of atherosclerosis and peripheral vascular disease?

  1. Low concentration of triglycerides.
  2. High levels of high-density lipid (HDL) cholesterol.
  3. High levels of low-density lipid (LDL) cholesterol.
  4. Low levels of LDL cholesterol.
    3

An increased LDL cholesterol concentration has been documented as a risk
factor for the development of atherosclerosis. LDL cholesterol is not broken down in
the liver but is deposited into the intima of the blood vessels. Low triglyceride levels
are desirable. High HDL and low LDL levels are beneficial and are known to be
protective for the cardiovascular system.

When assessing an individual with peripheral vascular disease, which clinical
manifestation would indicate complete arterial obstruction in the lower left leg?

  1. Aching pain in the left calf.
  2. Burning pain in the left calf.
  3. Numbness and tingling in the left leg.
  4. Coldness of the left foot and ankle
    4
    Coldness in the left foot and ankle is consistent with complete arterial
    obstruction. Other expected findings would include paralysis and pallor. Aching pain, a
    burning sensation, or numbness and tingling are earlier signs of tissue hypoxia and
    ischemia and are commonly associated with incomplete obstruction.

A client with peripheral vascular disease returns to the surgical care unit after
having femoral-popliteal bypass grafting. Indicate in which order the nurse should
conduct assessment of this client.

  1. Postoperative pain.
  2. Peripheral pulses.
  3. Urine output.
  4. Incision site.
    (2,4,3,1)
    Because assessment of the presence and quality of the pedal pulses in the affected
    extremity is essential after surgery to make sure that the bypass graft is functioning, this
    step should be done first. The nurse should next ensure that the dressing is intact, and
    then that the client has adequate urine output. Lastly, the nurse should determine the
    client’s level of pain.

A client with heart failure has bilateral +4 edema of the right ankle that extends
up to midcalf. The client is sitting in a chair with the legs in a dependent position.
Which of the following goals is the priority?

  1. Decrease venous congestion.
  2. Maintain normal respirations.
  3. Maintain body temperature.
  4. Prevent injury to lower extremities.
    1
    Decreasing venous congestion in the extremities is a desired outcome for
    clients with heart failure. The nurse should elevate the client’s legs above the level of
    the heart to achieve this goal. The client is not demonstrating difficulty breathing or
    being cold. The nurse should prevent injury to the swollen extremity; however, this is
    not the priority.

The nurse is assessing an older Caucasian male who has a history of peripheral
vascular disease. The nurse observes that the man’s left great toe is black. The
discoloration is probably a result of:

  1. Atrophy.
  2. Contraction.
  3. Gangrene.
  4. Rubor.
    3
    The term gangrene refers to blackened, decomposing tissue that is devoid of
    circulation. Chronic ischemia and death of the tissue can lead to gangrene in the affected
    extremity. Injury, edema, and decreased circulation lead to infection, gangrene, and
    tissue death. Atrophy is the shrinking of tissue, and contraction is joint stiffening
    secondary to disuse. The term rubor denotes a reddish color of the skin

A client has peripheral vascular disease (PVD) of the lower extremities. The
client tells the nurse, “I’ve really tried to manage my condition well.” Which of the
following routines should the nurse evaluate as having been appropriate for this client?

  1. Resting with the legs elevated above the level of the heart.
  2. Walking slowly but steadily for 30 minutes twice a day.
  3. Minimizing activity.
  4. Wearing antiembolism stockings at all times when out of bed
    2

Slow, steady walking is a recommended activity for clients with peripheral
vascular disease because it stimulates the development of collateral circulation. The
client with PVD should not remain inactive. Elevating the legs above the heart or
wearing antiembolism stockings is a strategy for alleviating venous congestion and may
worsen peripheral arterial disease

A client is scheduled for an arteriogram. The nurse should explain to the client
that the arteriogram will confirm the diagnosis of occlusive arterial disease by:

  1. Showing the location of the obstruction and the collateral circulation.
  2. Scanning the affected extremity and identifying the areas of volume changes.
  3. Using ultrasound to estimate the velocity changes in the blood vessels.
  4. Determining how long the client can walk.
    1
    An arteriogram involves injecting a radiopaque contrast agent directly into the
    vascular system to visualize the vessels. It usually involves computed tomographic
    scanning. The velocity of the blood flow can be estimated by duplex ultrasound. The
    client’s ankle-brachial index is determined, and then the client is requested to walk. The
    normal response is little or no drop in ankle systolic pressure after exercise.

A client is scheduled to have an arteriogram. During the arteriogram, the client
reports having nausea, tingling, and dyspnea. The nurse’s immediate action should be
to:

  1. Administer epinephrine.
  2. Inform the physician.
  3. Administer oxygen.
  4. Inform the client that the procedure is almost over.
    2

.Clients may have an immediate or a delayed reaction to the radiopaque dye.
The physician should be notified immediately because the symptoms suggest an allergic
reaction. Treatment may involve administering oxygen and epinephrine. Explaining that
the procedure is over does not address the current symptoms

Which of the following is an expected outcome when a client is receiving an IV
administration of furosemide?

  1. Increased blood pressure.
  2. Increased urine output.
  3. Decreased pain.
  4. Decreased premature ventricular contractions.
    2

Furosemide is a loop diuretic that acts to increase urine output. Furosemide
does not increase blood pressure, decrease pain, or decrease arrhythmias.

A client has had a pulmonary artery catheter inserted. In performing
hemodynamic monitoring with the catheter, the nurse will wedge the catheter to gain
information about which of the following?

  1. Cardiac output.
  2. Right atrial blood flow.
  3. Left end-diastolic pressure.
  4. Cardiac index
    3

When wedged, the catheter is “pointing” indirectly at the left end-diastolic
pressure. The pulmonary artery wedge pressure is measured when the tip of the catheter
is slowing inflated and allowed to wedge into a branch of the pulmonary artery. Once
the balloon is wedged, the catheter reads the pressure in front of the balloon. During
diastole, the mitral valve is open, reflecting left ventricular end diastolic pressure.
Cardiac output is the amount of blood ejected by the heart in 1 minute and is determined
through thermodilution and not wedge pressure. Cardiac index is calculated by dividing
the client’s cardiac output by the client’s body surface area, and is considered a more
accurate reflection of the individual client’s cardiac output. Right atrial blood pressure
is not measured with the pulmonary artery catheter.

After a myocardial infarction, the hospitalized client is taught to move the legs
while resting in bed. The expected outcome of this exercise is to:

  1. Prepare the client for ambulation.
  2. Promote urinary and intestinal elimination.
  3. Prevent thrombophlebitis and blood clot formation.
  4. Decrease the likelihood of pressure ulcer formation.
    3

Encouraging the client to move the legs while in bed is a preventive strategy
taught to all clients who are hospitalized and on bed rest to promote venous return. The
muscular action aids in venous return and prevents venous stasis in the lower
extremities. These exercises are not intended to prepare the client for ambulation. These
exercises are not associated with promoting urinary and intestinal elimination. These
exercises are not performed to decrease the risk of pressure ulcer formation

Which of the following is the most appropriate diet for a client during the acute
phase of myocardial infarction?

  1. Liquids as desired.
  2. Small, easily digested meals.
  3. Three regular meals per day.
  4. Nothing by mouth
    2

Recommended dietary principles in the acute phase of MI include avoiding
large meals because small, easily digested foods are better tolerated. Fluids are given
according to the client’s needs, and sodium restrictions may be prescribed, especially
for clients with manifestations of heart failure. Cholesterol restrictions may be
prescribed as well. Clients are not prescribed diets of liquids only or restricted to
nothing by mouth unless their condition is very unstable.

The nurse is caring for a client who recently experienced a myocardial
infarction and has been started on clopidogrel (Plavix). The nurse should develop a
teaching plan that includes which of the following points? Select all that apply.

  1. The client should report unexpected bleeding or bleeding that lasts a long time.
  2. The client should take Plavix with food.
  3. The client may bruise more easily and may experience bleeding gums.
  4. Plavix works by preventing platelets from sticking together and forming a clot.
  5. The client should drink a glass of water after taking Plavix.
    1,3,4

Plavix is generally well absorbed and may be taken with or without
food; it should be taken at the same time every day and, while food may help prevent
potential GI upset, food has no effect on absorption of the drug. Bleeding is the most
common adverse effect of Plavix; the client must understand the importance of reporting
any unexpected, prolonged, or excessive bleeding including blood in urine or stool.
Increased bruising and bleeding gums are possible side effects of Plavix; the client
should be aware of this possibility. Plavix is an antiplatelet agent used to prevent clot
formation in clients that have experienced or are at risk for myocardial infarction,
ischemic stroke, peripheral artery disease, or acute coronary syndrome. It is not
necessary to drink a glass of water after taking Plavix.

Which client is at greatest risk for coronary artery disease?

  1. A 32-year-old female with mitral valve prolapse who quit smoking 10 years ago.
  2. A 43-year-old male with a family history of CAD and cholesterol level of 158
    (8.8 mmol/L).
  3. A 56-year-old male with an HDL of 60 (3.3 mmol/L) who takes atorvastatin.
  4. A 65-year-old female who is obese with an LDL of 188 (10.4 mmol/L).
    4

The woman who is 65 years old, overweight, and has an elevated LDL is at
greatest risk. Total cholesterol greater than 200 (11.1 mmol/L), LDL greater than 100
(5.5 mmol/L), HDL less than 40 (2.2 mmol/L) in men, HDL less than 50 (2.8 mmol/L) in
women, men 45 years and older, women 55 years and older, smoking and obesity
increase the risk of CAD. Atorvastatin reduces LDL and decreases risk of CAD. The
combination of postmenopausal, obesity, and high LDL places this client at greatest risk.

. A middle-aged adult with a family history of CAD has the following: total
cholesterol 198 (11 mmol/L); LDL cholesterol 120 (6.7 mmol/L); HDL cholesterol 58
(3.2 mmol/L); triglycerides 148 (8.2 mmol/L); blood sugar 102 (5.7 mmol/L); and Creactive
protein (CRP) 4.2. The health care provider prescribes a statin medication and
aspirin. The client asks the nurse why these medications are needed. Which is the best
response by the nurse?

  1. “The labs indicate severe hyperlipidemia and the medications will lower your
    LDL, along with a low-fat diet.”
  2. “The triglycerides are elevated and will not return to normal without these
    medications.”
  3. “The CRP is elevated indicating inflammation seen in cardiovascular disease,
    which can be lowered by the medications prescribed.”
  4. “These medications will reduce the risk of type 2 diabetes.”
    3

CRP is a marker of inflammation and is elevated in the presence of
cardiovascular disease. The high sensitivity CRP (hs-CRP) is the blood test for greater
accuracy in measuring the CRP to evaluate cardiovascular risk. The family history,
postmenopausal age, LDL above optimum levels, and elevated CRP place the client at
risk of CAD. Statin medications can decrease LDL, whereas statins and aspirin can
reduce CRP and decrease the risk of MI and stroke. The blood sugar is within normal
limits.

The client has been managing angina episodes with nitroglycerin. Which of the
following indicate the drug is effective?

  1. Decreased chest pain.
  2. Increased blood pressure.
  3. Decreased blood pressure.
  4. Decreased heart rate
    1

Nitroglycerin acts to decrease myocardial oxygen consumption. Vasodilation
makes it easier for the heart to eject blood, resulting in decreased oxygen needs.
Decreased oxygen demand reduces pain caused by heart muscle not receiving sufficient
oxygen. While blood pressure may decrease ever so slightly due to the vasodilation
effects of nitroglycerine, it is only secondary and not related to the angina the patient is
experiencing. Increased blood pressure would mean the heart would work harder,
increasing oxygen demand and thus angina. Decreased heart rate is not an effect of
nitroglycerine.

If a client displays risk factors for coronary artery disease, such as smoking
cigarettes, eating a diet high in saturated fat, or leading a sedentary lifestyle, techniques
of behavior modification may be used to help the client change the behavior. The nurse
can best reinforce new adaptive behaviors by:

  1. Explaining how the risk factor behavior leads to poor health.
  2. Withholding praise until the new behavior is well established.
  3. Rewarding the client whenever the acceptable behavior is performed.
  4. Instilling mild fear into the client to extinguish the behavior.
    3

A basic principle of behavior modification is that behavior that is learned and
continued is behavior that has been rewarded. Other reinforcement techniques have not
been found to be as effective as reward.

Alteplase recombinant, or tissue plasminogen activator (t-PA), a thrombolytic
enzyme, is administered during the first 6 hours after onset of myocardial infarction
(MI) to:

  1. Control chest pain.
  2. Reduce coronary artery vasospasm.
  3. Control the arrhythmias associated with MI.
  4. Revascularize the blocked coronary artery.
    4

The thrombolytic agent t-PA, administered intravenously, lyses the clot
blocking the coronary artery. The drug is most effective when administered within the
first 6 hours after onset of MI. The drug does not reduce coronary artery vasospasm;
nitrates are used to promote vasodilation. Arrhythmias are managed by antiarrhythmic
drugs. Surgical approaches are used to open the coronary artery and re-establish a
blood supply to the area.

After the administration of t-PA, the nurse should:

  1. Observe the client for chest pain.
  2. Monitor for fever.
  3. Review the 12-lead electrocardiogram (ECG).
  4. Auscultate breath sounds
    1

Although monitoring the 12-lead ECG and monitoring breath sounds are
important, observing the client for chest pain is the nursing assessment priority because
closure of the previously obstructed coronary artery may recur. Clients who receive t-
PA frequently receive heparin to prevent closure of the artery after administration of t-
PA. Careful assessment for signs of bleeding and monitoring of partial thromboplastin
time are essential to detect complications. Administration of t-PA should not cause
fever.

When monitoring a client who is receiving tissue plasminogen activator (t-PA),
the nurse should have resuscitation equipment available because reperfusion of the
cardiac tissue can result in which of the following?

  1. Cardiac arrhythmias.
  2. Hypertension.
  3. Seizure.
  4. Hypothermia.
    1

Cardiac arrhythmias are commonly observed with administration of t-PA.
Cardiac arrhythmias are associated with reperfusion of the cardiac tissue. Hypotension
is commonly observed with administration of t-PA. Seizures and hypothermia are not
generally associated with reperfusion of the cardiac tissue.

Prior to administering tissue plasminogen activator (t-PA), the nurse should
assess the client for which of the following contradictions to administering the drug?

  1. Age greater than 60 years.
  2. History of cerebral hemorrhage.
  3. History of heart failure.
  4. Cigarette smoking.
    2

A history of cerebral hemorrhage is a contraindication to administration of t-
PA because the risk of hemorrhage may be further increased. Age greater than 60 years,
history of heart failure, and cigarette smoking are not contraindications.

A client has driven himself to the emergency department. He is 50 years old,
has a history of hypertension, and informs the nurse that his father died from a heart
attack at age 60. The client has indigestion. The nurse connects him to an
electrocardiogram monitor and begins administering oxygen at 2 L/min per nasal
cannula. The nurse’s next action should be to:

  1. Call for the physician.
  2. Start an IV infusion.
  3. Obtain a portable chest radiograph.
  4. Draw blood for laboratory studies
    2

Advanced cardiac life support recommends that at least one or two IV lines
be inserted in one or both of the antecubital spaces. Calling the physician, obtaining a
portable chest radiograph, and drawing blood for the laboratory are important but
secondary to starting the IV line.

Crackles heard on lung auscultation indicate which of the following?

  1. Cyanosis.
  2. Bronchospasm.
  3. Airway narrowing.
    4.Fluid-filled alveoli.
    4

Crackles are auscultated over fluid-filled alveoli. Crackles heard on lung
auscultation do not have to be associated with cyanosis. Bronchospasm and airway
narrowing generally are associated with wheezing sounds.

A 68-year-old client on day 2 after hip surgery has no cardiac history but
reports having chest heaviness. The first nursing action should be to:

  1. Inquire about the onset, duration, severity, and precipitating factors of the
    heaviness.
  2. Administer oxygen via nasal cannula.
  3. Offer pain medication for the chest heaviness.
  4. Inform the physician of the chest heaviness.
    1 Further assessment is needed in this situation. It is premature to initiate other
    actions until further data have been gathered. Inquiring about the onset, duration,
    location, severity, and precipitating factors of the chest heaviness will provide pertinent
    information to convey to the physician.

The nurse is assessing an older adult with a pacemaker who leads a sedentary
lifestyle. The client reports being unable to perform activities that require physical
exertion. The nurse should further assess the client for which of the following?

  1. Left ventricular atrophy.
  2. Irregular heartbeats.
  3. Peripheral vascular occlusion.
  4. Pacemaker placement
    1

In older adults who are less active and do not exercise the heart muscle,
atrophy can result. Disuse or deconditioning can lead to abnormal changes in the
myocardium of the older adult. As a result, under sudden emotional or physical stress,
the left ventricle is less able to respond to the increased demands on the myocardial
muscle. Decreased cardiac output, cardiac hypertrophy, and heart failure are examples
of the chronic conditions that may develop in response to inactivity, rather than in
response to the aging process. Irregular heartbeats are generally not associated with an
older sedentary adult’s lifestyle. Peripheral vascular occlusion or pacemaker placement
should not affect response to stress.

. Following diagnosis of angina pectoris, a client reports being unable to walk up
two flights of stairs without pain. Which of the following measures would most likely
help the client prevent this problem?

  1. Climb the steps early in the day.
  2. Rest for at least an hour before climbing the stairs.
  3. Take a nitroglycerin tablet before climbing the stairs.
  4. Lie down after climbing the stairs.
    3

Nitroglycerin may be used prophylactically before stressful physical
activities such as stair climbing to help the client remain pain free. Climbing the stairs
early in the day would have no impact on decreasing pain episodes. Resting before or
after an activity is not as likely to help prevent an activity-related pain episode.

The client who experiences angina has been told to follow a low-cholesterol
diet. Which of the following meals would be best?

  1. Hamburger, salad, and milkshake.
  2. Baked liver, green beans, and coffee.
  3. Spaghetti with tomato sauce, salad, and coffee.
  4. Fried chicken, green beans, and skim milk
    3

Pasta, tomato sauce, salad, and coffee would be the best selection for the
client following a low-cholesterol diet. Hamburgers, milkshakes, liver, and fried foods
tend to be high in cholesterol.

The nurse should caution the client with diabetes mellitus who is taking a
sulfonylurea that alcoholic beverages should be avoided while taking these drugs
because they can cause which of the following?

  1. Hypokalemia.
  2. Hyperkalemia.
  3. Hypocalcemia.
    4.Disulfiram (Antabuse)-like symptoms
    4

A client with diabetes who takes any first- or second-generation sulfonylurea
should be advised to avoid alcohol intake. Sulfonylureas in combination with alcohol
can cause serious disulfiram (Antabuse)-like reactions, including flushing, angina,
palpitations, and vertigo. Serious reactions, such as seizures and possibly death, may
also occur. Hypokalemia, hyperkalemia, and hypocalcemia do not result from taking
sulfonylureas in combination with alcohol.

Which of the following conditions is the most significant risk factor for the
development of type 2 diabetes mellitus?

  1. Cigarette smoking.
  2. High-cholesterol diet.
  3. Obesity.
  4. Hypertension.
    3

The most important factor predisposing to the development of type 2 diabetes
mellitus is obesity. Insulin resistance increases with obesity. Cigarette smoking is not a
predisposing factor, but it is a risk factor that increases complications of diabetes
mellitus. A high-cholesterol diet does not necessarily predispose to diabetes mellitus,
but it may contribute to obesity and hyperlipidemia. Hypertension is not a predisposing
factor, but it is a risk factor for developing complications of diabetes mellitus.

Which of the following indicates a potential complication of diabetes mellitus?

  1. Inflamed, painful joints.
  2. Blood pressure of 160/100 mm Hg.
  3. Stooped appearance.
  4. Hemoglobin of 9 g/dL (90 g/L).
    2
    The client with diabetes mellitus is especially prone to hypertension due to
    atherosclerotic changes, which leads to problems of the microvascular and
    macrovascular systems. This can result in complications in the heart, brain, and kidneys.
    Heart disease and stroke are twice as common among people with diabetes mellitus as
    among people without the disease. Painful, inflamed joints accompany rheumatoid
    arthritis. A stooped appearance accompanies osteoporosis with narrowing of the
    vertebral column. A low hemoglobin concentration accompanies anemia, especially
    iron deficiency anemia and anemia of chronic disease.

The nurse is teaching the client about home blood glucose monitoring. Which of
the following blood glucose measurements indicates hypoglycemia?

  1. 59 mg/dL (3.3 mmol/L).
  2. 75 mg/dL (4.2 mmol/L).
  3. 108 mg/dL (6 mmol/L).
  4. 119 mg/dL (6.6 mmol/L).
    1

Although some individual variation exists, when the blood glucose level
decreases to less than 70 mg/dL (3.9 mmol/L), the client experiences or is at risk for
hypoglycemia. Hypoglycemia can occur in both type 1 and type 2 diabetes mellitus,
although it is more common when the client is taking insulin. The nurse should instruct
the client on the prevention, detection, and treatment of hypoglycemia.

Assessment of the diabetic client for common complications should include
examination of the:

  1. Abdomen.
  2. Lymph glands.
  3. Pharynx.
  4. Eyes.
    4

Diabetic retinopathy, cataracts, and glaucoma are common complications in
diabetics, necessitating eye assessment and examination. The feet should also be
examined at each client encounter, monitoring for thickening, fissures, or breaks in the
skin; ulcers; and thickened nails. Although assessments of the abdomen, pharynx, and
lymph glands are included in a thorough examination, they are not pertinent to common
diabetic complications.

The client with type 1 diabetes mellitus is taught to take isophane insulin
suspension NPH (Humulin N) at 5 PM each day. The client should be instructed that the
greatest risk of hypoglycemia will occur at about what time?

  1. 11 AM, shortly before lunch.
  2. 1 PM, shortly after lunch.
  3. 6 PM, shortly after dinner.
    4.1 AM, while sleeping
    4

The client with diabetes mellitus who is taking NPH insulin (Humulin N) in
the evening is most likely to become hypoglycemic shortly after midnight because this
insulin peaks in 6 to 8 hours. The client should eat a bedtime snack to help prevent
hypoglycemia while sleeping.

A nurse is teaching a client with type 1 diabetes mellitus who jogs daily about
the preferred sites for insulin absorption. What is the most appropriate site for a client
who jogs?

  1. Arms.
  2. Legs.
  3. Abdomen.
  4. Iliac crest.
    3

If the client engages in an activity or exercise that focuses on one area of the
body, that area may cause inconsistent absorption of insulin. A good regimen for a
jogger is to inject the abdomen for 1 week and then rotate to the buttock. A jogger may
have inconsistent absorption in the legs or arms with strenuous running. The iliac crest
is not an appropriate site due to a lack of loose skin and subcutaneous tissue in that area.

A client with diabetes is taking insulin lispro (Humalog) injections. The nurse
should advise the client to eat:

  1. Within 10 to 15 minutes after the injection.
  2. 1 hour after the injection.
  3. At any time, because timing of meals with lispro injections is unnecessary.
  4. 2 hours before the injection.
    1

Insulin lispro (Humalog) begins to act within 10 to 15 minutes and lasts
approximately 4 hours. A major advantage of Humalog is that the client can eat almost
immediately after the insulin is administered. The client needs to be instructed regarding
the onset, peak, and duration of all insulin, as meals need to be timed with these
parameters. Waiting 1 hour to eat may precipitate hypoglycemia. Eating 2 hours before
the insulin lispro could cause hyperglycemia if the client does not have circulating
insulin to metabolize the carbohydrate.

The best indicator that the client has learned how to give an insulin self injection
correctly is when the client can:

  1. Perform the procedure safely and correctly.
  2. Critique the nurse’s performance of the procedure.
  3. Explain all steps of the procedure correctly.
  4. Correctly answer a posttest about the procedure
    1

The nurse should judge that learning has occurred from the evidence of a
change in the client’s behavior. A client who performs a procedure safely and correctly
demonstrates that he has acquired a skill. Evaluation of this skill acquisition requires
performance of that skill by the client with observation by the nurse. The client must
also demonstrate cognitive understanding, as shown by the ability to critique the nurse’s
performance. Explaining the steps demonstrates acquisition of knowledge at the
cognitive level only. A posttest does not indicate the degree to which the client has
learned a psychomotor skill.

The nurse is instructing the client on insulin administration. The client is performing a
return demonstration for preparing the insulin. The client’s morning dose of insulin is 10
units of regular and 22 units of NPH. The nurse checks the dose accuracy with the client.
The nurse determines that the client has prepared the correct dose when the syringe
reads how many units?
32 units

Angiotensin-converting enzyme (ACE) inhibitors may be prescribed for the
client with diabetes mellitus to reduce vascular changes and possibly prevent or delay
development of:

  1. Chronic obstructive pulmonary disease (COPD).
  2. Pancreatic cancer.
  3. Renal failure.
  4. Cerebrovascular accident.
    3

Renal failure frequently results from the vascular changes associated with
diabetes mellitus. ACE inhibitors increase renal blood flow and are effective in
decreasing diabetic nephropathy. Chronic obstructive pulmonary disease is not a
complication of diabetes, nor is it prevented by ACE inhibitors. Pancreatic cancer is
neither prevented by ACE inhibitors nor considered a complication of diabetes.
Cerebrovascular accident is not directly prevented by ACE inhibitors, although
management of hypertension will decrease vascular disease.

The nurse should teach the diabetic client that which of the following is the
most common symptom of hypoglycemia?

  1. Nervousness.
  2. Anorexia.
  3. Kussmaul’s respirations.
  4. Bradycardia.
    1

The four most commonly reported signs and symptoms of hypoglycemia are
nervousness, weakness, perspiration, and confusion. Other signs and symptoms include
hunger, incoherent speech, tachycardia, and blurred vision. Anorexia and Kussmaul’s
respirations are clinical manifestations of hyperglycemia or ketoacidosis. Bradycardia
is not associated with hypoglycemia; tachycardia is.

The nurse is assessing the client’s use of medications. Which of the following
medications may cause a complication with the treatment plan of a client with diabetes?

  1. Aspirin.
  2. Steroids.
  3. Sulfonylureas.
  4. Angiotensin-converting enzyme (ACE) inhibitors
    2

Steroids can cause hyperglycemia because of their effects on carbohydrate
metabolism, making diabetic control more difficult. Aspirin is not known to affect
glucose metabolism. Sulfonylureas are oral hypoglycemic agents used in the treatment of
diabetes mellitus. ACE inhibitors are not known to affect glucose metabolism.

A client with type 1 diabetes mellitus has influenza. The nurse should instruct
the client to:

  1. Increase the frequency of self-monitoring (blood glucose testing).
  2. Reduce food intake to diminish nausea.
  3. Discontinue that dose of insulin if unable to eat.
  4. Take half of the normal dose of insulin
    1 Colds and influenza present special challenges to the client with diabetes
    mellitus because the body’s need for insulin increases during illness. Therefore, the
    client must take the prescribed insulin dose, increase the frequency of blood glucose
    testing, and maintain an adequate fluid intake to counteract the dehydrating effect of
    hyperglycemia. Clear fluids, juices, and Gatorade are encouraged. Not taking insulin
    when sick, or taking half the normal dose, may cause the client to develop ketoacidosis.

Which of the following is a priority goal for the diabetic client who is taking
insulin and has nausea and vomiting from a viral illness or influenza?

  1. Obtaining adequate food intake.
  2. Managing own health.
  3. Relieving pain.
  4. Increasing activity.
    1

The priority goal for the client with diabetes mellitus who is experiencing
vomiting with influenza is to obtain adequate nutrition. The diabetic client should eat
small, frequent meals of 50 g of carbohydrate or food equal to 200 cal every 3 to 4
hours. If the client cannot eat the carbohydrates or take fluids, the health care provider
should be called or the client should go to the emergency department. The diabetic
client is in danger of complications with dehydration, electrolyte imbalance, and
ketoacidosis. Increasing the client’s health management skills is important to lifestyle
behaviors, but it is not a priority during this acute illness of influenza. Pain relief may
be a need for this client, but it is not the priority at this time; neither is increasing
activity during the illness.

A client with diabetes begins to cry and says, “I just cannot stand the thought of
having to give myself a shot every day.” Which of the following would be the best
response by the nurse?

  1. “If you do not give yourself your insulin shots, you will die.”
  2. “We can teach your daughter to give the shots so you will not have to do it.”
  3. “I can arrange to have a home care nurse give you the shots every day.”
  4. “What is it about giving yourself the insulin shots that bothers you?”
    4

The best response is to allow the client to verbalize her fears about giving
herself a shot each day. Tactics that increase fear are not effective in changing behavior.
If possible, the client needs to be responsible for her own care, including giving selfinjections.
It is unlikely that the client’s insurance company will pay for home-care
visits if the client is capable of self-administration.

A client is to have a transsphenoidal hypophysectomy to remove a large,
invasive pituitary tumor. The nurse should instruct the client that the surgery will be
performed through an incision in the:

  1. Back of the mouth.
  2. Nose.
  3. Sinus channel below the right eye.
    4.Upper gingival mucosa in the space between the upper gums and lip.
    4

With transsphenoidal hypophysectomy, the sella turcica is entered from
below, through the sphenoid sinus. There is no external incision; the incision is made
between the upper lip and gums.

To help minimize the risk of postoperative respiratory complications after a
hypophysectomy, during preoperative teaching, the nurse should instruct the client how
to:

  1. Use incentive spirometry.
  2. Turn in bed.
  3. Take deep breaths.
  4. Cough.
    3

Deep breathing is the best choice for helping prevent atelectasis. The client
should be placed in the semi-Fowler’s position (or as prescribed) and taught deep
breathing, sighing, mouth breathing, and how to avoid coughing. Blow bottles are not
effective in preventing atelectasis because they do not promote sustained alveolar
inflation to maximal lung capacity. Frequent position changes help loosen lung
secretions, but deep breathing is most important in preventing atelectasis. Coughing is
contraindicated because it increases intracranial pressure and can cause cerebrospinal
fluid to leak from the point at which the sella turcica was entered.

Following a transsphenoidal hypophysectomy, the nurse should assess the client
for:

  1. Cerebrospinal fluid (CSF) leak.
  2. Fluctuating blood glucose levels.
  3. Cushing’s syndrome.
  4. Cardiac arrhythmias
    1

A major focus of nursing care after transsphenoidal hypophysectomy is the
prevention of and monitoring for a CSF leak. CSF leakage can occur if the patch or
incision is disrupted. The nurse should monitor for signs of infection, including elevated
temperature, increased white blood cell count, rhinorrhea, nuchal rigidity, and persistent
headache. Hypoglycemia and adrenocortical insufficiency may occur. Monitoring for
fluctuating blood glucose levels is not related specifically to transsphenoidal
hypophysectomy. The client will be given IV fluids postoperatively to supply
carbohydrates. Cushing’s disease results from adrenocortical excess, not insufficiency.
Monitoring for cardiac arrhythmias is important, but arrhythmias are not anticipated
following a transsphenoidal hypophysectomy.

A male client expresses concern about how a hypophysectomy will affect his
sexual function. Which of the following statements provides the most accurate
information about the physiologic effects of hypophysectomy?

  1. Removing the source of excess hormone should restore the client’s libido,
    erectile function, and fertility.
  2. Potency will be restored, but the client will remain infertile.
  3. Fertility will be restored, but impotence and decreased libido will persist.
  4. Exogenous hormones will be needed to restore erectile function after the
    adenoma is removed.
    1

The client’s sexual problems are directly related to the excessive prolactin
level. Removing the source of excessive hormone secretion should allow the client to
return gradually to a normal physiologic pattern. Fertility will return, and erectile
function and sexual desire will return to baseline as hormone levels return to normal.

The nurse instructs the unlicensed nursing personnel (UAP) on how to provide
oral hygiene for clients who cannot perform this task for themselves. Which of the
following techniques should the nurse tell the UAP to incorporate into the client’s daily
care?

  1. Assess the oral cavity each time mouth care is given and record observations.
  2. Use a soft toothbrush to brush the client’s teeth after each meal.
  3. Swab the client’s tongue, gums, and lips with a soft foam applicator every 2
    hours.
  4. Rinse the client’s mouth with mouthwash several times a day.
    2

A soft toothbrush should be used to brush the client’s teeth after every meal
and more often as needed. Mechanical cleaning is necessary to maintain oral health,
stimulate gingiva, and remove plaque. Assessing the oral cavity and recording
observations is the responsibility of the nurse, not the nursing assistant. Swabbing with
a safe foam applicator does not provide enough friction to clean the mouth. Mouthwash
can be a drying irritant and is not recommended for frequent use.

The nurse is developing standards of care for a client with gastroesophageal
reflux disease and wants to review current evidence for practice. Which one of the
following resources will provide the most helpful information?

  1. A review in the Cochrane Library.
  2. A literature search in a database, such as the Cumulative Index to Nursing and
    Allied Health Literature (CINAHL).
  3. An online nursing textbook.
  4. The policy and procedure manual at the health care agency.
    1

. The Cochrane Library provides systematic reviews of health care
interventions and will provide the best resource for evidence for nursing care. CINAHL
offers key word searches to published articles in nursing and allied health literature, but
not reviews. A nursing textbook has information about nursing care, which may include
evidence-based practices, but textbooks may not have the most up-to-date information.
While the policy and procedure manual may be based on evidence-based practices, the
most current practices will be found in evidence-based reviews of literature.

The nurse in the intensive care unit is giving a report to the nurse in the post
surgical unit about a client who had a gastrectomy. The most effective way to assure
essential information about the client is reported is to:

  1. Give the report face to face with both nurses in a quiet room.
  2. Audiotape the report for future reference and documentation.
  3. Use a printed checklist with information individualized for the client.
  4. Document essential transfer information in the client’s electronic health record
    3

Using an individualized printed checklist assures that all key information is
reported; the checklist can then serve as a record to which nurses can refer later. Giving
a verbal report leaves room for error in memory; using an audiotape or an electronic
health record requires nurses to spend unnecessary time retrieving information.

A client reports vomiting every hour for the past 8 to 10 hours. The nurse
should assess the client for risk of which of the following? Select all that apply.

  1. Metabolic acidosis.
  2. Metabolic alkalosis.
  3. Hypokalemia.
  4. Hyperkalemia.
  5. Hyponatremia.
    2,3

Gastric acid contains a substantial amount of potassium, hydrogen ions, and
chloride ions. Frequent vomiting can induce an excessive loss of these acids leading to
alkalosis. Excessive loss of potassium produces hypokalemia. Frequent vomiting does
not lead to the condition of too much potassium (hyperkalemia) or too little sodium

The nurse explains to the client with Hodgkin’s disease that a bone marrow
biopsy will be taken after the aspiration. What should the nurse explain about the
biopsy?

  1. “Your biopsy will be performed before the aspiration because enough tissue may
    be obtained so that you won’t have to go through the aspiration.”
  2. “You will feel a pressure sensation when the biopsy is taken but should not feel
    actual pain; if you do, tell the doctor so that you can be given extra numbing
    medicine.”
  3. “You may hear a crunch as the needle passes through the bone, but when the
    biopsy is taken, you will feel a suction-type pain that will last for just a
    moment.”
  4. “You will be shaved and cleaned with an antiseptic agent, after which the doctor
    will inject a needle without making an incision to aspirate out the bone marrow.”
    2

A biopsy needle is inserted through a separate incision in the anesthetized
area. The client will feel a pressure sensation when the biopsy is taken but should not
feel actual pain. The client should be instructed to inform the physician if pain is felt so
that more anesthetic agent can be administered to keep the client comfortable. The
biopsy is performed after the aspiration and from a slightly different site so that the
tissue is not disturbed by either test. The client will feel a suction-type pain for a
moment when the aspiration is being performed, not the biopsy. A small incision is
made for the biopsy to accommodate the larger-bore needle. This may require a stitch

A client with advanced Hodgkin’s disease is admitted to hospice because
death is imminent. The goal of nursing care at this time is to:

  1. Reduce the client’s fear of pain.
  2. Support the client’s wish to discontinue further therapy.
  3. Prevent feelings of isolation.
  4. Help the client overcome feelings of social inadequacy.
    3

Terminally ill clients most often describe feelings of isolation because they
tend to be ignored, they are often left out of conversations (especially those dealing with
the future), and they sense the attitudes of discomfort that many people feel in their
presence. Helpful nursing measures include taking the time to be with the client, offering
opportunities to talk about feelings, and answering questions honestly.

The client is a survivor of non-Hodgkin’s lymphoma. Which of the following
statements indicates the client needs additional information?

  1. “Regular screening is very important for me.”
  2. “The survivor rate is directly proportional to the incidence of second
    malignancy.”
  3. “The survivor rate is indirectly proportional to the incidence of second
    malignancy.”
  4. “It is important for survivors to know the stage of the disease and their current
    treatment plan.”
    2

It is incorrect that the survivor rate is directly proportional to the incidence
of second malignancy. The survivor rate is indirectly proportional to the incidence of
second malignancy, and regular screening is very important to detect a second
malignancy, especially acute myeloid leukemia or myelodysplastic syndrome. Survivors
should know the stage of the disease and their current treatment plan so that they can
remain active participants in their health care.

Which of the following is the most important goal of nursing care for a client
who is in shock?

  1. Manage fluid overload.
  2. Manage increased cardiac output.
  3. Manage inadequate tissue perfusion.
  4. Manage vasoconstriction of vascular beds
    3

Nursing interventions and collaborative management are focused on
correcting and maintaining adequate tissue perfusion. Inadequate tissue perfusion may
be caused by hemorrhage, as in hypovolemic shock; by decreased cardiac output, as in
cardiogenic shock; or by massive vasodilation of the vascular bed, as in neurogenic,
anaphylactic, and septic shock. Fluid deficit, not fluid overload, occurs in shock

Which of the following indicates hypovolemic shock in a client who has had a
15% blood loss?

  1. Pulse rate less than 60 bpm.
  2. Respiratory rate of 4 breaths/min.
  3. Pupils unequally dilated.
  4. Systolic blood pressure less than 90 mm Hg.
    4

Typical signs and symptoms of hypovolemic shock include systolic blood
pressure less than 90 mm Hg, narrowing pulse pressure, tachycardia, tachypnea, cool
and clammy skin, decreased urine output, and mental status changes, such as irritability
or anxiety. Unequal dilation of the pupils is related to central nervous system injury or
possibly to a previous history of eye injury.

Which of the following findings is the best indication that fluid replacement
for the client in hypovolemic shock is adequate?

  1. Urine output greater than 30 mL/h.
  2. Systolic blood pressure greater than 110 mm Hg.
  3. Diastolic blood pressure greater than 90 mm Hg.
  4. Respiratory rate of 20 breaths/min.
    1

Urine output provides the most sensitive indication of the client’s response to
therapy for hypovolemic shock. Urine output should be consistently greater than 35
mL/h. Blood pressure is a more accurate reflection of the adequacy of vasoconstriction
than of tissue perfusion. Respiratory rate is not a sensitive indicator of fluid balance in
the client recovering from hypovolemic shock.

Which of the following is a risk factor for hypovolemic shock?

  1. Hemorrhage.
  2. Antigen-antibody reaction.
  3. Gram-negative bacteria.
  4. Vasodilation.
    1

Causes of hypovolemic shock include external fluid loss, such as
hemorrhage; internal fluid shifting, such as ascites and severe edema; and dehydration.
Massive vasodilation is the initial phase of vasogenic or distributive shock, which can
be further subdivided into three types of shock: septic, neurogenic, and anaphylactic. A
severe antigen-antibody reaction occurs in anaphylactic shock. Gram-negative bacterial
infection is the most common cause of septic shock. Loss of sympathetic tone
(vasodilation) occurs in neurogenic shock.

Which is a priority assessment for the client in shock who is receiving an IV
infusion of packed red blood cells and normal saline solution?

  1. Fluid balance.
  2. Anaphylactic reaction.
  3. Pain.
  4. Altered level of consciousness.
    2

The client who is receiving a blood product requires astute assessment for
signs and symptoms of allergic reaction and anaphylaxis, including pruritus (itching),
urticaria (hives), facial or glottal edema, and shortness of breath. If such a reaction
occurs, the nurse should stop the transfusion immediately, but leave the IV line intact,
and notify the physician. Usually, an antihistamine such as diphenhydramine
hydrochloride (Benadryl) is administered. Epinephrine and corticosteroids may be
administered in severe reactions. Fluid balance is not an immediate concern during the
blood administration. The administration should not cause pain unless it is extravasating
out of the vein, in which case the IV administration should be stopped. Administration
of a unit of blood should not affect the level of consciousness.

The client who does not respond adequately to fluid replacement has a
prescription for an IV infusion of dopamine hydrochloride at 5 mcg/kg/min. To
determine that the drug is having the desired effect, the nurse should assess the client
for:

  1. Increased renal and mesenteric blood flow.
  2. Increased cardiac output.
  3. Vasoconstriction.
  4. Reduced preload and afterload.
    2

At medium doses (4 to 8 mcg/kg/min), dopamine hydrochloride slightly
increases the heart rate and improves contractility to increase cardiac output and
improve tissue perfusion. When given at low doses (0.5 to 3.0 mcg/kg/min), dopamine
increases renal and mesenteric blood flow. At high doses (8 to 10 mcg/kg/min),
dopamine produces vasoconstriction, which is an undesirable effect. Dopamine is not
given to affect preload and afterload.

A client is receiving dopamine hydrochloride for treatment of shock. The
nurse should:

  1. Administer pain medication concurrently.
  2. Monitor blood pressure continuously.
  3. Evaluate arterial blood gases at least every 2 hours.
  4. Monitor for signs of infection.
    2

The client who is receiving dopamine hydrochloride requires continuous
blood pressure monitoring with an invasive or noninvasive device. The nurse may
titrate the IV infusion to maintain a systolic blood pressure of 90 mm Hg. Administration
of a pain medication concurrently with dopamine hydrochloride, which is a potent
sympathomimetic with dose-related alpha-adrenergic agonist, beta 1-selective
adrenergic agonist, and dopaminergic blocking effects, is not an essential nursing action
for a client who is in shock with already low hemodynamic values. Arterial blood gas
concentrations should be monitored according to the client’s respiratory status and acidbase
balance status and are not directly related to the dopamine hydrochloride dosage.
Monitoring for signs of infection is not related to the nursing action for the client
receiving dopamine hydrochloride.

A client who has been taking warfarin has been admitted with severe acute
rectal bleeding and the following laboratory results: International Normalized Ratio
(INR), 8; hemoglobin, 11 g/dL (110 g/L); and hematocrit, 33% (0.33). In which order
should the nurse implement the following physician prescriptions?

  1. Give 1 unit fresh frozen plasma (FFP).
  2. Administer vitamin K 2.5 mg by mouth.
  3. Schedule client for sigmoidoscopy.
  4. Administer IV dextrose 5% in 0.45% normal saline solution.
    4,1,2,3

Analysis of the client’s laboratory results indicate that an INR of 8 is increased
beyond therapeutic ranges. The client is also experiencing severe acute rectal bleeding
and has a hemoglobin level in the low range of normal and a hematocrit reflecting fluid
volume loss. The nurse should first establish an IV line and administer the dextrose in
saline. Next the nurse should administer the FFP. FFP contains concentrated clotting
factors and provides an immediate reversal of the prolonged INR. Vitamin K 2.5 mg PO
should be given next because it reverses the warfarin by returning the PT to normal
values. However, the reversal process occurs over 1 to 2 hours. Last, the nurse can
schedule the client for the sigmoidoscopy.

When assessing a client for early septic shock, the nurse should assess the
client for which of the following?

  1. Cool, clammy skin.
  2. Warm, flushed skin.
  3. Increased blood pressure.
  4. Hemorrhage.
    2

Warm, flushed skin from a high cardiac output with vasodilation occurs in
warm shock or the hyperdynamic phase (first phase) of septic shock. Other signs and
symptoms of early septic shock include fever with restlessness and confusion; normal or
decreased blood pressure with tachypnea and tachycardia; increased or normal urine
output; and nausea and vomiting or diarrhea. Cool, clammy skin occurs in the
hypodynamic or cold phase (later phase). Hemorrhage is not a factor in septic shock.

A client with toxic shock has been receiving ceftriaxone sodium (Rocephin), 1
g every 12 hours. In addition to culture and sensitivity studies, which other laboratory
findings should the nurse monitor?

  1. Serum creatinine.
  2. Spinal fluid analysis.
  3. Arterial blood gases.
  4. Serum osmolality.
    1

The nurse monitors the blood levels of antibiotics, white blood cells, serum
creatinine, and blood urea nitrogen because of the decreased perfusion to the kidneys,
which are responsible for filtering out the Rocephin. It is possible that the clearance of
the antibiotic has been decreased enough to cause toxicity. Increased levels of these
laboratory values should be reported to the physician immediately. A spinal fluid
analysis is done to examine cerebral spinal fluid, but there is no indication of central
nervous system involvement in this case. Arterial blood gases are used to determine
actual blood gas levels and assess acid-base balance. Serum osmolality is used to
monitor fluid and electrolyte balance.

Which nursing intervention is most important in preventing septic shock?

  1. Administering IV fluid replacement therapy as prescribed.
  2. Obtaining vital signs every 4 hours for all clients.
  3. Monitoring red blood cell counts for elevation.
  4. Maintaining asepsis of indwelling urinary catheters.
    4

Which of the following is an indication of a complication of septic shock?

  1. Anaphylaxis.
  2. Acute respiratory distress syndrome (ARDS).
  3. Chronic obstructive pulmonary disease (COPD).
  4. Mitral valve prolapse.
    2

A nurse has two middle-aged clients who have a prescription to receive a
blood transfusion of packed red blood cells at the same time. The first client’s blood
pressure dropped from the preoperative value of 120/80 mm Hg to a postoperative
value of 100/50. The second client is hospitalized because he developed dehydration
and anemia following pneumonia. After checking the patency of their IV lines and vital
signs, what should the nurse do next?

  1. Call for both clients’ blood transfusions at the same time.
  2. Ask another nurse to verify the compatibility of both units at the same time.
  3. Call for and hang the first client’s blood transfusion.
  4. Ask another nurse to call for and hang the blood for the second client.
    4

The nurse identifies deficient knowledge when the client undergoing induction
therapy for leukemia makes which of the following statements?

  1. “I will pace my activities with rest periods.”
  2. “I can’t wait to get home to my cat!”
  3. “I will use warm saline gargle instead of brushing my teeth.”
  4. “I must report a temperature of 100°F (37.7°C).”
    2

The nurse identifies that the client does not understand that contact with
animals must be avoided because they carry infection and the induction therapy will
destroy the client’s white blood cells (WBCs). The induction therapy will cause anemia,
and the client will experience fatigue and will have to pace activities with rest periods.
Platelet production will be decreased, and the client will be at risk for bleeding
tendencies; oral hygiene will have to be provided by using a warm saline gargle instead
of brushing the teeth and gums. The client will be at risk for infection owing to the
decrease in WBC production and should report a temperature of 100°F (37.8°C) or
higher.

. A client with acute myeloid leukemia (AML) reports overhearing one of the
other clients say that AML had a very poor prognosis. The client has understood that the
client’s physician informed the client that his physician told him that he has a good
prognosis. Which is the nurse’s best response?

  1. “You must have misunderstood. Who did you hear that from?”
  2. “AML does have a very poor prognosis for poorly differentiated cells.”
  3. “AML is the most common nonlymphocytic leukemia.”
  4. “Your doctor stated your prognosis based on the differentiation of your cells.”
    4

The statement “Your doctor stated your prognosis based on the differentiation
of your cells” addresses the client’s situation on an individual basis. The nurse is
clarifying that clients have different prognoses—even though they may have the same
type of leukemia—because of the cell differentiation. Stating that the client
misunderstood is inappropriate for an advocate of the client and serves no useful
purpose. The other statements are true but do not address this client’s individual
concern.

The goal of nursing care for a client with acute myeloid leukemia (AML) is to
prevent:

  1. Cardiac arrhythmias.
  2. Liver failure.
  3. Renal failure.
  4. Hemorrhage.
    4

Bleeding and infection are the major complications and causes of death for
clients with AML. Bleeding is related to the degree of thrombocytopenia, and infection
is related to the degree of neutropenia. Cardiac arrhythmias rarely occur as a result of
AML. Liver or renal failure may occur, but neither is a major cause of death in AML.

The nurse is assessing a client with chronic myeloid leukemia (CML). The
nurse should assess the client for:

  1. Lymphadenopathy.
  2. Hyperplasia of the gum.
  3. Bone pain from expansion of marrow.
  4. Shortness of breath
    4

Although the clinical manifestations of CML vary, clients usually have
confusion and shortness of breath related to decreased capillary perfusion to the brain
and lungs. Lymphadenopathy is rare in CML. Hyperplasia of the gum and bone pain are
clinical manifestations of AML.

Which of the following individuals is most at risk for acquiring acute
lymphocytic leukemia (ALL)? The client who is:

  1. 4 to 12 years.
  2. 20 to 30 years.
  3. 40 to 50 years.
  4. 60 to 70 years.
    1

The peak incidence of ALL is at 4 years of age. ALL is uncommon after 15
years of age. The median age at incidence of CML is 40 to 50 years. The peak incidence
of AML occurs at 60 years of age. Two-thirds of cases of chronic lymphocytic leukemia
occur in clients older than 60 years of age.

The client with acute lymphocytic leukemia (ALL) is at risk for infection. The
nurse should:

  1. Place the client in a private room.
  2. Have the client wear a mask.
  3. Have staff wear gowns and gloves.
  4. Restrict visitors
    1

Clients with ALL are at risk for infection due to granulocytopenia. The nurse
should place the client in a private room. Strict hand-washing procedures should be
enforced and will be the most effective way to prevent infection. It is not necessary to
have the client wear a mask. The client is not contagious and the staff does not need to
wear gloves. The client can have visitors; however, they should be screened for
infection and use hand-washing procedures.

In assessing a client in the early stage of chronic lymphocytic leukemia (CLL),
the nurse should determine if the client has:

  1. Enlarged, painless lymph nodes.
  2. Headache.
  3. Hyperplasia of the gums.
  4. Unintentional weight loss.
    4

Clients with CLL develop unintentional weight loss; fever and drenching night
sweats; enlarged, painful lymph nodes, spleen, and liver; decreased reaction to skin
sensitivity tests (anergy); and susceptibility to viral infections. Enlarged, painless lymph
nodes are a clinical manifestation of Hodgkin’s lymphoma. A headache would not be
one of the early signs and symptoms expected in CLL because CLL does not cross the
blood-brain barrier and would not irritate the meninges. Hyperplasia of the gums is a
clinical manifestation of AML.

The nurse is planning care with a client with acute leukemia who has mucositis.
The nurse should advise the client that after every meal and every 4 hours while awake
the client should use:

  1. Lemon-glycerin swabs.
  2. A commercial mouthwash.
  3. A saline solution.
  4. A commercial toothpaste and brush
    3

Simple rinses with saline or a baking soda and water solution are effective
and moisten the oral mucosa. Commercial mouthwashes and lemon-glycerin swabs
contain glycerin and alcohol, which are drying to the mucosa and should be avoided.
Brushing after each meal is recommended, but every 4 hours may be too traumatic.
During acute leukemia, the neutrophil and platelet counts are often low and a soft-bristle
toothbrush, instead of the client’s usual brush, should be used to prevent bleeding gums.

The client with acute leukemia and the health care team establish mutual client
outcomes of improved tidal volume and activity tolerance. Which measure would be
least likely to promote outcome achievement?

  1. Ambulating in the hallway.
  2. Sitting up in a chair.
  3. Lying in bed and taking deep breaths.
  4. Using a stationary bicycle in the room.
    3

The client with acute leukemia experiences fatigue and deconditioning. Lying
in bed and taking deep breaths will not help achieve the goals. The client must get out of
bed to increase activity tolerance and improve tidal volume. Ambulating in the hall
(using a HEPA filter mask if neutropenic) is a sensible activity and helps improve
conditioning. Sitting up in a chair facilitates lung expansion. Using a stationary bicycle
in the room allows the client to increase activity as tolerated.

The nurse is evaluating the client’s learning about combination chemotherapy.
Which of the following statements by the client about reasons for using combination
chemotherapy indicates the need for further explanation?

  1. “Combination chemotherapy is used to interrupt cell growth cycle at different
    points.”
  2. “Combination chemotherapy is used to destroy cancer cells and treat side effects
    simultaneously.”
  3. “Combination chemotherapy is used to decrease resistance.”
  4. “Combination chemotherapy is used to minimize the toxicity from using high
    doses of a single agent.”
    2

Combination chemotherapy does not mean two groups of drugs, one to kill the
cancer cells and one to treat the adverse effects of the chemotherapy. Combination
chemotherapy means that multiple drugs are given to interrupt the cell growth cycle at
different points, decrease resistance to a chemotherapy agent, and minimize the toxicity
associated with use of a high dose of a single agent (ie, by using multiple agents with
different toxicities).

In providing care to the client with leukemia who has developed
thrombocytopenia, the nurse assesses the most common sites for bleeding. Which of the
following is not a common site?

  1. Biliary system.
  2. Gastrointestinal tract.
  3. Brain and meninges.
  4. Pulmonary system.
    1

The biliary system is not especially prone to hemorrhage. Thrombocytopenia
(a low platelet count) leaves the client at risk for a potentially life-threatening
spontaneous hemorrhage in the gastrointestinal, respiratory, and intracranial cavities.

The nurse’s best explanation for why the severely neutropenic client is placed
in reverse isolation is that reverse isolation helps prevent the spread of organisms:

  1. To the client from sources outside the client’s environment.
  2. From the client to health care personnel, visitors, and other clients.
  3. By using special techniques to dispose of contaminated materials.
  4. By using special techniques to handle the client’s linens and personal items.
    1

The primary purpose of reverse isolation is to reduce transmission of
organisms to the client from sources outside the client’s environment.

Which of the following clinical manifestations does the nurse most likely
observe in a client with Hodgkin’s disease?

  1. Difficulty swallowing.
  2. Painless, enlarged cervical lymph nodes.
  3. Difficulty breathing.
  4. A feeling of fullness over the liver.
    2

Painless and enlarged cervical lymph nodes, tachycardia, weight loss,
weakness and fatigue, and night sweats are signs of Hodgkin’s disease. Difficulty
swallowing and breathing may occur, but only with mediastinal node involvement.
Hepatomegaly is a late-stage manifestation.

A client with a suspected diagnosis of Hodgkin’s disease is to have a lymph
node biopsy. The nurse should make sure that personnel involved with the procedure do
which of the following when obtaining the lymph node biopsy specimen for histologic
examination for this client?

  1. Maintain sterile technique.
  2. Use a mask, gloves, and a gown when assisting with the procedure.
  3. Send the specimen to the laboratory when someone is available to take it.
  4. Ensure that all instruments used are placed in a sealed and labeled container.
    1

The nurse must ensure that sterile technique is used when a biopsy is obtained
because the client is at high risk for infection. In most cases, a lymph node biopsy is sent
immediately to the laboratory once it is placed in a specific solution in a closed
container. It is not necessary to wear a gown and mask when obtaining the specimen. It
is not necessary to use special handling procedures for the instruments used.

The client with Hodgkin’s disease undergoes an excisional cervical lymph node
biopsy under local anesthesia. After the procedure, which does the nurse assess first?

  1. Vital signs.
  2. The incision.
  3. The airway.
  4. Neurologic signs.
    3

Assessing for an open airway is always first. The procedure involves the
neck; the anesthesia may have affected the swallowing reflex, or the inflammation may
have closed in on the airway, leading to ineffective air exchange. Once a patent airway
is confirmed and an effective breathing pattern established, the circulation is checked.
Vital signs and the incision are assessed as soon as possible, but only after it is
established that the airway is patent and the client is breathing normally. A neurologic
assessment is completed as soon as possible after other important assessments.

When assessing the client with Hodgkin’s disease, the nurse should observe the
client for which of the following findings?

  1. Herpes zoster infections.
  2. Discolored teeth.
  3. Hemorrhage.
  4. Hypercellular immunity.
    1

Herpes zoster infections are common in clients with Hodgkin’s disease.
Discoloring of the teeth is not related to Hodgkin’s disease but rather to the ingestion of
iron supplements or some antibiotics such as tetracycline. Mild anemia is common in
Hodgkin’s disease, but the platelet count is not affected until the tumor has invaded the
bone marrow. A cellular immunity defect occurs in Hodgkin’s disease in which there is
little or no reaction to skin sensitivity tests. This is called anergy.

The client with Hodgkin’s disease develops B symptoms. These manifestations
indicate which of the following?

  1. The client has a low-grade fever (temperature lower than 100°F [37.8°C]).
  2. The client has a weight loss of 5% or less of body weight.
  3. The client has night sweats.
  4. The client probably has not progressed to an advanced stage.
    3

A temperature higher than 100.4°F (38°C), profuse night sweats, and an
unintentional weight loss of 10% of body weight represent the cluster of clinical
manifestations known as the B symptoms. Forty percent of clients with Hodgkin’s
disease have B symptoms, and B symptoms are more common in advanced stages of the
disease.

The client probably has not progressed to an advanced stage.

  1. The nurse is developing a discharge plan about home care with a client who has
    lymphoma. The nurse should emphasize which of the following?
  2. Use analgesics as needed.
  3. Take a shower with perfumed shower gel.
  4. Wear a mask when outside of the home.
  5. Take an antipyretic every morning.
    1

Analgesics are used as needed to relieve painful encroachment of enlarged
lymph nodes. Perfumed shower gel will increase pruritus. Wearing a mask does not
protect the client from infection if pathogens are not spread by airborne droplets.
Antipyretics should be used to treat fever symptomatically after infection is ruled out.

The client asks the nurse to explain what it means that his Hodgkin’s disease is
diagnosed at stage 1A. Which of the following describes the involvement of the
disease?

  1. Involvement of a single lymph node.
  2. Involvement of two or more lymph nodes on the same side of the diaphragm.
  3. Involvement of lymph node regions on both sides of the diaphragm.
  4. Diffuse disease of one or more extralymphatic organs
    1

In the staging process, the designations A and B signify that symptoms were or
were not present when Hodgkin’s disease was found, respectively. The Roman numerals
I through IV indicate the extent and location of involvement of the disease. Stage I
indicates involvement of a single lymph node; stage II, two or more lymph nodes on the
same side of the diaphragm; stage III, lymph node regions on both sides of the
diaphragm; and stage IV, diffuse disease of one or more extralymphatic organs.

A client is undergoing a bone marrow aspiration and biopsy. What is the best
way for the nurse to help the client and two upset family members handle anxiety during
the procedure?

  1. Allow the client’s family to stay as long as possible.
  2. Stay with the client without speaking.
  3. Encourage the client to take slow, deep breaths to relax.
  4. Allow the client time to express feelings.
    3

Encouraging the client to take slow, deep breaths during uncomfortable parts
of procedures is the best method of decreasing the stress response of tightening and
tensing the muscles. Slow, deep breathing affects the level of carbon dioxide in the
brain to increase the client’s sense of well-being. Allowing the client’s family to stay
may be appropriate if the family has a calming effect on the client, but this family is
upset and may contribute to the client’s stress. Silence can be therapeutic, but when the
client is faced with a potentially life-threatening diagnosis and a new, invasive
procedure, taking deep breaths will be more effective in reducing the stress response.
Expressing feelings is important, but deep breathing will promote relaxation; the nurse
can encourage the client to express feelings when the procedure is completed.

The nurse is completing a health assessment of a 42-year-old female with
suspected Graves’ disease. The nurse should assess this client for:

  1. Anorexia.
  2. Tachycardia.
  3. Weight gain.
  4. Cold skin.
    2

Graves’ disease, the most common type of thyrotoxicosis, is a state of
hypermetabolism. The increased metabolic rate generates heat and produces tachycardia
and fine muscle tremors. Anorexia is associated with hypothyroidism. Loss of weight,
despite a good appetite and adequate caloric intake, is a common feature of
hyperthyroidism. Cold skin is associated with hypothyroidism.

When conducting a health history with a female client with thyrotoxicosis, the
nurse should ask about which of the following changes in the menstrual cycle?

  1. Dysmenorrhea.
  2. Metrorrhagia.
  3. Oligomenorrhea.
  4. Menorrhagia.
    3.

A change in the menstrual interval, diminished menstrual flow
(oligomenorrhea), or even the absence of menstruation (amenorrhea) may result from the
hormonal imbalances of thyrotoxicosis. Oligomenorrhea in women and decreased libido
and impotence in men are common features of thyrotoxicosis. Dysmenorrhea is painful
menstruation. Metrorrhagia, blood loss between menstrual periods, is a symptom of
hypothyroidism. Menorrhagia, excessive bleeding during menstrual periods, is a
symptom of hypothyroidism.

A 34-year-old female is diagnosed with hypothyroidism. The nurse should assess
the client for which of the following? Select all that apply.

  1. Rapid pulse.
  2. Decreased energy and fatigue.
  3. Weight gain of 10 lb (4.5 kg).
  4. Fine, thin hair with hair loss.
  5. Constipation.
  6. Menorrhagia.
    2, 3, 5, 6.

Clients with hypothyroidism exhibit symptoms indicating a lack of
thyroid hormone. Bradycardia, decreased energy and lethargy, memory problems,
weight gain, coarse hair, constipation, and menorrhagia are common signs and
symptoms of hypothyroidism.

Propylthiouracil (PTU) is prescribed for a client with Graves’ disease. The nurse
should teach the client to immediately report which of the following?

  1. Sore throat.
  2. Painful, excessive menstruation.
  3. Constipation.
  4. Increased urine output.
    1

The most serious adverse effects of PTU are leukopenia and agranulocytosis,
which usually occur within the first 3 months of treatment. The client should be taught to
promptly report to the health care provider signs and symptoms of infection, such as a
sore throat and fever. Clients having a sore throat and fever should have an immediate
white blood cell count and differential performed, and the drug must be withheld until
the results are obtained. Painful menstruation, constipation, and increased urine output
are not associated with PTU therapy.

A client with thyrotoxicosis says to the nurse, “I am so irritable. I am having
problems at work because I lose my temper very easily.” Which of the following
responses by the nurse would give the client the most accurate explanation of her
behavior?

  1. “Your behavior is caused by temporary confusion brought on by your illness.”
  2. “Your behavior is caused by the excess thyroid hormone in your system.”
  3. “Your behavior is caused by your worrying about the seriousness of your
    illness.”
    2

A typical sign of thyrotoxicosis is irritability caused by the high levels of
circulating thyroid hormones in the body. This symptom decreases as the client responds
to therapy. Thyrotoxicosis does not cause confusion. The client may be worried about
her illness, and stress may influence her mood; however, irritability is a common
symptom of thyrotoxicosis and the client should be informed of that fact rather than
blamed.

The nurse is evaluating a client with hyperthyroidism who is taking
Propylthiouracil (PTU) 100 mg/day in three divided doses for maintenance therapy.
Which of the following statements from the client indicates the desired outcome of the
drug?

  1. “I have excess energy throughout the day.”
  2. “I am able to sleep and rest at night.”
  3. “I have lost weight since taking this medication.”
  4. “I do perspire throughout the entire day.”
    2.
    PTU is a prototype of thioamide antithyroid drugs. It inhibits production of
    thyroid hormones and peripheral conversion of T4 to the more active T3. A client taking
    this antithyroid drug should be able to sleep and rest well at night since the level of
    thyroid hormones is reduced in the blood. Excess energy throughout the day, loss of
    weight and perspiring through the day are symptoms of hyperthyroidism indicating the
    drug has not produced its outcome.

HESI MED SURG VERSION 1 2020/2021 Questions & Answers

1.

What instruction should the nurse include in the discharge teaching plan

of a client who had a cataract extraction today?

a.

Sexual activities may be resumed upon return home

b.

Light housekeeping is permitted but avoid heavy lifting

c.

Use a metal eye shield on operative eye during the day

d.

Administer eye ointment before applying eye drops

2.

A male adult comes to the urgent care clinic 5 days after being diagnose with

influenza. He is short ofbreath, febrile, and coughing green colored

sputum. Which intervention should the nurse implement first?

a.

Obtain a sputum sample for culture

b.

Check his oxygen saturation level

c.

Administer an oral antipyretic

d.

Auscultate bilateral lung sound

3.

An elder male client tells the nurse that he is loosing sleep because he

has to get up several times at night to go to the bathroom that he has

trouble starting his urinary stream and that he does not feel like his

bladder is ever completelyempty. Which intervention should the nurse

implement?

a.

collect a urine specimen for culture analysis

b.

obtain a fingerstick blood glucose level

c.

palpate the bladder above the symphysis pubis

d.

review the client fluid intake

4.

An adult client is admitted with diabetic ketoacidosis (DKA) and a urinary

tract infection (UTI) Prescriptions for intravenous antibiotics and insulin

infusion are initiated. Which serum laboratory value warrants the most

immediate intervention by the nurse?

a.

blood ph of 7.30

b.

glucose of 350 mg /dl

c.

white blood cell count of 15000mm

d.

potassium of 2.5 meq/l

5.

A client with sickle cell anemia develops a fever during the last hour of

administrationofaunitofpackedredbloodcell.Whennotifyingthe

healthcareproviderwhatinformationshouldthenurseprovidefirst

using the SBAR communication process?

a.

explain specific reason for urgent notification

b.

preface the report by stating the clients name and admitting

diagnosis

c.

communicate the pre-transfusion temperatures

d.

optain prn prescription for acetaminophen for fever 101f

6.

An adult male client is admitted for pneumocystis carinil pneumonia

(PCP) secondary to aids. While hospitalize he receives IV pentamidine

isethionate therapy. In preparing this client for discharge what important

aspect regarding his medication therapy should the nurse explain?

a.

AZT therapy must be stopped when IV aerosol pentamine is being

used.

b.

IV pentamine will be given until oral pentamine can be tolerated

c.

It will be necessary to continue prophylactic doses of IV or

aerosol pentamine everymonth

d.

Iv pentamine may offer protection to others aids related conditions

such as kaposis sarcoma

7.

A client subjective data includes dysuria, urgency, and urinary frequency.

What action should the nurse implement next?

a.

collect a clean catch specimen

b.

palpate the suprapubic region

c.

instruct to wipe from front to back

d.

inquire about recent sexual activity

8.

A client tells the nurse that her biopsy results indicate that the cancer

cells are well differentiated How should the nurse respond?

a.

offer the client reassurance that this information indicates that the

clients cancer cells are benign

b.

explain that these tissue cells often respond more effectively to

radiation than to chemotherapy

c.

ask the client in the healthcare provider has giving her any

information about the classification of her cancer

d.

help the client make plans to begin inmediate treatment since her

canceris likely to spread quickly

9.

A client with a chronic kidney disease is treated on hemodialysis. During

the 1 treatment clients blood pressure drops from 150/90 to 80/30 Which

action should the nurse take first?

a.

monitor bp q45 minutes

b.

lower the head of the chair and elevatefeet

c.

stop dialysis treatment

d.

administer 5%albumin IV

10.

A client with deep vain thrombosis (DVT) is receiving a continues infusion

of heparin sodium 25,000 unit in 5% dextrose injection 250ml. The

prescription indicates the dosage should be increase 900 units/hr. The

nurse should program the infusion pump to deliver how many ml/hr?

=9

11.

The nurse is obtaining the admission history for a client with suspected

peptic ulcer disease (PUD). Which subjective data reported bythe client

supports this diagnosis?

a.

upper mid abdominal gnawing and burning pain

b.

severe abdominal cramps and diarrhea after eating spicy foods

c.

marked loss of weight and appetite over the last few months

d.

use of chewable and liquid antacids for indigestion

12.

The nurse is providing preoperative education for a jewish client

schedule to receive a xenograft graft to promote burn healing. Which

information should the nurse provide this client?

a.

the xenograft is taken from nonhuman sources

b.

grafting increases the risk for bacterial infection

c.

as the burn heals the graft permanently attaches

d.

grafts are later removed by debriding procedure

13.

A client who took a camping vacation two weeks ago in a country with a

tropical climate comes to the clinic describing vague symptoms and

diarrhea for the past week. Which finding is most important for the nurse

to report?

a.

jaundice sclera

b.

intestinal cramping

c.

weakness and fatigue

d.

weight loss

14.

During a home visit the nurse assesses the skin of a client with eczema

who reports than an exacerbation of symptoms has occurred during the

last week. Which information is most useful in determining the possible

cause of the symptoms?

a.

an old friend with eczema came for visit

b.

recently received an influenza immunization

c.

corticosteroid cream was applied to eczema

d.

a grandson and his new dog recently visited

15.

When explaining dietary guidelines to a client with acute

glomerulonephritis (AGN) which instruction should the nurse include in

the dietary teaching?

a.

select a protein rich food daily

b.

restrict sodium intake

c.

eat high potassium foods

d.

Avoid foods high in carbohydrate

16.

A male client who is 24hr post operative for an exploratory laparoctomy

complains that he is starving because he has had no real food since before

surgery. Prior to advancing his diet which intervention should the nurse

implememt?

a.

discontinue intravenous therapy

b.

Assess for abdominal distension and tenderness

c.

Obtain a prescription for a diet change

d.

Auscultate bowel sound in all four quadrants

17.

A client diagnose with stable angina secondary to ischemic heart disease

has a prescription for sublingual (SL) nitroglycerin (NTG). The nurse should

tell the client to follow which instructions if chest pain is not relieved after

taking 3 NTG tables 5min apart?

a.

drive to the nearest emergency department

b.

takeanother NTG SL tablet and lie downuntil angina subsides

c.

call primaryhealthcare provider

d.

call 911 pain is unrelieved and chew a tablet of aspirin 325mg

18.

After taking orlistat (Xenical) for one week a femela client tells the home

health nurse that she is experiencing increasingly frequent oily stools and

flatus. What action should the nurse take?

a.

obtain stool specimen to evaluate for occult blood and fat content

b.

instruct the client to increase her intake of saturated fats over the

next week

c.

ask the client to describe her dietary intake history for the last

several days

d.

advice the client to stop taking the drug and contact the healthcare

provider

19.

Two days after an abscess of the chin was drained the client returns to

the clinic with fever chills and a maculopapular rash with pruritis. The

client has taken an oral antibiotic and cleansed the wound today with

provide iodine (Betadine) solution.Which intervention should the nurse

implement first?

a.

determine if the client has a history of diabetes

b.

assess airway patency and oxygen saturation

c.

review recent medication history and allergies ( POSSIBLE ANSWER

TOO)

d.

obtain samples for complete blood count and cultures

20.

A client experiences an ABO incompatibility reaction after multiple blood

transfusions. Which finding should the nurse report immediately to the

health care provider?

a.

low back pain and hypotension

b.

rhinitis and nasal stuffiness

c.

delayed painful rash with urticarial

d.

arthritic joint changes and chronic pain

21.

A young adult male who has had type 2 diabetes mellitus (DM) is admitted

to the intensive care unit with hyperglycemic nonketotic syndrome (HHNS).

A sliding scale protocol for an isotonic IV solution with regular insulin is

prescribed based on the results of acontinuous blood glucosemonitoring

device that is attached to the client’s central venous catheter.When the

client’s respirations become labored and his lungs sound indicate crackles

what actionshould the nurse take?

a.

collect a specimen for a white blood cell count and cultures

b.

determine the clients glycosylated hemoglobin (A1C) (POSSIBLE

ANSWER)

c.

administer insulin IV push until the clients fluid volume is adjusted

d.

decrease infusion rate to address fluid overload

22.

When preparing to apply a fentanyl(Duragesic) transdermal patch the

nurse notes that the previously applied patch is intact on the client’s

upper back and the client denies pain. What action should the nurse

take?

a.

Remove the patch and consult with the healthcare provider about

the client pain resolution

b.

Place the patch on the clients shoulder and leave both patches in

place for 12 hours

c.

Administer an oral analgesic and evaluate its effectiveness before

applying a new patch

d.

Apply a new patch in a different location after removing the

original patch

23.

A client who had a myocardial infarction is admitted to the coronary critical

care unit (CCU) with a nitroglycerin drip infusing. The clients last blood

pressure measurements was 78/36.What action should the nurse

implement?

a.

obtain blood pressure q5 minutes using duranap machine

b.

change the dilution of the nitroglycerin infusion

c.

reduce the rate of the nitroglycerin infusion

d.

begin dopamine infusion at 5mcg/kg per minute

24.

An adolescent is admitted to the hospital because of a suicide attempt

with an overdose of acetaminophen (Tylenol). Which blood values are

most important for the nurse to monitor during the first 72 hours

following ingestion of this overdose?

a.

BUN creatinine specific gravity

b.

White blood count, hemoglobin hematocrit

c.

PH,PCO2, HC03

d.

LDH OR LD, SGOT OR ALT, SGPT OR AST

25.

An elderly post-operative female client is receiving morphinesulfate via a

PCA pump. Which assessment finding should prompt a nurse to administer

the prescribed PRNmedication naloxone?

a.

her respiratory rate is 7 breath/minute

b.

she indicates that she feels as if she cannot get enough air to

breath

c.

she has intercostal retractions and bilateral wheezing is

auscultated

d.

her pulse oximeter is 89% on room air

26.

Which assessment finding indicates to the nurse that the muscarinic

agent bethanechol (Urecholine) is effective for a client diagnose with

urinary retention?

a.

urinary output equal to intake

b.

no terminal urinary dribbling

c.

denies stress incontinence

d.

absence of xerostomia

27.

Following involvement in a motor vehicle collision, a middle aged adult client

is admitted to the hospital with multiple facial fractures. The client’s blood

alcohol level is high on admission. Which PRN prescription should be

administer if the clients begins to exhibit signs and symptoms of delirium

tremens (DT s)?

a.

Lorazepam (Ativan) 2mg IM

b.

Chlorpromazine (thorazine) 50mg IM

c.

Prochlorperazine (Compazine) 5mgIM

d.

Hydromorphone (Dilaudid) 2mg IM

28.

Which instructions should the nurse include in the teaching plan of a

client who is taking the diuretic spironolactone (Aldactone)?

a.

call the healthcare provider f you develop gynecomastia

b.

Take the medication in the morning

c.

Avoid caffeine and smoking

d.

Increase your consumption of bananas and oranges

29.

A glucagon emergency kit is prescribed for a client with type 1 diabetes

mellitus. When should the nurse instruct the client to take the glucagon?

a.

after meals to increase endogenous insulin secretion

b.

after insulin administration to prevent hypoglycemia

c.

when recognized signs of severe hypoglycemia occur

d.

when unable to eat during sick days

30.

A client with hyperthyroidism is being treated with radioactive iodine (I-

131). Which explanation should be included in preparing this client for this

treatment?

a.

describe radioactive iodine as a tasteless, colorless medication

administered by the healthcare provider

b.

explain the need for using lead shields for 2 to 3 weeks after the

treatment

c.

describe the signs of goiter because this is a common side effects of

radioactive iodine

d.

explain that relief of the signs/ symptoms of hyperthyroidism will

occur immediately

31.

A female client is being treated for tuberculosis with rifampin (rifadin)

which statement indicates that further teaching is needed?

a.

I will take my usual contraceptive for birth control

32.

A client is discharged with a prescription for warfarin (Coumadin). What

discharge instructions should the nurse emphasize to the client?

a.

take a multi vitamin supplement daily

b.

use an astringent for superficial bleeding

c.

avoid going barefoot especially outside

d.

include large amounts of spinach in the diet

33.

In caring for a client with diabetes insipidus who is receiving an

antidiuretic hormone intranasal which serum lab test is most important

for the nurse to monitor?

a.

osmolality

b.

calcium

c.

platelets

d.

glucose

34.

After administering dihydroergotamine(Migranal) 1 mg subcutaneously to

a client with a severe migraine headache the nurse should explain that

relief can be expected within what time frame?

a.

2 hours

b.

5 minutes

c.

1 hour

d.

15 minutes

35.

Aclientwithhypertensionwhohasbeentakinglabetalolfortwoweeks,

reportsafivepound(2.2kg)weightgain.Whichfollowupassessmentis

most important for the nurse to obtain?

a.

capillary refill

b.

body temperature

c.

muscle strength

d.

breath sounds

36.

A male client is receiving pilocarpine hydrochloride (Isopto Carpine)

ophthalmic drops for glaucoma. He calls the clinic and ask the nurse why

he has difficulty seeing at night. What explanation should the nurse

provide?

a.

The eye drops slow pupil response to accommodate for darkness

b.

The dropsincrease the fluid in the eyes and cloud the visual field (

possible answer)

c.

The drug can cause lens to become more opaque

d.

The medication causes pupils to dilate which reduces night vision

37.

A client who is taking and oral dose of tetracyclinecomplains of

gastrointestinal upset. What snack should the nurse instruct the client to

take with the tetracycline?

a.

toasted wheat bread and jelly

b.

cheese and crackers

c.

cold cereal with skim milk

d.

fruit flavored yogurt

38.

The therapeutic effect of insulin in treating type 1 diabetes mellitus is

based on which physiologic action?

a.

Facilitates transport of glucose into the cell

b.

Increases intracellular receptor site sensitivity

c.

Stimulates function of beta cells in the pancreas

d.

Delays carbohydrates digestion and absorption

39.

The health care provider prescribe a medication for an older adult client

who is complaining of insomnia. And instructs the client to return in 2

weeks. The nurse should question which prescription?

a.

Eszoplicone (Lunesta)10 mg orally at bed time

b.

Zolpidem 10 mgorally at bed time

c.

Temazepan orally at bed time

d.

Ramelteon orally at bedtime

40.

A male client reports to the nurse that he is experiencing GI distress from

high dose of a corticosteroid and is planning to stop taking the

medication. In response to the client’s statement what nursing action is

most important for the nurse to take?

a.

Encourage the client to take medication with food to decrease GI

distress

b.

Advice the client that the medication should be stopped

gradually rather than abruptly.

c.

Review the clients dosing schedule to ensure he is taking the

prescribed amount

d.

Assess the client for other indication of adverse effects of

corticosteroid

41.

Fifteen minutes after receiving sulfa athenozole. A male client report a

burning sensation over his abdomen chest and groin. Which intervention

is

most important for the nurse to implement?

a.

Auscultate lung sounds for wheezing

b.

Review the clients list if drugs allergies

c.

Add sulfamethinozole to clients allergies

d.

Check neurological vital signs

42.

Antibiotic resistant organism are a major infection control problems. To

help minimize the emergence of resistant bacteria what instruction

should the nurse provide to the clients?

a.

stop taking prescribed antibiotics when symptoms decrease

b.

avoid using antibiotics when suffering from colds or the flu

c.

ask the healthcare provider to prescribe the newest antibiotic when

needed

d.

request a prescription for first time vancomysin for a sore throat

43.

A client with symptoms of influenzathat started the previous day ask the

clinic nurse about taking oseltamivir (Tamiflu) to treat the infection. Which

response should the nurse provide?

a.

Advise the client once symptoms occur is too late to receive an

influenza vaccination

b.

Refer the client to the healthcare provider at the clinic to obtain

a medication prescription

c.

Explain to the client that antibiotics are not useful in treating viral

infections such as influenza

d.

Instruct the client that over the counter medications are sufficient

to manage influenza symptoms

44.

Twenty minutes after the nurse starts a secondary IV infusion of cafepime

(maxipime) 2 grams using an infusion pump to deliver the dose in one

hour, the client reports feeling nauseated. What action should thenurse

implement?

a.

stop medication infusion and notify the healthcare provider of the

adverse effect

b.

increase the rate of the infusion to complete the dose of the

medication more rapidly

c.

continue the infusion and administer a prn antiemetic

prescription

d.

reassure the client that the nausea is not related to the iv infusion

45.

The nurse administer donepezil hydrochloride (Aricept) to a client with

Alzheimer’s disease as an intervention for which client problem?

a.

fluid volume excess

b.

disturbed thought processes

c.

chronic pain

d.

altered breathing patterns

46.

To prevent deep vein thrombosis following knee replacement surgery, an

adult male client is receiving enoxaparin (Lovenox) subcutaneously daily.

Which laboratory finding requires immediate action by the nurse?

a.

blood urea nitrogen (BUN) 20mg/dl or 7.1 mmol/L (SI)

b.

Hematocrit 45%

c.

Serum creatinine 1.0 mg/dl or 88.4 mol/L (SI)

d.

Platelet count of 100,000/mm3 or 100×10??/ L (SI)

47.

A client with type 2 diabetes mellitus is managed with metformin

(Glucophage), an oral hypoglycemic agent. The primary health care

provider prescribes ad additional medication injected exenatide (byetta).

Which information is most important for the nurse to teach this client?

a.

Administer subcutaneously after meals

b.

Consume additional sources of potassium

c.

Notify the healthcare provider if anorexia occurs

d.

Watchfor signsof jitteriness or diaphoresis ( POSSIBLE ANSWER)

48.

A client is who is diagnose with schizophrenia receives a prescription for

an atypical antipsychotic drug aripipazole (Abilify). Which assessment

should the nurse perform to monitor for an adrenergic receptor

antagonist side effect that commonly occurs atypical antipsychotic

agents?

a.

observe the client hallucinatorybehaviors

b.

obtain the client finger stick glucose levels

c.

measure the clients lying and standing blood pressure

d.

determine the clients abnormal involuntary movements scale

(AIMS)

1-

A client with pheocromocytoma reports the onset of a severe headache. The

nurse observes that the client is very diaphoretic. Which assessment data should

the nurse obtain first?

Blood pressure

2-

The drainage in the chest tube of a client with emphysema has changed from

clear watery fluid. What action would be best for the nurse to take/

Maintain the current IV antibiotic schedule

3-

A client is admitted with a sudden onset of right sided the nurse complete

first?

Observe for peripheral edema

4-

When planning care for a client newly diagnose with open angle glaucoma, the

nurse identifies a priority nursing diagnosis of “ Visual sensory/perceptual

alterations”.This diagnosis is based on which etiology?

Decreased peripheral vision

5-

A client in the operating room received succinylcholine. The client is

experiencing muscle rigidity and has an extremely high temperature. What

action should the nurse implement?

Call the PACU nurse to prepare for prolonged ventilatory support

Also know that PACU is BP, Respiration and Pulse

6-

A client who is receiving packed red blood cells develops nausea and

vomiting. What action should the nurse take first?

Stop the infusion of blood

Te lo pueden poner como hemodialysis y tambien es STOP transfusion

7- A client with type 2 diabetes mellitus is admitted to the hospital for

uncontrolled DM. Insulin therapy is initiated with initial dose of Humulininsulin

at 8:00 at 16:00 the client complains of diaphoresis, rapid heart beat, and feeling

shaky. What should the nurse dofirst?

Determine the client current glucose level

8-

After suctioning the patient with an endotracheal tube, which assessment

finding indicates to the nurse that the intervention was effective?

Increase inbreath sounds

9-

The nurse observes an increase number of blood clots in the drainage tubing of

a client with continuous bladder irrigation following a transurethral resection of

the prostate (TURP). What is the best initial nursing action?

Provide additional oral fluid intake

Also with TURP you must know that 3l of water a day is needed

10-

Which nursing diagnosis should be selected for a client who is receiving

thrombolytic infusions for treatment of an acute myocardial infarction?

Risk for injury related to effects of thrombolysis

11-

The nurse is assessing a client who has returned from surgery following a

thoracotomy.Whichfindingindicatestheclientisexperiencingadequategas

exchange?

The client demonstrates effective coughing and deep breathing exercises

12-

When caring for a client with nephrotic syndrome which assessment is most

important for the nurse to obtain?

Daily Weight

13-

A client who had a biliopancreatic diversion procedure (BOP) 3 months ago is

admitted with severe dehydration. Which assessment finding warrants immediate

intervention by the nurse?

Gastroccult positive emesis

14-

A female client with possible acute renal failure (ARF) is admitted to the

hospital and mannitol (Osmitrol) is prescribed as a fluid challenge. Prior to

carrying out this prescription, what intervention should the nurse implement?

No specific nursing action is required

Instruct the client to empty the bladder

Collect a clean catch urine specimen

Obtain vital signs and breathe sounds

15-

The nurse positions a male client for a lumbar puncture by placing him in

the side-lying position with his knees flexed and pulled toward his trunk. What

action should the nurse implement next?

Call another nurse to assist the healthcare provider

Provide a small pillow for the client to curl around

Instruct the client to perform a Valsalva maneuver

Support the client’s head bent forward to the chest

16-

When teaching a client with osteoporosis to increase weight-bearing

exercise, how should the nurse explain the purpose of this activity?

Strengthen leg muscles

Promotevenous return

Increase bone strength

Restore range of motion

17-

A male tells the clinic nurse that he is experiencing burning on urination,

and assessment that he had sexual intercourse four days ago with a woman he

casually met. Which action should the nurse implement?

Observe the perineal area for a chancroid-like lesion

Obtain a specimen of urethral drainage for culture (POSSIBLE ANSWER)

Identify all sexual partners in the last four days

Assess for perineal itching, erythemia, and excoriation

18-

An older female client with long term type 2 diabetes mellitus (DM) is seen in

the doctor routine health assessment.To determine if the client is experiencing

any long-term complications of DM, which assessments should the nurse obtain?

Select all that apply:

Visual acuity

Serum creatinine and blood urea nitrogen (BUN)

Signs of respiratory tract infection

Sensation in feet and legs

Skin condition of lower extremities

19-

Which laboratory test result is most important for the nurse to report to the

surgeon prior to a client’s scheduled abdominal surgery?

Potassium level of 4 mEq/liter

Blood glucose of 90 mg/dl

Serum creatinine of 5 mg/dl (POSSIBLE ANSWER)

Hemoglobin level of 13 grams

20-

A client who has a history of long-standing back pain treated with

methadone (Dolophine), is admitted to the surgical unit following urological

surgery. What modifications in the plan of care should the nurse make for this

client’s pain management during the postoperative period?

Use minimal parenteral opioids for surgical pain, in addition to oral

methadone

Maintain client’s methadone, and medicate surgical pain based on pain

rating

Consult with surgeonabout increasing methadone in lieu of parenteral

opioids

Make no changes in standard pain management for this surgery and

hold methadone

21-

Thenurseappliesanautomaticexternaldefibrillator(AED)toaclientwho

collapsed inan exam room at a community clinic. What action should the nurse

takenext?

Determine the defibrillator reading

Assess the client’s oxygen saturation

Bring a crash cart to the exam room

Measure the client’s blood pressure

22-

Which change in lab values would indicate to the nurse that treatment for

gout is successful?

Decreased serum uric acid

Decreased serum purine

Increased serum uric acid

Increased serum purine

23-

The nurse reports that a client is at risk for a brain attack (stroke) finding?

Jugular vein distention

Palpable cervical lymph node

Carotid bruit

Nuchal rigidity

24-

The nurse is assessing a group of older adults. What factor in a male client’s

history puts him at greatest risk for developing coloncancer?

Is excessively exposed to sunlight

Eats a high-fat diet

Smokes cigars (POSSIBLE ANSWER)

Has intestinal polyps

25-

While taking routine vital signs at 0400 AM, the nurse notes that a client

who had a total knee replacement the previous day has a heart rate of 126

beats/minute. What action should the nurse take first?

Compare heart rate trends with blood pressure trends ( POSSIBLE

ANSWER)

Review the medical record for a history of cardiac disease

Check surgical drainage system and bandage for bleeding

Determine current pain level using a 10-point scale

26-

A client who suffered an electrical injury on the left foot is admitted to the

burn include in this client’s plan of care? (incomplete)

Assess lung sounds q4 hours

Perform passive range of motion

Evaluate level of consciousness

Continuous cardiac monitoring

27-

The nurse is taking a client’s blood pressure sphygmomanometer cuff is

inflated. What (incomplete)

Administer a prescribed PRN antianxiety (POSSIBLE ANSWER)

Assess the client’s recent serum calcium

Notify the healthcare provider of the

Prepare to implement seizure precautions

28-

A client with eczema is using an over-the-counter (OTC) topical product with

urea 10% OTC (Aqua Care Cream) to the affected skin areas. Which finding

reflects the expected therapeutic response?

Decreased weeping of ulcerations in affected area (POSSIBLE ANSWER)

Healing with a return to normal skin appearance

Reduced pain in eczematous areas

Hydration of affected dry skin areas

29-

During an annual health check, the clinic nurse updates an adult female’s

health history. When discussing the woman’s history of lactose intolerance, the

client reports that it has been years since she last consumed dairy products.

What dietary suggestions should the nurse recommend to help ensure that the

client receivesan adequate intake of calcium? Select all that apply:

Increase intake of salmon, sardines, tofu, and leafy green vegetables

Sip a half-cup of mil during a mid-day meal at least every other day

Eat at least six servings of citrus fruits weekly

Include 2 to3 servings of yellow and green squash weekly

Take a calcium supplement with vitamin D daily

30-

A healthcare worker with no known exposure to tuberculosis has received a

Mantoux tuberculosis skin test. The nurse’s assessment of the test after 72 hours

indicates 5mm of erythema without induration. What is the best initial nursing

action?

Review client’s history for possible exposure toTB

Instruct the client to return for a repeat test in 1 week

Refer client to a healthcare provider for isoniazid (INH) therapy

Document negative results in the client’s medical record

31.A

male client in skeletal traction tells the nurse that he is frustrated because

he needs help repositioning himself in bed. Which intervention should the nurse

implement?

Inform the client that it is the nurse’s responsibility to reposition

Provide an overhead trapeze to the bed for the clientto use

Place a draw sheet under the client to assist with repositioning

Administer an intravenous PRN anti-anxiety medication

32-

In planning care for a client with pneumonia, which nursing problem should

the nurse identify as the priority?

Impaired gas exchange related to the effects of alveolar-capillary

membrane changes

Acute pain related to the effects of inflammation of the parietal pleura

Deficient fluid volume related to fever, infection, and increased

metabolic rate

Disturbed sleep pattern related to pain, dyspnea, and hospitalization

33.A

hospitalized client with chemotherapy-induced stomatitis complains of

mouth pain. What is the best initial nursing action?

Encourage frequent mouth care

Administer a topical analgesic per PRN protocol

Cleanse the tongue and mouth with glycerin swabs

Obtain a soft diet for the client

33-

A client returns from surgery following a hiatal hernia repair via Nissen

fundoplication. Which position should the nurse implement for this client?

Right side-lying to promote stomach emptying

Prone to apply external pressure to the suture line

Left side-lying to reduce stress on the suture line

30 degree semi-Fowler’s to drop the diaphragm

34-

An adult woman with Grave’s disease is admitted with severe dehydration is

currently restless and refusing to eat. Which action is most important for the

nurse to implement?

Keep room temperature cool

Determine the client’s food preferences

Maintain a patent intravenous site

Teach the client relaxation techniques

35-

The nurse admits a client who has a medical diagnosis of bacterial

meningitis to the unit. Which intervention has the highest priority in providing

care for this client?

Administer initial dose of broad-spectrum antibiotic

Instruct the client to force fluids hourly

Obtain results of culture and sensitivity of CSF

Assess the client for symptoms of hyponatremia

36-

A client uses triamcinolone (Kenalog), a corticosteroid ointment, to manage

pruritis caused by a chronic skin rash. The client calls the clinic nurse to report

increased erythema with purulent exudate at the site. What action should the

nurse implement?

Schedule an appointment for the client to see the healthcare provider

Advise the client to apply plastic wrap over the ointment to promote

healing

Explain that the client needs to complete all prescribed doses of the

medication

Instruct the client to continue the ointment until all erythema is

relieved

37-

During aparacentesis, two liters offluid are removed from the abdomen ofa

clientwithascites.Adrainagebagisplaced,and50mlofclear,straw-colored

fluid drains within the first hour. What action should the nurse implement?

Palpate for abdominal distention

Clamp drainage tube for 5 minutes

Continue to monitor the fluid output

Send fluid to the lab for analysis

38-

The nurse assesses the dressing of a client who has just returned from post-

anesthesia and finds that the dressing is wet with a moderate amount of bright

red bloody drainage. What action should the nurse take?

Replace dressing with a new sterile dressing, and monitor the wound

hourly until bleeding is stopped

Call surgery and request that the surgeon see the wound prior to

leaving the hospital

Reinforce the dressing and document that a moderate amount of

sanguineous drainage was on the dressing

Document that the dressing was saturated with serious drainage, and

do not change the dressing

39-

While the home health nurse is making a home visit,a client with a history

ofseizuresdemonstratestonic-clonicseizureactivity.Whatactionshouldthe

nurse implement first?

Direct a family member to call emergency services

Ascertain the trigger event

Protect the client’s head with a pillow

Observe the postictal breathing pattern

40-

A client who weighs 176 pounds is admitted to the intensive care unit with a

serum glucose level of 600 mg/dl and a serum acetone level of 50 mg/dl.

Regular insulin at a rate of 0.1unit/kg/hour is prescribed. The pharmacy provides

a solution of Regular insulin 100 units/100 ml of normal saline. The nurse

should set the infusion pump to deliver how many ml/hour? (Enter numeric value

only)

= 8ML/H

41-

A client whose history includes IV drug abuse is admitted to the intensive

care unit (ICU) with Kaposi’s sarcoma associated with Acquired Immune

Deficiency Syndrome (AIDS). Which intervention is most important forthe nurse

to include in the client’s plan of care?

Observe for adverse medication reactions

Assess for signs of AIDS dementia

Identify signs of opportunistic infections

Locate local HIV support groups

42-(Photo) The charge nurse observes a newly employed nurse gathering

equipment to obtain a venous blood sample from a client’s implanted port. The

nurse has obtained the equipment seen in the photo. What actionsshould the

chargenurse take? (Select all that apply)

Guide the nurse in inserting the needle at a 45 degree angle

Remind the nurse to wear sterile gloves for this procedure

Instruct the nurse to obtain several red-topped tubes

Determine if the nurse has ever performed this skill

Assist in obtaining the correct needle to access the port

43-

After a computer tomography(CT) scan with intravenous contrastmedium,

a

client returns to the room complaining of shortness of breath and itching.

Which intervention should the nurse implement?

A.

Send another nurse for an emergency tracheotomy set

B.

Call respiratory therapy to give a breathing treatment

C.

Review the client’s complete list of allergies

D.

Prepare a doseof Epinephrine (Adrenalin

44-

The nurse is reviewing blood pressure readings for a group of client’s on a

medical unit. Which client is at the highest risk for complications related to

hypertension?

A.

Young adult Hispanic female who has a hemoglobin of 11 gm and

drinks beer every day

B.

Middle-aged African-American male who has a serum creatinine level

of 2.9 mg/dL

C.

Older Asian male who eats a diet consisiting of smoked, cured, and

pickled foods.

D.

Post-menopausal Caucasian female who overeats and is 20% above ideal

body weight

1.

Shingles

Teach the pt about phantom pain

2.

Shingles Select all the apply

pain

ability

skin integrity

3.

PATIENT W/ EZCEMA APPLYINGCREAM TTO IS WORKING:

HEALING WITH A RETURN SKIN TO NORMAL APPEARANCE.

4.

PT WITH OBESITY HIGH GLUCOSE LEVEL IS AT RISKFOR?

CARDIOVASCULAR DISEASE

5.

FOR ANEMIA WHAT DOESN’T HAVE IRON, WHICH FOODS ARE NOT RICH IN

IRON?

NO ORANGE

6.

PT. W/ RISK OF DVT

PERFORM ROM EXERCISES ALSO LEGS EXERCISE CAN BE OTHER

WAY TO ANSWER

7.

DISCHARGE FOR VENOUS ULCERS SELECT ALL APPLY?

ELEVATE THE FEET WHEN LAYING DOWN

CHECK BROWNISH SKIN AROUND THE ANKLES

VITAMINS

8.

PT W/ SIADH:

HARD CANDY FOR THIRST.

9.

PT ARRIVE TO PACU POSTOP MOANING WHAT TO DO:

CHECK PULSE, BP AND RESPIRATIONS.

10.

Pt. DIAGNOSED RECENTLY W/ DM HAVE NOT BEEN ABLE TO CONTROL

GLUCOSE LEVEL DURING 3 MONTH WHAT SHOULD BE DONE:

CHECK FOR A1C LEVEL

(OTHER SAY ASSESS FOR WHAT SHE HAVE BEEN EATING 3 DAYS

AGO).

11.

WHEN BP IS HIGH

ADMINISTER (LASIX)

12.

PATIENT W/ ESOPHAEGAL VARICES HAVE NOT BE BLEEDING FOR 3

DAYS:

PROVIDE LUKE WARM BROTH, ICE TEA AND LEMON POPSICLE.

13.

CALCULO:

-0.75

14.

PT WITH OSTEOMALCIA

RISK FOR INJURY

15.

SBAR—EXPLAIN SPECIFIC REASON FOR URGENT NOTIFICATON

TEMPERATURE

16.

INTESTINAL BOWEL OBSTRUCTION

PLACE THE PT 90 DEGREES SITTING

17.

OSTEOARTHRITIS

RISK FORINJURY RELATED TO JOINT PAIN

18.

BONE CANCER TYPE IV:

GIVE OPIODS- NON OPIODS ANALGESICS.

19.

HYPOTHYROIDISM

RESTRICT SODIUM NA 122

20.

PT ARRIVES TO CLINIC W/ NUCHALRIGIDITY FEVER FOR6HOURS

WHAT TO DO:

PREPARE FOR ISOLATION PRECAUTIONS

( I PUT THIS ONE AND NO LUMBAR PUNCTURE)

21.

INTERMITENT CLAUDICATION TEACHING

BANDAGE ELASTIC WRAPED AROUND LEGS

TAMBIEN PUEDE SALIR COMO PAIN TRACTION CAST NOTIFY MD

(CAST NO MORE THEN 4HR)

22.

PREOPERATIVE NURSING CARE

ASSESS EMOTIONAL PREPAREDNESS

ALSO CAN BE CONCERNS AND ANXIETY FOR SURGERY DEPENDE

LA QUE PONGAN

23.

TRACHESTOMY CARE:

LEAVE OLD TIES ON UNTIL NEW ONES BE ON PLACE OR SECURE.

24.

STERNAL TRACTION COMPLAINS OF PAIN

ADMINISTER PRN MEDS

25.

EXTERNAL FIXATION

ADMINISTER PRN MEDS

26.

MULTIPLE SCLEROSIS (MS)

ADMINISTER ANTIMEDICS/ PRN AS PRESCRIBED

27.

FEMALE PATIENT HOW HAVE EPIGASTRIC PAIN FOR 3 DAYS HAVE BEEN

TAKIN ANTACIDS AND NO RESOLVE ARRIVE TO HOSPITAL W/HR;128BPM,

BP110/70 WHAT IS THE MOST IMPORTANT INTERVENTION FINDING IN

ASSESSMENT:

ASSESS FOR RADIATING JAW PAIN.

28.

Pt. W. RADIACTIVE THERAPY WHAT TO TEACH/ RECOMMEND TO

PROTECT THAT PART OF THE SKIN SPECIALLY FROM THE SUN

29.

Pt WITH ALS WHAT TO DO TO PREVENT RESPIRATORY COMPLICATIONS:

TEACH BREATHING TECNIQUES, USES SPIROMETER, AUSCULTATE

FOR BREATH OR LUNG SOUNDS.

30.

PT WITH LEFT LEF ULCER:

KEEP LEG ELEVATED AS MUCH AS HE CAN.

31.

PT WITH AN EXTERNAL DEVICE COMPLAINING OF PAIN:

ASSESS FOR PHERIPHERAL PULSES.

32.

CALCULATION 1G/0.4 G

-= 2.5

33.

EXAMPLES OF DASH DIET:

PEEL FRUITS AND VEGETABLES.

34.

CHEST TUBE W/ A DRAINAGE CHANGING FROM CLEAR TO GREEN:

KEEP IV FLUIDS.

35.

PT W/ OPEN ANGLE GLAUCOMA SELECT ALL THAT APPLY:

FREQUENT EYE EXAM TO ASSES FOR VISSION,

USE DROPS TO DIMINSH IOP,

AVOID EXTRENOUS EXERCICESLIKE JOGGING OR RUNNING

( YO PUSE SOLO ESAS 3 RESPUESTAS).

36.

PT W/ HYPERTHYROIDISM DEVELOPING EXOSPHTALMUS:

PRESCRIBE TEAR EYE DROPS.

37.

PT VOMITING BLOOD LIKE THE PICTURE SAME AS HEMATENSIS:

CHECK VITAL SIGNS ( ASI ESTA EN TODOS LOS PAPELES)

AUSCULTATE LUNGS SOUNDS ( FUE LO QUE PUSO YADIRA)

38.

PATIENT W/ ML FELL AND WHEN RECEIVING THE NURSE HE HAVE 2

PROJECTILEVOMITS WHAT SHE DO:

PROVIDE ANTIEMETICS PRN .

39.

PT W/ RAYNAUD SYNDROME WHICH WORK AS A DATA ENTRY CLERK:

PROVIDE A SPACE TO WARM THE ENVIROMENT NEXT TO HER

( ALGO ASI ERA LA RESPUESTA). Y HAY OTRA RESPUESTA QUE

SOLO DICE KEEP MONITORING

40.

PATIENT THAT HAVE THE K= 6.7 WHAT MEDICATION PROVIDE:

KAYELAXATE (TREATS HYPERKALEMIA).

41.

COLON CANCER PT

KAYELAXATE Med

42.

RENAL INJURY

KAYELAXATE MED

43.

PT WITH A BRONCHOSCOPY AND DRINK A GLASS OF JUICE :

DELAY THE PROCEDURE 6 HOURS

44.

NEW PATIENT DIAGNOSES WITH DM TYPE IS RECEIVING TEACHING IN

WHICH GLUCOMETER WILL BE THE BEST:

ASSESS FOR VISUAL ACUITY AND ABILITY TO READ OR

SOMETHING LIKE THAT.

45.

ABG (PH 7.25 PCO2 50 SODIUM 60

TACHY AND CONFUSION/ RESPIRATORY

46.

ACUTE AGNDIET:

RESTRICT NA INTAKE.

47.

PT W/ A EXPRESSIVE APHASIA IS ANGER WHAT SHOULD DO THE NURSE:

CVA- COMMUNICATE W/ PICTURE BOARDS.

48.

NURSE IS TEACHING THE WIFE IF A PATIENT DIAGNOSED W/ SEIZURE

WHAT TO DO:

TEACH HER HOW TO POSITIONHIM

49.

PT AFTER TTO OF SOMETHING AND WANTS TO EAT:

NURSE ASSESS FOR BOWEL MOVEMENTS.

50.

SLE:

ASSESS FOR HEMATURIA

51.

PATIENT ALLERGIC TO BANANA (LATEX):

CALL TO MD AND OR STAFF TO BE CHANGE EVERYTHING FOR

SINTHETIC MATERIALS,

52.

SUBCUT EMPHYSEMA- TORACOTOMY WAS ASELECTALL THATAPPLY:

ASSESS FOR LUNG SOUNDS,

53.

NECK DISTENTION

THINK IT WAS AND OTHER CHOICE THAT I NOT REMEMBER NOW.

54.

RESTLESS LEG SYNDROME CON FEOSOL:

ASSESSFOR IRON AND FERRITIN.

55.

BNP

ADMINISTRATIVE FUROSEMIDE LASIX IV

56.

PARKINSON PT WALKING

REASURE THAT STEPPING ON CRACKLESIS NOT HARMFUL

57.

ADDISON DISEASE

TAKE CORTICOSTEROID MEDS

58.

CARPO TONIC SYNDROME

WEAR BRACE IN BOTH WRIST

59.

PARKINSON AND ALZAIMERS PT

TATICARDIC AND CONFUSION

60.

MID ABDOMEN BURNING PAIN

PEPTIC ULCER

61.

ANTIBIOTICS

CLEAR DRAINAGE IMPROVE

62.

ALLOPRINOL FOR GOUT

TAKE MEDS ALWAYS

63.

BLOOD TRANSFUSION HIGH TEMPERATURE

BACK PAIN AND HYPOTENSION

( ABO- LOW BACK PAIN AND HYPOTENSION)

64.

CENTRAL FALL RISK

CARDIOVASCULAR DISEASE

65.

RIGHT HIP FRACTURE

O2 SAT LEVEL

66.

DESCRIBE PAIN NEUROPATHY

NERVOUS SYSTEM

67.

ACUTE ABDOMINALPAIN, NASUA, PROJECTIBLE VOMITING

SEVERE HEADECHE AND PHOTO Sensitivity

68.

UROLITHISIS O LITHOTRIPSY PROCEDURE

RESTRICT PHYSICAL ACTION

69.

UAP( DICE EL PACIENTE QUE TIENE ABD PAIN LARGE TARRY STOOL

TEST STOOL FOR OCCULT BLOOD

70.

Insulin for a glucose level of 255 (Pte tmeblando despues que le pusieron

insulin.)

Obtain capillary glucose.

71.

NGT proper tube procedure

Elevate dead 60 to 90 degree….

72.

RA (rheuma)

Impaired peripheral mobility relate to join pain.

73.

Finger stick glucose finding 50

OC Level of conscious

74.

BMI (una persona que pueden tener colon cancer)

Large waist circumference with central fat

Review for Hesi: Recopilation:

1. Community Health/Geriatrics/Professional Issues-Leadership-Geriatric

syndrome-home health

RN needs to go 4 patients and which one needs to see first:

A. The patient discharge yesterday and dehydrated

B. The patient start a new medication and is incontinence

C. The patient that doesn’t want to take a shower

2. Community Health/Medical Surgical-Renal/Reproductive-TURP-home

care

The nurse is reinforcing home care instructions with a client who is being

discharged following transurethral resection of the prostate (TURP). Which

intervention is most important for the nurse to include in the clients

discharge instructions?

A- Avoid strenuous activity for 6

weeks B- Report fresh blood in the

urine

C- Take acetaminophen for fever 101

D- Consume 6 to 8 glasses of water daily

3. Community Health/Pediatrics/ProfessionalIssues-

Leadership/Legal/Ethical-School nurse role

The school nurse is implementing standards to manage students and provide

a safe and healthy school setting. Which action is most important for the

nurse to implement?

A- Maintain student immunization records

B- Develop an emergency plan for the school

C- Ensure that medical supplies are available

D- Conduct annual student health assessments

4. Community Health/Psychiatric/Mental

Health/Fundamentals/Professional Issues/Medical Surgical-

TestBankWorld.org

Anxiety/Communications/Basic Nursing Skills/Safety/Teaching-

Infection-communication

A pt with possible pneumonia come to the hospital and the nurse need to do

an assessment but the family don’t want to leave the room, what the nurse

need to do first?

A –Call the security

B- Put the family out of the room

C- Put a pneumonia droplet sign in the door

D – Continue with the assessment and put mask to the family

5. Critical Care/Fundamentals-Med Administration/Math-IV-mcg/min-

dopamine

DOPAMINE 198 LBS 7mcg/kg/minute, 500 mg and 400 ml. ml/hour?

Answer: 47

198:2.2=90

7x60x90=37800mcg

37800mcg:1000 to mlg=37.8

mlg 500mg:400ml=1.25

37.8:1.25=30.24

6. Critical Care/Fundamentals/Maternity/Pediatrics/Professional Issues-

Basic Nursing Skills/Nutrition/Antepartum/Leadership-Community-

primary prevention

A public health nurse receives funding to initiate a primary prevention

program in the community. Which program best fits the nurse’s proposal?

A. Case management and screening for clients with HIV.

B. Regional relocation center for earthquake victims.

C. Vitamin supplements for high-risk pregnant

women.

D. Lead screening for children in low-income housing.

7. Critical Care/Geriatrics/Medical Surgical-Renal-Acute Tubular

Necrosis -GERI

Diabetic,renal no function,decrease urine or not urine, septic shock,

check urine specific Gravity and osmolarity urine.

Acute Renal Failure: Low Protein

Chronic Renal Failure: NOT Protein at

all

Asw possible:Urine claude and check input and output

8. Critical Care/Medical Surgical-Cardiovascular?

Immune/Hematology/Integumentary/Respiratory-MODS-central line

placement

NOTE: The Multiple Organ Dysfunction Syndrome (MODS) can be defined as

the development of potentially reversible physiologic derangement involving

two or more organ systems not involved in the disorder that resulted in ICU

admission, and arising in the wake of a potentially life-threatening physiologic

insult.

Answer: Shock

26. Fundamentals/Medical Surgical-Basic Nursing Skills-Fluid

volume overload

After receiving IV fluids in the emergency department, an elderly client is admitted

to the acute care unit with a medical diagnosis of dehydration. The client is

receiving 0.9% normal saline at 125ml/hr. via a saline lock and has a bounding

pulse, tachycardia, and pedal edema. When contacting the healthcare provider, the

nurse anticipates a prescription for what intervention?

a. Decrease the rate of the normal saline infusion

b. Increase the rate of the normal saline solution

c. Change the IV solution to 0.45 saline solution

d. Remove the saline lock from the client’s arm

27. Fundamentals/Medical Surgical-Basic Nursing

Skills/Elimination- Acute abdominal pain

Lower abdominal pain (Order):

1. POSITION BENT KNEES

2. Ask for last food that eat

3. DETERMINE BOWEL MOVEMENT

4. INSPECT ABDOMINAL

5. AUSCULTATE 4 QUADRANTS

28. Fundamentals/Medical Surgical-Basic Nursing

Skills/Nutrition- Parkinson’s-meals

Answer: Provide privacy and give extra time to eat meals and snack

OJO

The spouse of a client with Parkinson’s wants to know how to best assist her

husband during feeding as he is having “increasing problems with drooling and

swallowing.” What instruction should the nurse provide to the family member?

A) “Use thickened liquids along with upright positioning during feeding.”

B) “It might be time to switch to enteral feedings if you are afraid that

your husband may choke.”

C) “Increase the amount of fluids he receives to decrease saliva formation

and improve swallowing.”

D) “Use a straw during feedings to facilitate swallowing.”

29. Fundamentals/Medical Surgical-Basic Nursing Skills/Nutrition-Visually

impaired-feeding-UAP

Reloj posisiones manecillas

A patient with chemicals in the eyes and is in the hospital. What the nurse

tells to the UAP to do to help the patient with the food?

A- Give food to the patient in the mouth

B- Indicate to the patient where is the tray ( reorient )

C- Look how the patient eat

D- Finger food

30. Fundamentals/Medical Surgical-Basic Nursing

Skills/Safety- Huntington’s chorea

*ANSWER: padding on the side rail

Or llevarlo a la cafeteria

31. Fundamentals/Medical Surgical-Basic Nursing

Skills/Safety- Hyperglycemia-vomiting

TYPE 1 DIABETES MELLITUS BLOOD GLUCOSE 420 BEGINS

VOMIT:

A- TURN THE CLIENT TO A LATERAL position

B- OBTAIN A FINGER STICK GLUCOSE

32. Fundamentals/Medical Surgical-Basic Nursing Skills/Safety-MRI

A PATIENT SCHEDULED MRI AND SAID THAT HAS A METAL TOOTH.

WHAT THE RN NEED TO DO?

A- ASSESS PT FEAR TO THE

TEST B- CONSULTS RADIOLOGY

C- SEND PT TO X-RAY INSTEAD

D- CANCEL THE TEST.

33. Fundamentals/Medical Surgical-Integumentary/Operative-JP drain full

POSTOPERATIVE DRESSING: ABDOMINAL WOUND WITH

JACKSON PRATT DRAIN. WHAT THE NURSE DO FIRST? (PICTURE)

A- ASSESS THE SURGICAL WOUND

B- SQUEEZE

C- EMPTY

34. Fundamentals/Medical Surgical-Med Administration-IV-gravity

infusion flow rate

(Question with 4 pictures) Overflow:

A- ARM

B- ARM AND

FOREARM C- IV DRIP

D- IV REGULATION

35. Fundamentals/Medical Surgical-Med Administration/Math-IV-

Heparin-units

HEPARIN SODIUM 25000 IN 5% 500 ml

Answer: 36

36. Fundamentals/Medical Surgical-Renal-Diuretic & daily weight

Discharge teaching to a patient with heart failure what parameter is most

important for weight monitoring

*Answer: Weight the patient at the same time, Same Scale, same cloth type)

The nurse is preparing a teaching plan for a client taking a prescribed diuretic for

edema in the lower extremities. What instruction should the nurse include in this

teaching plan?

A- Stop taking the medication when the edema in the lower extremities subsides.

B- Take the diuretic every day, regardless of weight loss or muscle weakness.

C- Limit fluid intake while taking the diuretic to reduce fluid retention.

D- Weight yourself daily at the same time and report excessive weight loss.

37. Fundamentals/Pathophysiology-Basic Nursing Skills/Hygiene/Safety-

Handwashing

HAND WASHING:

A- Reduces spread of microorganism. Bio…..

B- Lock virus

C- Lock in human virus

38. Fundamentals/Pathophysiology/Professional Issues/Medical Surgical-

Basic Nursing Skills/Nutrition/Teaching-DM2 and CKD-diet

Ketoacidosis Diet

A- Banana, whole bread…

B- Oatmeal……

C- 6 oz Coffee, strawberry, artificial

sweetening D-Egg, butter

39. Fundamentals/Pediatrics-Basic Nursing Skills/Nutrition-infant weight-

1-month

AT THE 1 MONTH OLD CLINIC VISIT, AN INFANTS NUDE WEIGHT IS 600

GRAM MORE THATAT BIRTH. WHICH INTERVENTION SHOULD THE

NURSE IMPLEMENT?

A. ENCOURAGE GIVING 2 OUNCES OF WATER BETWEEN FEEDINGS.

B. RECOMMENDED ADING KARO SYRUP TO EACH FORMA FEEDING

C. DOCUMENT INFANT’S WEIGHT ON GROWTH CHART

D. CHECK THE INFANT’S WEIGHT USING A METRIC SCALE.

NOTE: ANSWER: 600 grams

40. Fundamentals/Pediatrics-Med Administration-Oral susp-resisting-

PEDI

A child that resists taking the medication:

a. Parents help the nurse holding him

b. Provide the child juice with the medication

c. Explain to the child that if he doesn’t take the medication, he won’t

feel better.

41. Fundamentals/Pediatrics-Med Administration/Math-Calculation-PO

dose-3x/wk/BSA

The healthcare provider prescribes methotrexate 7.5 mg PO weekly, in 3

divided doses for a child with rheumatoid arthritis whose body surface area

(BSA) is 0.6 m2. The therapeutic dosage of methotrexate PO is 5 to 15

mg/m2/week. How many mg should the nurse administer in each of the

three doses given week?

Answer: 2.5

42. Fundamentals/Pediatrics-Med Administration/Math-IV-ml/hour-PEDI

Vanco

400 mg 6 hours, 100 ml one and half

hour Answer: 67

43. Fundamentals/Pediatrics/Professional Issues/Medical Surgical-Basic

Nursing Skills/Safety/Leadership-Airborne precautions

Un Nino que los Padres lo llevaron al ER

A. Mandarlo a la casa

B. RN ponerse el precaution

C. Ponerle una mascara al nino.

B

*(Isolated room)

* Airborne precautions:

1. Diseases

a. Measles

b. Chickenpox (varicella)

c. Disseminated varicella zoster

d. Tuberculosis

2. Barrier protection

a. Single room is maintained under negative pressure; door remains

closed except upon entering and exiting.

b. Negative airflow pressure is used in the room, with a minimum of 6 to 12

air exchanges p hour depending on health care agency protocol.

c. Ultraviolet germicide irradiation or high-efficiency particulate air filter is

used in the room

d. Health care workers wear mask or personal respiratory protection device.

e. Mask placed on client when client is out of the room; client leaves the

room only if necessary.

44. Fundamentals/Professional Issues-Basic Nursing

Skills/Nutrition/Cultural/Spiritual-Hindu diet

A Hindu patient… what can the nurse do?

A- REMOVE BEEF FROM PT MEAL TRAIL

B- ENCOURAGE FAMILY TO BRING FOOD FROM HOME

C- SHOW THE CARDIAC MENU TO THE PATIENT

D- GIVE TO THE PATIENT WHAT HE WANTS

45. Fundamentals/Professional Issues-Med Administration/Documentation-

Bar code scan-med administration

When administering a new medication to a client, the nurse uses a scanner to

register the nurse?

A) Use the scanner to register the bar code on the client’s identification bracelet.

B) Document the medication administration on the client’s

computerized record.

C) Remove the medication from the unit dose packaging while verifying

the dose.

D) Reconcile the medication to be administered with the initial

client prescription.

46. Fundamentals/Professional Issues/Medical Surgical-Basic Nursing

Skills/Nutrition/Teaching- Hypertension diet

A PATIENTWITH HIGH BP, THE NURSE GIVE ATEACHING FOR

WHAT CAN HE EAT FOR LUNCH?

A- TOMATO JUICE AND GLUTEN FREE CRACKERS

B- BAKED SWEET POTATO

47. Fundamentals/Professional Issues/Medical Surgical-Basic Nursing

Skills/Safety/Teaching-Influenza precautions

Patient with influenza. Dehydrated and pneumonia:

A. Droplet precaution

B. Family member wear mask

NOTE: Droplet precautions should be implemented for patients with

suspected or confirmed influenza for 7 days after illness onset or until 24

hours after the resolution of fever and respiratory symptoms, whichever is

longer, while a patient is in a healthcare facility.

48. Fundamentals/Professional Issues/Medical Surgical-Med

Administration/Teaching-Insulin adm-teaching 1

(PICTURE)

The nurse shows the mom of the child how to use insulin for the child that is

diabetic:

A- ASSIST THE MOTHER IN

B- THE CORRECT ANGLE

C- LOCATING THE CORRECT SITE

Or assess

45 Angle

Pen 90 angle

49. Fundamentals/Professional Issues/Medical Surgical-Teaching-Pursed

lip breathing 2

VIDEO *Pursed lip Breathing: IN and OUT

(Inhale through the nose and exhale by mouth)

50. Geriatrics/Medical Surgical-Integumentary-Skin care-GERI

An older male resident of a long-term care facility has been scratching his legs

for the past 2 days. Which intervention should the nurse implement?

A) Explain the importance of bathing or showering daily.

B) Keep the legs covered as much as possible.

C) Apply emollient to affect area at least twice daily.

D) Encourage fluid intake of at least 2,000 ml daily.

51. Maternity–Antepartum –Fetal stress – Tachycardia

The nurse is assessing a primigravida at 39-weeks gestation during a weekly

prenatal visit. Which finding is most important for the nurse to report to the

healthcare provider?

A) Reports intermittent low back pain.

B) Fetal heart rate of 200 beats/minutes

C) Complains of early morning heartburn

D) Maternal hemoglobin of 11.0 g/ dl or 110 g/l (SI)

*Note: Normal FHR pregnant women: 120-160

52. Maternity – Intrapartum – Intrapartum pain management

PREGNANT WOMEN WITH 8 CM DE DILATATION Y 100%, SHE

WANTS TO GET HYDROCHLORIDE (DON’T REMEMBER THE

EXACTLY NAME) FOR PAIN:

A- ADMINISTER EPIDURAL

B- ADMINISTER HYDROCHLORIDE

C- RELAXATION TECHNIQUE

53. Maternity – Postpartum – Hemorrhage

postpartum Possible asw: Check for clots and lochia

54. Maternity – Postpartum – Priority management-postpartum

After receiving shift report, the nurse working on a postpartum unit should

assessment first?

A) Vaginal birth today whose infant is refusing to breastfeed.

B) Cesarean birth of twin today who is new complaining of pain.

C) Post-cesarean birth today with fundus at the umbilicus.

D- Multipara vaginal birth yesterday saturating two pads hours.

55. Maternity/Medical Surgical – Antepartum – Barbiturates & pregnancy

The nurse is evaluating medication teaching. Which statement by a female who

takes a barbiturate for sleep indicates she understands the teaching?

a) “I should ensure that I do not become pregnant while taking this medication.”

b) “I must take my birth control pill in the morning and my sleeping pill

at night.”

c) “I will increase the amount I take in small doses if I can’t sleep through

the night.”

d) “I should take my anxiety pill, alprazolam, only when I really need it.”

56. Maternity/Medical Surgical –Postpartum –Post vaginal

delivery- diaphragm

Patient that had a vaginal birth, diaphragm. What teaching the nurse need to give to

the patient?

A- 2 or 6 hours before intercourse

B- Re-adapt

C- Resisted diaphragm

D- Is no anticoncertive

57. Maternity/Professional Issues-Antepartum/Cultural/Spiritual-Pregnancy-

cultural awareness

Pregnant women first prenatal visit at 12 weeks

A – Concern about delivery

B – Parenting

C – Complication during pregnancy

D – CHILDHOOD

58. Maternity/Professional Issues-Antepartum/Leadership-BPP-fetal well-

being

Four clients arrive on the labor and delivery unit at the same time. Which client

should the nurse assess first?

a) A 41-week multigravida who is scheduled induction of labor today.

b) A 38-week primagravida who reports contractions occurring every 10 minutes.

c) A 36-week multigravida with a prescription for serial blood pressure.

d) A 39-week primigravida with biophysical profile score of 5 out of 8

59. Medical Surgical-Cardiovascular-Angina-exercise

A male client with angina pectoris is being discharged from the hospital. What

instructions should the nurse plan to include to the discharge teaching?

a. Engage in physical exercise immediately after eating to help

decrease cholesterol levels.

b. Walk briskly in cold weather to increase cardiac output.

c. Keep nitroglycerin in a light-colored plastic bottle and readily available.

d. Avoid all isometric exercises, but walk regularly.

60. Medical Surgical-Cardiovascular-Arterial sheath

Saunder 791

Arterial sheath : Pedal pulses and colour, warmth movement and sensation of

affected leg & footAsses insertion site for bleeding, pain, tenderness, swelling

or haematoma. No levantarse hasta despues de 8 hrs

A patient recovering left femoral atrial sheath. What finding requires immediate

intervention (Select all that apply?)

A. Tenderness on insertion site

B. Left groin egg size

C. Quarter size of drainage

D. Unrelieved back, flank pain

E. Cool/pale left foot

The nurse in the outpatient unit is caring for a client who had a right femoral

cardiac cauterization two hours ago .What assessment findings requires immediate

intervention?

A. The client wants assistance walking to the bathroom

B. Clients pulse oximeter is 98%

C. The client right feed is warn to touch

D. The client B/P is 110/70 and pulse 90

OJO CON ESTA NO SALIO PERO HAY QUE VERLA

61. Medical Surgical-Cardiovascular-Atenolol

The healthcare provider prescribes atenolol 50 mg PO daily for a client with angina

pectoris. Which finding should the nurse report to the healthcare provider before

administering the medication?

a) Chest pain.

b) Urinary frequency.

c) Tachycardia.

d) Irregular

pulse.

62. Medical Surgical-Cardiovascular-Atrial fibrillation-assess

Atrial fibrillation, or A-Fib, is the most common heart rhythm disorder in the

United States. It’s a condition in which the electrical impulses that control

muscle contractions in the upper chambers of the heart become rapid and

chaotic. About 160,000 new cases of A-Fib are diagnosed in the U.S. each

year–but physicians believe that many people who have A-Fib have not been

diagnosed.

The likelihood of developing A-Fib increases with age. The majority of people

diagnosed with A-Fib are 55 or older. Between three and five percent of

people over age 65 and nine percent of people over the age of 80 have A-Fib.

Diagnosing and treating A-Fib are important because, left untreated, it can

lead to a number of serious heart conditions. Patients with A-Fib are also five

times more likely to suffer a stroke. (Although you should see a doctor to

diagnose A-Fib, one way to help asses your risk is to take your pulse. Click

here for a step-by-step guide–or watch Archie Manning show how it’s done.)

One complicating factor is that the signs and symptoms of A-Fib can vary

greatly from patient to patient. Some people experience a sudden heart flutter

or tremor, or feel their heart “speed up” suddenly; other patients with A-Fib

may not feel anything at all. Other symptoms can include:

Shortness of

breath Fatigue

Weakness or difficulty

exercising Chest pain

Sweating

Dizziness

Fainting

A-Fib is not an emergency–but it is a serious condition. If you suspect you

have A-Fib you should see your doctor immediately. Contact your primary

care doctor–or find a St.Vincent doctor near you. Or make an appointment to

see an A-Fib specialist at the St.Vincent A-Fib Center of Excellence. We can

discuss the many treatment options available to treat and cure A-Fib–and

help choose the one that’s right for you.

63. Medical Surgical-Cardiovascular-BP-variance in arms

*Change the arm or wait 5 min and change the arm

64. Medical Surgical-Cardiovascular-High BP-vasoconstriction

A patient is diagnosed with MALIGNANT HYPERTENSION, patient likes

skiing and asks if is ok to continue:

A. “COLD WEATHER MAY CONSTRICT YOUR BLOOD VESSELS

AND INCREASE BP”

B. “SKIING MIGHT PRODUCE TOO MUCH EXERTION”

C. “SHOULD BE OK AS SOON AS YOU CONFINE SKIING

D. “GO FOR IT IS A TERRIFIC WORKOUT

65. Medical Surgical-Cardiovascular-Pitting edema

4+

66. Medical Surgical-Cardiovascular-SVT-cardioversion-priority

Electro shock

Posible asw: synchronic

67. Medical Surgical-Cardiovascular/GI/Hepatic-Bariatric surgery-abd pain

A patient get to ER and had a week before a bariatric surgery, patient is shortness

of breath and has abdominal pain.

A. Blood pressure 88/50

B. Left shoulder pain

C. Sustained sinus tachycardia

D. 101 temperature

A woman who had bariatric surgery 2 months ago is admitted because of vomiting

and inability to tolerate food and liquids. She states that she is pain free. Which

intervention should the nurse include in the client’s plan of care?

a. Maintain the client on a NPO status

b. Administer daily vitamin supplements

c. Determine if the client is over-hydrating to feel satiated

d. Encourage positive self-accolades for dietary adherence

OJO: ESTA NO SALIO EN ESTE PERO NOS PUEDE SALIR.

68. Medical Surgical-Cardiovascular/Physical Assessment-Carotid bruit

Femoral carotid

69. Medical Surgical –Cardiovascular/Renal-Lasix-outcome PEDI

Child taking Lasix. Nurse look for effective of the medicine:

Answer: Lose 2 pounds weekly

70. Medical Surgical-Cardiovascular/Trauma/Emergency-Unstable angina

Unstable angina is more intense that stable angina. Make a pt awake when

they sleep. Made more that 5 min no relies by nitroglycerin. Is a stereo

sclerotic plaque rupture. Thrombus formation MI. No always we can see in a

elevated ST changes.

Stable angina is realize with nitroglycerin and relaxation .

OJO: ESTA NO SALIO PERO HAY QUE VERLA

Client was admitted to the cardiac observation unit 2 hour ago complaining of

chest pain .On admission the client EKG showed bradycardia ,ST depression ,but

no ventricular ectopic .The client reports a sharp pain ,telling the nurse ,I feel like

an elephant just stepped on my chest .The EKG now shows Q waves and ST

elevations in the anterior leads .What intervention should the nurse perform ?

A. Administer prescribed morphine sulfate IV and provide oxygen at 2L

per minute per nasal cannula

B. Obtain a stat 12 lead EKG and perform a venipuncture to check cardiac

enzyme levels

C. Notify the HCP of the clients increased chest pain and call for

defibrillator crash cart

D. Increased the peripheral IV rate to 175 ml/hr. to prevent hypotension and

shock

71. Medical Surgical-Endocrine-Chvostek’s sign-tetany-POC

The nurse is caring for a client with hyperparathyroidism. Which assessment

should the nurse include the plan of care?

A) Chvostek’s sign

B) Brudzinski’s sign.

C) Battle’s sign.

D) Pupillary response.

72. Medical Surgical-Endocrine-Diabetes-acute confusion

A female client with pancreatic cancer is NPO for implantation of a venous

sedation. Suddenly, the client becomes unresponsive, and her skin is cool pulse 96

beats/minute, respiratory rate 18 breaths/minute, which are within her outpatient

surgery nurse implement first?

A- Administer glucagon 0.5 mg IM

B- Infuse a 200 ml NS IV fluid bolus

C- Obtain a finger stick blood glucose

D- Insert a second peripheral IV catheter

73. Medical Surgical-Endocrine-DKA-IV

IV insulin: Trade hydration with rapid IV infusion 0.9 a 0.45 normal saline as

prescriber, because can elevated edema.

Intravenous fluid replacement should start immediately with 1 to 2 L of

normal saline over the first 1 to 2 hours of treatment for adults. In children

the initial fluid bolus is weight based (5 to 20 mL/kg, dependent on the child’s

perfusion status); volume replacement is carefully titrated because of the

high risk for cerebral edema in the pediatric population. The adult patient

may require up to 8 to 10 L of fluid. Volume is gradually replaced after the

initial

fluid bolus because rapid infusion of a large volume increases the risk for

development of cerebral edema. Close observation of intake and output is

essential; placement of a urinary catheter ensures accurate output assessment.

A continuous infusion of regular insulin is administered at 0.1 units/kg/hr to

stop ketogenesis and achieve a steady decrease in serum glucose level of 50 to

75 mg/dL/hr; an initial intravenous (IV) bolus of 0.15 units/kg of regular

insulin may be administered. The short duration of action for regular insulin

allows better control of serum glucose levels. After serum glucose level reaches

250 to 300 mg/dL, fluids should be converted to 5% dextrose in normal saline

(D5NS) to provide fuel until the patient is able to eat. Resolution of a

hyperglycemic emergency occurs when the serum glucose level is less than 200

mg/dL, serum bicarbonate level is greater than or equal to 18 mEq/L, and in

DKA, the venous pH is greater than 7.3.[3]

Fluid replacement dilutes serum potassium and promotes diuresis. In

addition, total body hypokalemia is exacerbated by metabolic acidosis, so

potassium replacement should begin after the initial liter of IV fluids is

infused, even when initial values are normal. Potassium levels frequently drop

precipitously in the first few hours after treatment has been initiated because

potassium moves back to the intracellular space along with the insulin and

existing glucose. Serum potassium levels must be repeated every 1 to 2 hours

during initial management. Cardiac monitoring is essential because

dysrhythmias can develop with significant hypokalemia.

Acidosis generally corrects with insulin therapy. Insulin allows the cells to

use available glucose for energy, leading to decreased proteinolysis and

lipolysis, and the ketoacidosis resolves. Insulin infusion should be continued

until the pH or serum bicarbonate level has normalized; IV fluids should be

converted to D5NS once the serum glucose level reaches 250 to 300 mg/dL to

prevent hypoglycemia. Acidosis in DKA is not routinely treated with sodium

bicarbonate because sodium bicarbonate administration can cause rebound

alkalosis, which can worsen hypokalemia and increases the risk for

development of cerebral edema.

Controlling nausea and vomiting not only improves patient comfort but

prevents worsening dehydration. The patient may require analgesia to relieve

abdominal pain, headaches, or other somatic complaints. Providing a quiet,

calm environment can improve patient comfort. Stress reduction plays an

important part in patient recovery. Thorough explanation of treatment,

medications, and plan of care can alleviate stress related to hospitalization.

Potential complications include hypoglycemia, hypokalemia, dysrhythmias,

and cerebral edema. Monitor capillary/serum glucose levels,

electrocardiogram (ECG), laboratory values, vital signs, intake and output,

and neurologic status carefully. If the serum glucose level falls rapidly, the

resulting fluid shift can lead to cerebral edema, which is associated with a

higher mortality rate. Cerebral edema remains the leading cause of death for

children presenting in DKA.

74. Medical Surgical-Endocrine/GI/Hepatic-Acute pancreatitis

Pancreatitis

The pancreas is an organ located behind the stomach that produces chemicals

called enzymes, which are needed to digest food. It also produces the hormones

insulin and glucagon. Most of the time, the enzymes are only active after they

reach the small intestine .

When these enzymes become active inside the pancreas, they digest the tissue

of the pancreas. This causes swelling, bleeding (hemorrhage), and damage to

the organ and its blood vessels . This condition is called acute pancreatitis .

Acute pancreatitis affects men more often than women. Certain diseases,

surgeries, and habits make you more likely to develop this condition. The two

most common causes of pancreatitis in the United States are heavy alcohol

use and gallstones.

Alcohol use is responsible for up to 70% of cases in the United States. Acute

pancreatitis typically requires 5 to 8 drinks per day for 5 or more years.

Gallstones are the next most common cause. The condition develops when

the gallstones travel out of thegallbladder i nto the bile ducts, where they block

the opening that drains the common bile duct and pancreatic duct (ampulla).

Genetics may be a factor in some cases. Sometimes, the cause is not

known. Other conditions that have been linked to pancreatitis are:

• Autoimmune problems (when the immune system attacks the body)

•Damage to the ducts or pancreas during surgery

•High blood levels of a fat called triglycerides (hypertriglyceridemia) usually above

1000 mg/dL

•Injury to the pancreas from an

accident Other causes include:

•Complications of cystic fibrosis

•Hemolytic uremic syndrome

•Hyperparathyroidism

•Kawasaki disease

•Reye syndrome

•Use of certain medications (especially estrogens, corticosteroids,

sulfonamides, thiazides and azathioprine)

•Viral infections, including mumps, coxsackie B, mycoplasma pneumonia ,

and campylobacter

•Injury to the pancreas after a procedure such as an ERCP (endoscopic

retrograde cholangiopancreatography ) or EUS (endoscopic ultrasound) with

FNA (fine needle aspirate)

Symptoms

The main symptom of pancreatitis is pain felt in the upper left side or middle

of the abdomen. The abdominal pain:

•May be worse within minutes after eating or drinking at first, especially if

foods have a high fat content

• Becomes constant and more severe, lasting for several days

• May be worse when lying flat on the back

•May spread (radiate) to the back or below the left shoulder blade

People with acute pancreatitis often look ill and have a fever, nausea, vomiting,

and sweating.

Other symptoms that may occur with this disease include:

• Clay-colored stools

•Gaseous abdominal fullness

•Hiccups

•Indigestion

•Mild yellowing of the skin and whites of the eyes (jaundice)

Swollen abdomen

75. Medical Surgical-Endocrine/Integumentary-DM cellulitis 3 eval

Cellulitis is a deep infection of the skin that extends to the subcutis. It begins

as a painful, tender, erythematous, warm area that spreads rapidly and

produces indistinct borders. Fever, chills, rigors, and sweats are frequent. The

infection most often begins at the site of antecedent trauma, which may be

minor or major. It may also occur as a result of infection associated with

closure of non-sterile wounds and at the site of sutures. Cellulitis frequently

extends via the lymphatic system and can produce lymphangitis,

lymphadenopathy, abscesses, and bacteremia.

While taking antibiotics, monitor your condition to see if symptoms

improve. In most cases, symptoms will improve or disappear

within a few days. In some cases, pain relievers are prescribed.

You should rest until your symptoms improve. While you rest,

you should raise the affected limb higher than your heart to

reduce any swelling.

76. Medical Surgical-GI/Hepatic-GERD-antacid

An antacid is prescribed for a client with gastro esophageal reflux (GERD). The

client asks to the nurse, “How does this help my GERD? What is the best response

by the nurse?

A.“Antacids decrease the production of gastric secretions.”

B. “It will improve the emptying of food through your stomach.”

C. “This medication will coat the lining of your

esophagus.” D.“Antacids will neutralize the acid in your

stomach.”

77. Medical Surgical-GI/Hepatic/Neurological-Lactulose

Asw: Lactulose

78. Medical Surgical-GI/Hepatic/Respiratory-EGD-recovery care

Following an esophagogastroduodenoscopy (EGD), a make client is drowsy and

difficult to and his respiratory are slow and shallow. Which action should the

nurse implement (Select all that apply?)

A) Initiate bag valve-mask ventilation.

B) Prepare medication reversal agent.

C) Apply oxygen via nasal cannula.

D) Check oxygen saturation level.

E) Begin cardiopulmonary resuscitation.

79. Medical Surgical-Immune/Hematology-Blood transfuse reaction

A client receiving a blood transfusion complains of itchy skin and appears flushed.

What action should the nurse take first?

A- Check the blood type on the bag

B- Notify the healthcare provider

C- Assess the client’s temperature

D- Stop the blood transfusion.

80. Medical Surgical-Immune/Hematology/Integumentary-Sunburn-severe

reaction

Pt expuesto al sol y no se puso sunblock and blisters. Que s/s vas a ver.

Posible asw: headache

No es chills and fever

Signs of Sunburn. When you get a sunburn, your skin turns red and hurts. If

the burn is severe, you can develop swelling and sunburn blisters. You may

even feel like you have the flu — feverish, with chills, nausea, headache, and

weakness.

81. Medical Surgical-

Immune/Hematology/Integumentary/Trauma/Emergency-Dog bite-adult

An adult arrives at the urgent care clinic after being bitten on the hand by an

aggressive dog that escaped from a neighbor’s fenced yard. The nurse cleanses the

wound with providone-iodine and administers Human Rabies Immune Globulin

(HRIG) and the first injection of the rabies vaccine. Which intervention is most

important for the nurse to implement?

A- Determine if the client has any allergies to antibiotics.

B- Send client for a magnetic resonance image (MRI) of the hand

C- Schedule administration of remaining rabies vaccine injections

D- Notify local Animal Control Bureau about the dog bite

82. Medical Surgical-Immune/Hematology/Musculoskeletal-Myasthenia

gravis-findings

MYASTHENIA GRAVIS TAKING (MESTINON). WHAT FINDING REQUIRES

INTERVENTION BY THE NURSE?

A. EYELID DROOPING

B.TINGLING EXTREMITIES

C.UNCONTROLLED DROOLING

83. Medical Surgical-Integumentary-Rule of nines-estimate

Patient with burns. What percent?

a. 36%

b. 27%

c. 50%

d.16%

84. Medical Surgical-Integumentary/Reproductive-Burns-monitor

A client with superficial burns to the face, neck, and hands resulting from a

house firesisadmittedtotheburnunit.Whichassessmentfindingindicatesto

the nurse that the client should he monitored for carbon monoxide poisoning?

A- Expiratory stridor and nasal flaring

B- Mucous membranes cherry red

color C- Carbonaceous particles in

sputum

D- Pulse oximetry reading of 80 percent.

85. Medical Surgical –Musculoskeletal-Fat embolism-S&S hesi pg 134

Fat embolism: A process by which fat tissue passes into the bloodstream and

lodges within a blood vessel.

Signs and symptoms: include central nervous system dysfunction that may

progress to coma or death, irregularities in the heartbeat, respiratory distress, and

fever. Anemi a and thrombocytopeni a (low platelet count) are common. Commonly,

small hemorrhages are seen on the neck, shoulders, armpits, and conjunctiva.

Posible asw: lowfever

86. Medical Surgical-Musculoskeletal-Gout-stress management

Which expected outcome statement should the nurse include in a teaching plan of

care. A client with management of an acute attack of gout?

A.The client will avoid use of alcohol in managing stress

B.The client will implement a high purine daily dietary regimen

C. The client will use local heat application for acute pain

D.The client will stop antigout medication once pain subsides

87. Medical Surgical-Musculoskeletal-RA-pain management

Patient with rheumatoid arthritis joint pain and swelling, taking prednisone and

ibuprofen, self management pain what information obtain

A. Presence of bruising, weakness

B. Amount of protein

C.Therapeutic exercise daily

D.Existence GI discomfort

88. Medical Surgical-Musculoskeletal-Risendronate teaching

Which instruction is most important for the nurse to provide a client who receives

a prescription for risendronate sodium to treat osteoporosis?

a. Remain upright after taking the medication.

b. Begin a weight-bearing exercise plan.

c. Increase intake of foods rich in calcium.

d. Schedule a bone test every year.

89. Medical Surgical-Neurological-Alzheimer’s-safe

In planning care for a client with early stage Alzheimer’s disease, the nurse

establishes the nursing diagnosis of, “Risk for injury related to impaired

judgment.” Which intervention is most important for the nurse to include in this

client’s plan of care?

A.Offer the client frequent reassurance that he/she will be safe.

B. Assign a UAP to provide the client with total personal care.

C. Engage the client in regularly scheduled activities during the day.

D.Arrange the client’s environment so the client can move about freely.

If the client is aggressive you can use the wrist with doctor’s authorization

90. Medical Surgical-Neurological- Cervical cord injury

Asw: Sharp pain

Symptoms of a spinal cord injury may include:

Head that is in an unusual position

Numbness or tingling that spreads down an arm or leg

Weakness

Difficulty walking

Paralysis (loss of movement) of arms or legs

Loss of bladder or bowel control

Shock (pale, clammy skin; bluish lips and fingernails; acting dazed or

semiconscious)

Lack of alertness (unconsciousness)

Stiff neck, headache, or neck pain

91. Medical Surgical-Neurological-Increased ICP-papilledema

Patient involves in an accident… which indicate increase of ICP:

A. Nuchal rigidity/dystonia

B.Confusion/papilledema

C.Periorbital eccymocis

D.Increase Glasgow scale

92. Medical Surgical-Neurological/Physical Assessment-Pupil constriction

Asw:Pupila accommodation

93. Medical Surgical –Neurological /Sensory-Pregabalin

Peripheral neuropathy Pregabalin 4 days, what indicate med is effective?

A. GRANULATING TISSUE IN FOOT ULCER

B. IMPROVED VISUAL ACUITY

C. FULL VOLUME OF PEDAL

PULSES D. REDUCE LEVEL OF PAIN

94. Medical Surgical –Oncology/Physical Assessment –Lymphatic cancer

The nurse is palpating the lymph nodes of a 10 month old. Which findings should

the nurse call to the attention of the health care provider?

a. Enlarged, warm, tender preauricular node

b. Enlarged no tender mobile occipital node

c. Small discrete, mobile, no tender, inguinal node

d. Small, firm, mobile nodules in the axial

Otra pregunta: pt q le encontraron unos immobility node enlargerged,tender.

Asw: maligna

95. Medical Surgical-Oncology/Reproductive-Breast cancer findings

A female client with breast cancer who completed her first chemotherapy

treatment out-patient cancer treatment center is preparing for discharge. Which

behavior the client understands her care needs for the next week?

a. Invite friends and family to visit while she is at home for the next week

b. Rent movies and borrow books to use while passing time at home

c. Schedule a lunch date with her best friend for 2 days from now

d. Stock her refrigerator with healthy foods including fruits and vegetables

OJO: SI APARECE UNA QUE HABLA DE

DIMPLING of the skin ESA ES LA RESPUESTA

96. Medical Surgical-Operative-OR-supine position

A patient surgery for more than 2 hours, what implementation or intervention?

Answer: Put padding to the bony prominences

(ESTO PUSO ANNERY Y LE SALIO BIEN)

No es tie the extremity

No es blanket and

pillow

97. Medical Surgical-Operative-Postop-ambulation

At 1615, prior to ambulating a postoperative client for the first time, the nurse

reviews the client’s medical record. Based on data contained in the record, what

action should the nurse take before assisting the client with ambulation (click on

each chart tab for additional information, be sure to scroll to the bottom right

corner of each tab to view all information contained in the client’s medical record)

A) Remove sequential compression devices

B) Apply PRN oxygen per nasal cannula

C) Administer a PRN dose of an antipyretic

D) Reinforce the surgical wound dressing

98. Medical Surgical-Physical Assessment-Pain assessment

Assessing client’s pain:

A- “Tell me more how you respond

(I DON’T HAVE ANSWER FOR THIS ONE, IF SOMEBODY GOT

CORRECT LET US KNOW)

99. Medical Surgical-Physical Assessment/Respiratory-Pneumonia-priority

assessment

Patient with pneumonia ABG ph 7.24, CO2 65, CO3 24. Which intervention…plan

of care daily

A. Hypertension

B. Maintain IV

C. Check electrocardiogram daily

D. Assess lung for increase pulmonary secretion

*The nurse is preparing to administer an oral antibiotic to a client with unilateral

weakness, mouth drooping, and aspiration pneumonia. What is the priority nursing

assessment that she will be done before administering the medication.

A- Determine which side of the body is

weak B- Auscultate and breathe sounds

C- Obtain and record client vital sign

D- Ask the client about soft food preferences

100. Medical Surgical-Renal-Acute kidney injury (AKI)-POC

A female client with chronic pyelonephritis expresses concern that she may have to

undergo dialysis. What is the best initial response by the nurse?

a. Offer to introduce the client to a dialysis nurse who can provide

teaching about dialysis

b. Explain the relationship between chronic kidney infection renal failure

and dialysis

c. Provide assurance that dialysis is not the usual treatment for

kidney infections

d. Assist the client to reduce anxiety and gain control by using guided

imagery exercise

101. Medical Surgical-Renal-Chronic renal insufficiency

Which symptoms is a characteristic of urethral colic in the client diagnosed with

renal calculi?

a. symptoms of irritation associated with urinary tract infection

b. Acute, excruciating pain, wave-like pain radiating to the gemnitalia

c. intense, deep ache in the cost vertebral region

d. chills, fever and dysuria

102. Medical Surgical-Reproductive-Endometriosis

A young adult female client with recurred pelvic pain for 3 years returns to the

clinic for relief of severe dysmenorrheal. The nurse reviews her medical

record

which indicates that the client has endometriosis. Based on this finding, what

information should the nurse provide this client?

A- Oral contraceptives increase the symptoms of endometriosis

B- An option to diagnose disease extent and provide therapeutic treatment is

laparoscopy

C- Infertility is successfully treated with removal of intra-abdominal endometrial

lesions

D- The symptoms of endometriosis can increase with menopause

103. Medical Surgical-Respiratory-Chest tube-respiratory distress

The UAP find a patient (chest tube) with shortness of breath call the RN. What is

the first thing that the nurse implements?

A- 2 L Oxygen

B- Check the tube connection

104. Medical Surgical-Respiratory-Pulmonary function test

A client with a 40 pack year history of smoking does not want to have a pulmonary

function test conducted. Which of the following should the nurse explain to the

client regarding this diagnostic test?

A- ¨It is used to diagnose lung cancer so treatment can be started

B- ¨It is used to determine the amount of oxygen that is in your lungs with every

breath¨.

C- ¨It measures your lung functioning¨.

D- ¨It identifies the best interventions to help you quit smoking.

105. Medical Surgical-Respiratory-TB precautions

ANSWER: AIRBORNE PRECAUTION

106. Medical Surgical-Respiratory-TB activated

A male client recently release from a correctional facility arrives at the clinic with

a cough, fever, and chills, active tuberculosis (TB) 10 years ago. What action

should the nurse implement? (Select all that apply)

A- Administer a purified protein derivate (PPD)

test B- Schedule the client for a chest radiograph

C- Obtain sputum for acid fast bacillus (AFB) testing

D- Place a mask on the client until he is moved to isolation

E- Send client home with instructions for a prescribed antibiotic

107. Medical Surgical-Sensory-Cataract extraction-nausea

ANSWER: ZOFRAN OR ANTIEMETIC

108. Medical Surgical-Sensory-Pilocarpine-action

A client is newly diagnosed with open-angle glaucoma and receives a prescription

for the meiotic pilocarpin. The client asks how the eyes pressure will be

controlled when the eyes drops are used on the surface. What explanation should

the nurse offer when teaching about the therapeutic action of the ophthalmic

drops?

A- Once the pupil gets smaller, the amount of liquid made inside the eyes is

reduced

B- It is necessary to open the pupil to allow movement of the fluid from behind

the iris

C- The drops will reduce eye swelling which is causing increased ocular

pressure

D- The iris will constrict and contract away from the opening, thereby allowing

it to drain

109. Pathophysiology-Neurological/Physical Assessment-Craniotomy-GCS

When assessing a client who had a supratentorial craniotomy, what action should

the nurse implement when determining the client’s Glasgow coma scale (GCS)

rating?

A- Determine the intracranial pressure.

B- Check the patellar and radial reflexes.

C- Inject cold water into the client’s

ear.D- Instruct the client to raise an

arm.

110. Pathophysiology/Medical Surgical-Immune/Hematology-Antihistamines

Which conditions are most likely to respond to the treatment with antihistamines?

(Select all that apply)

A) Otitis media

B) Allergy rhinitis

C) Contact dermatitis

D) Myocarditis

E) Bronchitis

111. Pathophysiology/Medical Surgical-Immune/Hematology-WBC level-

patho

A female adult who is undergoing chemotherapy tells the nurse that she plans to

volunteer at elementary school this winter. Which question is best for the nurse to

ask this client?

a) “Do you realize that you will be exposed to many different kinds of germs?”

b) “Have you considered that you are putting yourself at risk for developing

infections?”

c) “Are you aware that you do not have a fully functioning immune system?”

d) “Is it possible that you will be in direct contact with the children at the school?”

OJO CON ESTA DE ABAJO

A client’smorning laboratory test results include leukocytes 3,500/mm3 or3.5

x 10???/L (SI). Based on this laboratory result, which complaint is thisclient

most likely to report to the nurse?

a) Inability to walk without shortness of breath.

b) Superficial cuts do not readily stop bleeding.

c) A red streak and pain in right calf muscle.

d) Persistent cough with yellow-colored sputum.

112. Pathophysiology/Medical Surgical-Musculoskeletal-Osteoarthritis-risk

factor

The nurse is assessing a middle-aged adult who is diagnosed with osteoarthritis.

Which factor in the client’s history is a contributor to the osteoarthritis?

a. Lactose intolerant since childhood.

b. Recently treated for deep vein thrombosis.

c. Long distance runner since high school.

d. Photosensitive to a drug currently taking.

113. Pathophysiology/Medical Surgical-Neurological/Trauma/Emergency-

Head injury-diabetes insipidus

Polyuria can be defined as a urine output exceeding 3 L/24 h in adults and 2

L/m2 in children. It must be differentiated from the more common complaints

of frequency or nocturia, which are not associated with an increase in the total

urine output. Differential diagnosis has to be kept in mind when TBI patients

undergoing neurosurgery have huge amounts of urine output because most

cases of polyuria, at this time, are not caused by DI (Seckl and Dunger 1989).

The more common causes are excretion of excess fluid administered during

surgery and an osmotic diuresis, resulting from treatment aimed at

minimizing cerebral edema using mannitol or glucocorticoids (Bohn, Davids

et al. 2005).

114. Pathophysiology/Professional Issues/Medical Surgical-Nursing Process-

Resp acidosis-pneumonia

PNEUMONIA IS DIAPHORETIC AND CONFUSED:

ANSWER: OBSERVE FREQUENTLY

ESTA NO SALIO PERO HAY QUE VERLA

A client with a history of upper respiratory symptoms is admitted to the unit with

chest tightness, productive cough and difficult breathing. The client ABG is

respiratory acidosis. What lab the nurse expects to be high?

a. Ph

b. Arterial ph.

c. HCO3

d. Paco2

115. Pediatrics-Cardiovascular-Left sided heart failure

A nurse is assessing a 2 year-old child with left sided heart failure. Which

assessment finding should the nurse report to the healthcare provider

immediately?

a. Penorbital edema noted bilaterally after napping

b. Crackles heard in lower lobes of lungs bilaterally

c. An apical heart rate of 120 beats per minute

d. Liver palpated 2 cm below right costal margin

116. Pediatrics-GI/Hepatic-Diarrhea-specimen

Stool Specimens

Stool specimens are frequently collected in children to identify parasites and

other organisms that cause diarrhea, to assess gastrointestinal function, and

to check for occult (hidden) blood. Ideally, stool should be collected without

contamination with urine, but in children wearing diapers this is difficult

unless a urine bag is applied. Children who are toilet trained should urinate

first; flush the toilet; and then defecate in the toilet, a bedpan (preferably one

that is placed on the toilet to avoid embarrassment), or a commercial potty

hat.

Stool specimens should be large enough to obtain an ample sampling, not

merely a fecal fragment. Specimens are placed in an appropriate container,

which is covered and labeled. If several specimens are needed, the containers

are marked with the date and time and kept in a specimen refrigerator.

Special care is exercised in handling the specimen because of the risk of

contamination.

No creo q es la rest*A child with diarrhea: Antibiotics

117. Pediatrics-Gwth & Devlp/Physical Assessment-Child interview-school age

Child 9 years old:

A- Talk directly to the child

B- Ask the child if the parents are saying the true

C- Tell the parents to get out of the room

118. Pediatrics-Immune/Hematology- von Willebrand’s disease-POC

(Bleeding disorder)

A- Decrease exposure to infection

B- Decrease contact with other children

C- Decrease contact with cold

graft D- Guard against bleeding

injuries

119. Pediatrics-Integumentary-Burns-hydrotherapy

*Answer: Let the child touch the water

120. Pediatrics-Musculoskeletal- Congenital hip-1st action

2 days old infant legs flexed with limited abduction, what is the next action that the

nurse take:

A- Range of motion exercise

B- Notify MD

C- Document as an normal finding

D- Continue with the care

121. Pediatrics-Neurological-Vegetative state-adolescent

Un Nino en estado vegetative hace 5 meses. RN q hacer

*Answer: Talk to the child

122. Pediatrics- Renal-Nephrotic syndrome-I&O

2 year old child with nephrotic syndrome taking corticosteroids is edematous and

fatigue. What action the nurse implement first.

A- Sign and symptom of Cushing

B- Restrict sodium

C- Intake and output…

D- Measure abdominal girth for 2 days

123. Pediatrics-Respiratory-Cystic fibrosis-med

The nurse is evaluating the home care teaching of a family who has a child with

cystic fibrosis. Which parental action indicates correct understanding of the child’s

home care?

a. Performs postural drainage after meals

b. Supplements diet with water-soluble vitamins and fluids

c. Plans a diet high in fat and calories

d. Gives pancreatic enzymes before every meal and snack

124. Pediatrics-Respiratory-RSV-isolation

*Answer: Contact precautions

125. Pediatrics/Medical Surgical-Respiratory-Theophylline-toxicity PEDI

A 4-YEAR-OLD child hospitalized with asthma is receiving theophylline. Which

observation by the nurse warrants immediate intervention?

A) The child heart rate is 110.

B) The child’s breath sounds indicate bilateral expiratory wheezing.

C) The child is sitting straight up in

bed. D- The child is nauseated and

irritable.

126. Professional Issues-Cultural/Spiritual/Leadership/Legal/Ethical-Post

mortum care

PATIENT DIES AND FAMILY WANT TO SEE HIM BEFORE THE HOME

FUNERAL ARRIVED, THE NURSE SHOULD ENTER FIRST TO THE ROOM

(SELECT ALL THAT APPLY):

A- REMOVE THE RESUSCITATION EQUIPMENT

B- REMOVE THE DENTURES

C- CLOSE HIS EYES

D- PUT A PILLOW UNDER THE HEAD

E- USE A SHROUD BAG

127. Professional Issues-Documentation/Leadership/Legal/Ethical-Team

management-documentation

When the nurse manager and the nursing staff review entries into the electronic

medical records (EMR), they determine that procrastination is often the reason

for late-entries. What recommendation should the nurse manager offer to the

nursing staff?

A- Document routine care as provided and complex care at the end of

shift B- Enter tasks in the EMR as the client’s priority needs are addressed

C- Document nursing care procedures between time-dependent cares

D Keeps notes and enter all documentation at the end of the shift

128. Professional Issues-Leadership-Conflict resolution-arguing staff

Two unlicensed assistive personal (UAP) are arguing loudly in the hallway of an

extended care facility about who will shower a male resident who defecated in his

bed. What action is best for the charge nurse to take?

A) Instruct both UAPs are to shower the client immediately.

B) Shower the client with the help of a practical nurse.

C) Document the conflict in the employee personnel files.

D) Reassign the client’s care to another staff member.

129. Professional Issues-Leadership/Legal/Ethical-Decision to strike

NURSING STRIKE BARGAINING:

A- THE ROLE OF THE NURSE AS A PROFESSIONAL

(Esta no me recuerdo las otras opciones pero esa no es la respuesta)

(I don’t remember the other options but this is not the answer)

130. Professional Issues-Leadership/Legal/Ethical-Delegate-follow- up

As team leader, the nurse is caring for a group of clients with the help of a practical

nurse (PN) and experienced unlicensed assistive personnel (UAP). Which nursing

actions should the nurse assign to the PN? (Select all that apply)

A- Change surgical dressing daily for a client who had an abdominal

hysterectomy B- Obtain postoperative vital signs for a client with an epidural

analgesic after having a knee arthroplasty

C- Start a blood transfusion for client who just returned to the room following a

below knee amputation

D- Administer a dose of insulin per sliding scale for a client with type 2 diabetes

mellitus (DM)

OJO ESTA NO FUE LA QUE SALIO

131. Professional Issues-Leadership/Legal/Ethical-Prescription-poorly written

The unit clerk reports to the charge nurse that a healthcare provider has written

several prescriptions that are eligible and it appears the healthcare provider used

several abbreviations in the prescription. What action should the charge nurse

take?

a. Report the situation to the house supervisor

b. Complete and incident (variance) report

c. Call the healthcare provider who wrote the prescriptions

d. Contact the healthcare provider review board for instructions

132. Professional Issues-Leadership/Legal/Ethical-Sexual harassment

A- “I know you are a good nurse and can handle this client”

(Solo me recuerdo de esta opcion xq fue la que puse pero me salio mal)

(This is not correct but I just remember this option)

Possible asw: cambiar el pt creo q es

133. Professional Issues-Leadership/Legal/Ethical-UAP assign-escort client

*Answer: Transfer the patient in the wheelchair to another room

134. Professional Issues-Leadership/Legal/Ethical/Nursing Process-Advocate

135. Professional Issues/Medical Surgical-Documentation-Heart

sounds- murmur

SOUND

A- Murmur

B-S1, S2

C- S1, S2, S3

D- Peripheral (No me recuerdo bien esta opcion)

136. Professional Issues/Medical Surgical-Legal/Ethical-Forensic

nursing clothing

A young adult male is brought to the emergency room with a multiple gunshot

wounds in the chest abdomen, and head. After collecting the client’s blood-

saturated clothing as forensic evidence or the medical examiner, which action

should the nurse implement?

A) Drop the clothes in a plastic bag and seal the bag with transported tape.

B) Place clothing in a large specimen container and send to the

pathology lab.

C) Place the folded clothes in a paper bag transfers it to red biohazard bag.

D) Roll the clothing in a towel and cover it with an impermeable drape.

137. Professional Issues/Medical Surgical-Legal/Ethical-SBAR-call

HCP- femoral stent

ABDOMINAL LEFT FEMORAL ANGIOPLASTY:

A- SURGEON NEEDS TO SEE

B- LEFT PHERIPHERAL PULSES

138. Professional Issues/Medical Surgical-Teaching-Dronedarone-client

instructions

DRONEDARONE medication and the pt take grapefruit what we need to teach

indiet.

A- DISCONTINUE

B- AVOID

C- NOTIFY

(WE HAVE TO VERIFY THIS ONE)

139. Professional Issues/Medical Surgical-Teaching-Orchiectomy-wound care

Testiculo remove: asw: … Support

140. Psychiatric/Mental Hlth-Abuse-CAGE scoring

The questionnaire asks the following questions

•Have you ever felt you needed to Cut down on your drinking?

•Have people Annoyed you by criticizing your drinking?

•Have you ever felt Guilty about drinking?

•Have you ever felt you needed a drink first thing in the morning (Eye-

opener) to steady your nerves or to get rid of a hangover?

The CAGE questionnaire, among other methods, has been extensively

validated for use in identifying alcoholism. CAGE is considered a

validated screening technique, with one study determining that CAGE

test scores >=2 had a specificity of 76% and a sensitivity of 93% for

the identification of excessive drinking and a specificity of 77% and a

sensitivity of 91% for the identification of alcoholism.

The nurse is with a patient doing a CAGE questionary 3 positive response. What

the nurse….

A- Is a questionary for substance abuse

B- 1 positive seek help for alcohol dependence

C- Al least 2 positive strongly alcohol dependence

D- All positive suggest alcohol dependence

141. Psychiatric/Mental Health-Anxiety/Communications-Communication-

caregiver support

The home care nurse go to visit a patient with Alzheimer’s and find the wife

crying, what happen with your husband and the wife respond “watch it with your

own eyes”. What action should the nurse…?

A- Encourage wife to leave home

B- Ask the wife to observe the assessment to learn how to take deal with the

situation

C- As soon as the client care is completed provide wife with family support

group

D- Sit with the wife and talk about her concerns

142. Psychiatric/Mental Health-Anxiety/Communications-PTSD-maladaptive

behavior

ADULT MALE WITNESSED THE MURDER:

A- “IT’S BETTER THAN KILLING SOMEONE”

B- “TELL ME MORE ABOUT THE MURDERYOU

RECENTLY WITNESSED”

C- “YOU FEEL GUILTY FORTHE MURDER”

143. Psychiatric/Mental Health-Anxiety/Communications/Psychoses-Manic

acting out

A group of students along with the nurse are on a tour of the hospital in the area of

psychiatry and while they were down the hall one patient says “want to see a crazy

patient” and start to jumpand scream and make handslike a chicken . What

should the nurse do?

A- Ignore the patient and continue with a tour

B- Give PRN anxiety medication

C- Call the security

D- shake it and bring it to normal state

144. Psychiatric/Mental Health/Fundamentals-Abuse/Basic Nursing

Skills/Mobility/Safety-Restrains-physical harm

Chequiar la circulation en la restrains or reposition the restrains

145. Psychiatric/Mental Hlth/Fundamentals-Abuse/Basic Nursing

Skills/Nutrition-Bulimia-maintain weight

Asw:Scheduled meal and snack

Bulimia nervosa is an eating disorder characterized by binge eating and purging,

or consuming a large amount of food in a short amount of time followed by an

attempt to rid oneself of the food consumed (purging), typically by vomiting,

taking a laxative, diuretic, or stimulant, and/or excessive exercise, because of an

extensive concern for body weight.

146. Psychiatric/Mental Health/Fundamentals-Anxiety/Communications/Basic

Nursing Skills/Safety-Aggression-triggering

A client who has a history of aggressive and hostile behavior is in the triggering

phase of the aggression cycle. Which action should the nurse implement first?

A) Encourage the client to verbalize angry feeling.

B) Obtain staff assistance to confront the client.

C) Administer a PRN medication to the client

D) Physically escort the client to a quiet cooling off area.

147. Psychiatric/MentalHealth/Fundamentals/Maternity/ProfessionalIssues-

Depress/Grief/Basic Nursing Skills/Nutrition/Postpartum/Nursing Process-PP

depression-goals-POC

Una pt conpostpartun y dejo de comer y perdio peso . Short

goal Asw: 100ml and 3 food

148. Psychiatric/Mental Health/Fundamentals/Medical Surgical-

Psychoses/Basic Nursing Skill/Safety-Priority-Endocrine Unit

A male client, who had a total laryngectomy two days ago, is transferred from

the intensive care unit to a private room close to the nurse’s station. The nurse

recognizes that the client is anxious. Which intervention should the nurse

implement?

a. Encourage a family member to stay with the client at all times

b. Answer the client’s call signal in person quickly after the calls

c. Explain the emergency procedure for loss of airway to the client

d. Provide the client with a suction catheter to allow for self-suctioning

149. Psychiatric/Mental Health/Geriatrics/Medical Surgical-

Anxiety/Communications-Colostomy-postop confusion . Psychiatric/Mental

Health/Maternity-Abuse/Newborn-Neonate-cocaine-withdrawal

A neonate whose mother used cocaine during pregnancy is demonstrating

excessive shrill cry, and frequent vomiting. What action should the nurse perform

first?

A. Request a neurology assessment.

B. Wrap the infant in warm

blankets. C. Obtain a serum screen.

D. Burp the infant to eliminate gas.

151. Psychiatric/Mental

Health/Maternity/Anxiety/Communications/Intrapartum-Angry family

member

The patient was in pain and mom was saying she was ready for cesarean because

she knew what had already had 8 children.

A- Call the security

B- Ask her to leave the room

C- Call the charge nurse

152. Psychiatric/Mental Health /Medical Surgical-

Anxiety/Communications- Hip fracture

An elderly patient who lives alone and falls, hip fractures and goes to hospital. She

was worried about her dog. (Select all that apply):

a- Put 2 pillows

b- PRN med

c- Contact social worker

d- Ignore the patient

153. Psychiatric/Mental Health/Pathophysiology/Medical Surgical-Abuse-

Hepatic encephalopathy ( ammonia level alto)

A male client with a long history of alcoholism is admitted because of mild

confusion and fir motor tremors. He reports that he quit drinking alcohol and

stopped smoking cigarettes one month ago after his brother died of lung cancer.

Which intervention is most important for the nurse to include in the client’s plan

of care?

A) Observe for changes in level of consciousness

B) Provide grief counseling for client and his family

C) Involve the client’s family in healthcare decision

D) Determine client’s current blood alcohol level.

154. Psychiatric/Mental Health/Pediatrics-Depress/Grief-Grief-adolescent

The child at school said that had a lot of headache and go to the nurse. What

comment made by the child concerned the nurse?

Answer: The child says something that wants to see his mom

155. Psychiatric/Mental Health/Professional Issues-

Anxiety/Communications/Leadership/Legal/Ethical-EOL care-fear

Patient in hospice care at home fear dying will be

painful… A- Encourage to talk about….

B- Explain that pill will be given…

C- Provide therapeutic touch with comfort and support

156. Psychiatric/Mental Health/Professional Issues-

Anxiety/Communications/Leadership/Legal/Ethical-Med error-

communication

2 nurse discutiendo en el pasillo es el pt escucho que le dieron med mal. Q

hacer?

Asw: apologies with the pt

157. Psychiatric/Mental Health/Professional Issues-Depress/Grief/Nursing

Process-Depression-nursing problem

158. Psychiatric/Mental Health/Professional Issues-

Psychoses/Documentation/Nursing Process-Delusions-POC

Deliria/delusions

Delusions are beliefs that guide one’s interpretation of events and help make

sense out of disorder. The delusions may be comforting or threatening, but

they always form a structure for understanding situations that otherwise

might seem unmanageable. A delusional disorder is one in which conceivable

ideas, without foundation in fact, persist for more than 1 month. These beliefs

are not always bizarre and do not originate in psychotic processes. Common

delusions are of being poisoned, being followed, their children taking their

assets, being held prisoner, or being deceived by a spouse or lover.

One older woman persistently held onto the delusion that her son was coming

to pick her up and take her home, although her son had been dead for 10

years.

159. Psychiatric/Mental Health/Professional Issues-Medical

Surgical- Abuse/Anxiety/Communications/Legal/Ethical-Date rape-

denial

The patient from college that was drinking last night with friends. Go to the

hospital…

A- I’m sorry to hear this

B- You remembered if someone put something in the drink or if she remember

what she drink

C- You know the people who did this

D- You feel guilty about what happened to you

160. Psychiatric/Mental Health/Professional Issues-Medical Surgical

-Anxiety/Communications/Depress/Grief/Leadership-Liver transplant-anger

The RN sends the UAP to the room to do care to the patient but the patient was

anger and yelling to the UAP. What can the UAP do?

A- Schedule the care daily

B- Not enter more in the room

C- Give care earlier

D- Give care options participate

EXTRA QUESTIONS:

1- A PATIENT WITH CYSTIC FIBROSIS IS… HUMAN

DEOXYRIBONUCLEASE. WHAT FINDING REQUIRE THE

NURSE INTERVENTION?

A- INCREASE MUCUOS THINNED

B- INCREASE 2 POUNDS

C- DECREASE FREQUENCY STEATORRHEA

2- LARGE BLISTER (BACK) CHEST SOAKED SEROSANGUINIOUS

DURING ASSESSMENT. WHAT FINDING REQUIRES IMMEDIATE

INTERVENTION?

A- HEADACHE

B- FEVER/CHILLS

C- DECREASE BLOOD PRESSURE

D- DIZZNESS

3- TWO RN QUE DISCUTIAN POR UN MEDICAMENTO Y EL

PACIENTE ESCUCHA QUE ERA LO PRIMERO QUE HACIA EL

CHARGE NURSE:

-CHARGE NURSE APOLOGIZE WITH THE PT

-LOOK FOR DOCUMENTATION

-MEETING WITH 2 NURSES AND REVIEW THE PRIVACY OF THE

PT AND POLICY OF THE HOSPITAL

Hesi med surge

review Package of

Pictures

2 of 55

What instruction should the nurse include in the discharge teaching plan of

a client who had a cataract extraction today?

a. Sexual activities may be resumed upon return home

b. Light housekeeping is permitted but avoid heavy lifting

c. Use a metal eye shield on operative eye during the day

d. Administer eye ointment before applying eye drops

ANS: B

3 of 55

A male adult comes to the urgent care clinic 5 days after being diagnose

with influenza.Heisshortofbreath,febrile,andcoughinggreencolored

sputum. Which intervention should the nurse implement first?

a. Obtain a sputum sample for culture

b. Check his oxygen saturation level

c. Administer an oral antipyretic

d. Auscultate bilateral lung sound

ANS: A

4 0f 55

An elder male client tells the nurse that he is loosing sleep because he has to

get up several times at night to go to the bathroom that he has trouble starting

his urinary stream and that he does not feel like his bladder is ever completely

empty. Which intervention should the nurse implement?

a. collect a urine specimen for culture analysis

b. obtain a fingerstick blood glucose level

c. palpate the bladder above the symphysis pubis

d. review the client fluid intake

ANS: C

TestBankWorld.org

5 of 55

An adult client is admitted with diabetic ketoacidosis (DKA) and a urinary tract

infection (UTI) Prescriptions for intravenous antibiotics and insulin infusion are

initiated. Which serum laboratory value warrants the most immediate intervention

by the nurse?

a. blood ph of 7.30

b. glucose of 350 mg /dl

c. white blood cell count of 15000mm

d. potassium of 2.5 meq/l

ANS: D

6 of 55

A client with sickle cell anemia develops a fever during the last hour of

administration of a unit of packed red blood cell. When notifying the healthcare

provider what information should the nurse provide first using the SBAR

communication process?

a. explain specific reason for urgent notification

b. preface the report by stating the clients name and admitting diagnosis

c. communicate the pre-transfusion temperatures

d. optain prn prescription for acetaminophen for fever 101f

ANS: A

7 of 55

An adult male client is admitted for pneumocystis carinil pneumonia (PCP)

secondary to aids. While hospitalize he receives IV pentamidine isethionate

therapy. In preparing this client for discharge what important aspect regarding his

medication therapy should the nurse explain?

a. AZT therapy must be stopped when IV aerosol pentamine is being used.

b. IV pentamine will be given until oral pentamine can be tolerated

c. It will be necessary to continue prophylactic doses of IV or

aerosol pentamine every month

d. Iv pentamine may offer protection to others aids related conditions such

as kaposis sarcoma

ANS: C

8 of 55

A client subjective data includes dysuria, urgency, and urinary frequency. What

action should the nurse implement next?

a.collect a clean catch specimen

b.palpate the suprapubic region

c.instruct to wipe from front to

back

d.inquire about recent sexual activity

ANS: A

9 of 55

A client tells the nurse that her biopsy results indicate that the cancer cells are

well differentiated How should the nurse respond?

a. offer the client reassurance that this information indicates that the

clients cancer cells are benign

b. explain that these tissue cells often respond more effectively to

radiation than to chemotherapy

c. ask the client in the healthcare provider has giving her any

information about the classification of her cancer

d. help the client make plans to begin inmediate treatment since her

cancer is likely to spread quickly

ANS: C

10 of 55

A client with a chronic kidney disease is treated on hemodialysis. During the 1

treatment clients blood pressure drops from 150/90 to 80/30 Which action should

the nurse take first?

a. monitor bp q45 minutes

b. lower the head of the chair and elevate feet

c. stop dialysis treatment

d. administer 5%albumin IV

ANS: C

11 of 55

A client with deep vain thrombosis (DVT) is receiving a continues infusion of

heparin sodium 25,000 unit in 5% dextrose injection 250ml. The prescription

indicates the dosage should be increase 900 units/hr. The nurse should program

the infusion pump to deliver how many ml/hr?

=9

12 of 55

The nurse is obtaining the admission history for a client with suspected peptic

ulcer disease (PUD). Which subjective data reported by the client supports this

diagnosis?

a. upper mid abdominal gnawing and burning pain

b. severe abdominal cramps and diarrhea after eating spicy foods

c. marked loss of weight and appetite over the last few months

d. use of chewable and liquid antacids for indigestion

ANS: A

13 of 55

The nurse is providing preoperative education for a jewish client schedule to

receive a xenograft graft to promote burn healing. Which information should the

nurse provide this client?

a.the xenograft is taken from nonhuman sources

b.grafting increases the risk for bacterial infection

c.as the burn heals the graft permanently attaches

d.grafts are later removed by debriding procedure

ANS: A

14 of 55

A client who took a camping vacation two weeks ago in a country with a tropical

climate comes to the clinic describingvague symptoms and diarrhea for the

past week. Which finding is most important for the nurse to report?

a. jaundice sclera

b. intestinal cramping

c. weakness and fatigue

d. weight loss

ANS: A

15 of 55

During a home visit the nurse assesses the skin of a client with eczema who

reports than an exacerbation of symptoms has occurred during the last week.

Which information is most useful in determining the possible cause of the

symptoms?

a. an old friend with eczema came for visit

b. recently received an influenza immunization

c. corticosteroid cream was applied to eczema

d. a grandson and his new dog recently visited

ANS: D

16 of 55

When explaining dietary guidelines to a client with acute glomerulonephritis

(AGN) which instruction should the nurse include in the dietary teaching ?

a. select a protein rich food daily

b. restrict sodium intake

c. eat high potassium foods

d. Avoid foods high in carbohydrate

ANS: B

17 of 55

A male client who is 24hr post operative for an exploratory laparoctomy

complains that he is starving because he has had no real food since before

surgery. Prior to advancing his diet which intervention should the nurse

implememt?

a. discontinue intravenous therapy

b. Assess for abdominal distension and tenderness

c. Obtain a prescription for a diet change

d. Auscultate bowel sound in all four quadrants

ANS: D

18 of 55

A client diagnose with stable angina secondary to ischemic heart disease has a

prescription for sublingual (SL) nitroglycerin (NTG). The nurse should tell the

client to follow which instructions if chest pain is not relieved after taking 3 NTG

tables 5 min apart?

a. drive to the nearest emergency department

b. take another NTG SL tablet and lie down until angina subsides

c. call primary healthcare provider

d. call 911 pain is unrelieved and chew a tablet of aspirin 325mg

ANS: D

18a. of 55——Esta es otra 18 que hay en fotos pero es diferente

After taking orlistat (Xenical) for one week a femela client tells the home health

nurse that she is experiencing increasingly frequent oily stools and flatus.

What action should the nurse take?

a. obtain stool specimen to evaluate for occult blood and fat content

b. instruct the client to increase her intake of saturated fats over the

next week

c. ask the client to describe her dietary intake history for the last

several days

d. advice the client to stop taking the drug and contact the

healthcare provider

ANS: C

19 of 55

Two days after an abscess of the chin was drained the client returns to the clinic

with fever chills and a maculopapular rash with pruritis. The client has taken an

oral antibiotic and cleansed the wound today with provide iodine (Betadine)

solution. Which intervention should the nurse implement first?

a. determine if the client has a history of diabetes

b. assess airway patency and oxygen saturation

c. review recent medication history and allergies

d. obtain samples for complete blood count and cultures

ANS: B

21 OF 55

A client experiences an ABO incompatibility reaction after multiple blood

transfusions. Which finding should the nurse report immediately to the health

care provider?

a. low back pain and hypotension

b. rhinitis and nasal stuffiness

c. delayed painful rash with urticarial

d. arthritic joint changes and chronic pain

ANS: A

22 of 55

A young adult male who has had type 2 diabetes mellitus (DM) is admitted to the

intensive care unit with hyperglycemic nonketotic symdrome (HHNS). A sliding

scale protocol for an isotonic IV solution with regular insulin is prescribed based

on the results of a continuous blood glucose monitoring device that is attached to

the clients central venous catheter. When the clients respirations become

labored and his lungs sound indicate crackles what action should the nurse

take?

a. collect a specimen for a white blood cell count and cultures

b. determine the clients glycosylated hemoglobin (A1C)

(POSSIBLE ANSWER)

c. administer insulin IV push until the clients fluid volume is adjusted

d. decrease infusion rate to address fluid overload

ANS: D

22 of 55——ESTA ES OTRA 22 QUE ESTABA EN FOTOS PERO ES

DIFERENTE

When preparing to apply a fentanyl (Duragesic) transdermal patch the nurse

notes that the previously applied patch is intact on the clients upper back and the

client denies pain. What action should the nurse take?

a. Remove the patch and consult with the healthcare provider about

the client pain resolution

b. Place the patch on the clients shoulder and leave both patches in place

for 12 hours

c. Administer an oral analgesic and evaluate its effectiveness

before applying a new patch

d. Apply a new patch in a different location after removing the original patch

ANS: D

25 of 55

A client who had a myocardial infarction is admitted to the coronary critical care

unit (CCU) with a nitroglycerin drip infusing. The clients last blood pressure

measurements was 78/36.What action should the nurse implement?

a. obtain blood pressure q5 minutes using duranap machine

b. change the dilution of the nytroglycerin infusion

c. reduce the rate of the nitroglycerin infusion

d. begin dopamine infusion at 5mcg/kg per minute

ANS: C

26 of 55

An adolescent is admitted to the hospital because of a suicide attempt with an

overdose of acetaminophen (Tylenol). Which blood values are most important

for the nurse to monitor during the first 72 hours following ingestion of this

overdose?

a. BUN creatinine specific gravity

b. White blood count, hemoglobin hematocrit

c. PH,PCO2, HC03

d. LDH OR LD, SGOT OR ALT, SGPT OR AST.

ANS: D

27 OF 55

An elderly post operative female client is receiving morphine sulfate via a PCA

pump. Which assessment finding should prompt a nurse to administer the

prescribed PRN medication naloxone?

a. her respiratory rate is 7 breath/minute

b. she indicates that she feels as if she cannot get enough air to breath

c. she has intercostal retractions and bilateral wheezing is auscultated

d. her pulse oximeter is 89% on room air

ANS: A

28 of 55

Which assessment finding indicates to the nurse that the muscarinic agent

bethanechol (Urecholine) is effective for a client diagnose with urinary retention?

a. urinary output equal to intake

b. no terminal urinary dribbling

c. denies stress incontinence

d. absence of xerostomia

ANS: A

29 of 55

Followinginvolvementina motor vehicle collision,a middle aged adult client

is admitted to the hospital with multiple facial fractures. The clients blood

alcohol level is high on admission. Which PRN prescription should be

administer if the clients begins to exhibit signs and symptoms of delirium

tremens (DT s)?

a. Lorazepam (Ativan) 2mg IM

b. Chlorpromazine (thorazine) 50 mg IM

c. Prochlorperazine (Compazine) 5 mg IM

d. Hydromorphone (Dilaudid) 2 mg IM

ANS: A

33 of 55

Which instructions should the nurse include in the teaching plan of a client who is

taking the diuretic spironolactone (Aldactone)?

a. call the healthcare provider f you develop gynecomastia

b. Take the medication in the morning

c. Avoid caffeine and smoking

d. Increase your consumption of bananas and oranges

ANS: B

34 of 55

A glucagon emergency kit is prescribed for a client with type 1 diabetes mellitus.

When should the nurse instruct the client to take the glucagon?

a. after meals to increase endogenous insulin secretion

b. after insulin administration to prevent hypoglycemia

c. when recognized signs of severe hypoglycemia occur

d. when unable to eat during sick days

ANS: C

35 of 55

A client with hyperthyroidism is being treated with radioactive iodine (I-131).

Which explanation should be included in preparing this client for this treatment?

a. describe radioactive iodine as a tasteless, colorless

medication administered by the healthcare provider

b. explain the need for using lead shields for 2 to 3 weeks after the treatment

c. describe the signs of goiter because this is a common side effects

of radioactive iodine

d. explain that relief of the signs/ symptoms of hyperthyroidism will

occur immediately

ANS: A

36 OF 55

A female client is being treated for tuberculosis with rifampin (rifadin) Which

statement indicates that futher teaching is needed?

d- I will take my usual contraceptive for birth control

ESTA PREGUNTA ESTABA ESCRITA EN LAS OJAS DE FOTOS

38 of 55

A client is discharged with a prescription for warfarin ( Coumadin). What

discharge instructions should the nurse emphasize to the client?

a. take a multi vitamin supplement daily

b. use an astringent for superficial bleeding

c. avoid going barefoot especially outside

d. include large amounts of spinach in the diet

ANS: C

39 of 55

In caring for a client with diabetes insipidus who is receiving an antidiuretic

hormone intranasally which serum lab test is most important for the nurse to

monitor?

a. osmolality

b. calcium

c. platelets

d. glucose

ANS: A

40 OF 55

After administering dihydroergotamine (Migranal) 1 mg subcutaneously to a client

with a severe migraine headache the nurse should explain that relief can be

expected within what time frame?

a. 2 hours

b. 5 minutes

c. 1 hour

d. 15 minutes

ANS: D

41 of 55

A client with hypertension who has been taking labetalol for two weeks, reports a

five pound (2.2 kg) weight gain. Which follow up assessment is most important

for the nurse to obtain?

a. capillary refill

b. body temperature

c. muscle strength

d. breath sounds

ANS: D

43 of 55

A male client is receiving pilocarpine hydrochloride (Isopto Carpine) ophthalmic

drops for glaucoma. He calls the clinic and ask the nurse why he has difficulty

seeing at night. What explanation should the nurse provide?

a. The eye drops slow pupil response to accommodate for darkness

b. The drops increase the fluid in the eyes and cloud the visual field

c. The drug can cause lens to become more opaque

d. The medication causes pupils to dilate which reduces night vision

ANS: A

44 of 55

A client who is taking and oral dose of tetracycline complains of gastrointestinal

upset. What snack should the nurse instruct the client to take with the

tetracycline?

a. toasted wheat bread and jelly

b. cheese and crakers

c. cold cereal with skim milk

d. fruit flavored yogurt

ANS: A

45 of 55

The therapeutic effect of insulin in treating type 1 diabetes mellitus is based on

which physiologic action?

a. Facilitates transport of glucose into the cell

b. Increases intracellular receptor site sensitivity

c. Stimulates function of beta cells in the pancreas

d. Delays carbohydrates digestion and absorption

ANS: A

46 of 55

The health care provider prescribe a medication for an older adult client who is

complaining of insomnia. And instructs the client to return in 2 weeks. The nurse

should question which prescription?

a. Eszoplicone (Lunesta)10 mg orally at bed time

b. Zolpidem 10 mg orally at bed time

c. Temazepan orally at bed time

d. Ramelteon orally at bedtime

ANS: A

47 of 55

A male client reports to the nurse that he is experiencing GI distress from high

dose of a corticosteroid and is planning to stop taking the medication. In

response to the clients statement what nursing action is most important for

the nurse to take?

a. Encourage the client to take medication with food to decrease GI distress

b. Advice the client that the medication should be stopped gradually

rather than abruptly.

c. Review the clients dosing schedule to ensure he is taking the

prescribed amount

d. Assess the client for other indication of adverse effects of corticosteroid

ANS: B

48 of 55

Fifteen minutes after receiving sulfa athenozole . A male client report a burning

sensation over his abdomen chest and groin. Which intervention is most

important for the nurse to implement?

a. Auscultate lung sounds for wheezing

b. Review the clients list if drugs allergies

c. Add sulfamethinozole to clients allergies

d. Check neurological vital signs

ANS: B

49 of 55

Antibiotic resistant organism are a major infection control problems. To help

minimize the emergence of resistant bacteria what instruction should the nurse

provide to the clients?

a. stop taking prescribed antibiotics when symptoms decrease

b. avoid using antibiotics when suffering from colds or the flu

c. ask the healthcare provider to prescribe the newest antibiotic

when needed

d. request a prescription for first time vancomysin for a sore throat

ANS: B

50 of 55

A clientwith symptoms of influenza that started the previous day ask the

clinic nurseabouttakingoseltamivir(Tamiflu)totreattheinfection.Which

response should the nurse provide?

a. Advise the client once symptoms occur is too late to receive an

influenza vaccination

b. Refer the client to the healthcare provider at the clinic to obtain

a medication prescription

c. Explain to the client that antibiotics are not useful in treating

viral infections such as influenza

d. Instruct the client that over the counter medications are sufficient

to manage influenza symptoms

ANS: B

51 of 55

Twenty minutes after the nurse starts a secondary IV infusion of cafepime

(maxipime) 2 grams using an infusion pump to deliver the dose in one hour, the

client reports feeling nauseated. What action should the nurse implement?

a. stop medication infusion and notify the healthcare provider of the

adverse effect

b. increase the rate of the infusion to complete the dose of the

medication more rapidly

c. continue the infusion and administer a prn antiemetic prescription

d. reasurre the client that the nausea is not related to the iv infusion

ANS: C

52 of 55

The nurse administer donepezil hydrochloride (Aricept) to a client with

Alzheimers disease as an intervention for which client problem?

a. fluid volume excess

b. disturbed though processes

c. chronic pain

d. altered breathing patterns

ANS: B

53 of 55

To prevent deep vein thrombosis following knee replacement surgery, an adult

male client is receiving enoxaparin (Lovenox) subcutaneously daily. Which

laboratory finding requires immediate action by the nurse?

a. blood urea nitrogen (BUN) 20mg/dl or 7.1 mmol/L (SI)

b. Hematocrit 45%

c. Serum creatinine 1.0 mg/dl or 88.4 mol/L (SI)

d. Platelet count of 100,000/mm3 or 100×10??/ L (SI)

ANS: D

54 of 55

A client with type 2 diabetes mellitus is managed with metformin (Glucophage),

an oral hypoglycemic agent. The primary health care provider prescribes ad

additional medication injected exenatide (byetta). Which information is most

important for the nurse to teach this client?

a. Administer subcutaneously after meals

b. Consume additional sources of potassium

c. Notify the healthcare provider if anorexia occurs

d. Watch for signs of jitteriness or diaphoresis

ANS: B

55 of 55

A client is who is diagnose with schizophrenia receives a prescription for a

atypical antipsychotic drug aripipazole (Abilify). Which assessment should the

nurse perform to monitor for an adrenergic receptor antagonist side effect

that commonly occurs atypical antipsychotic agents?

a. observe the client hallucinatory behaviors

b. obtain the client fingerstick glucose levels

c. measure the clients lying and standing blood pressure

d. determine the clients abnormal involuntary movements scale (AIMS)

ANS: B

**Packete que empieza con HESI MED SURGE 2 Y

HESSI RETAKE AQUI

ESTA RESUMIDO SIN COSAS REPETIDAS

1- A client with pheocromocytoma reports the onset of a severe headache.

The nurse observes that the client is very diaphoretic. Which assessment

data should the nurse obtain first.

Blood pressure

2- The drainage in the chest tube of a client with emphysema has

changed from clear watery fluid. What action would be best for the

nurse to take/

Maintain the current IV antibiotic schedule

3- A client is admitted with a sudden onset of right sided the nurse

complete first?

Observe for peripheral edema

4- When planning care for a client newly diagnose with open angle

glaucoma, the nurse identifies a priority nursing diagnosis of “

Visual sensory/perceptual alterations”. This diagnosis is based on

which etiology?

Decreased peripheral vision

5- A client in the operating room received succinylcholine. The client is

experiencing muscle rigidity and has an extremely high temperature.

What action should the nurse implement?

Call the PACU nurse to prepare for prolonged ventilatory support

Also know that PACU is BP, Respiration and Pulse

6- A client who is receiving packed red blood cells develops nausea

and vomiting. What action should the nurse take first?

Stop the infusion of blood

Te lo pueden poner como hemodialysis y tambien es STOP transfusion

7- A client with type 2 diabetes mellitus is admitted to the hospital for

uncontrolled DM. Insulin therapy is initiated with initial dose of

Humulin insulin at 8:00 at 16:00 the client complains of diaphoresis,

rapid heart beat, and feeling shaky. What should the nurse do first?

Determine the client current glucose level

8- After suctioning the patient with an endotracheal tube, which

assessment finding indicates to the nurse that the intervention was

effective?

Increase in breath sounds

9- The nurse observes an increase number of blood clots in the

drainage tubing of a client with continuous bladder irrigation following

a transurethral resection of the prostate (TURP). What is the best

initial nursing action?

Provide additional oral fluid intake

Also with TURP you must know that 3l of water a day is needed

10-Which nursing diagnosis should be selected for a client who is

receiving thrombolytic infusions for treatment of an acute myocardial

infarction?

Risk for injury related to effects of thrombolysis

11-The nurse is assessing a client who has returned from surgery following

a thoracotomy. Which finding indicates the client is experiencing

adequate gas exchange?

The client demonstrates effective coughing and deep breathing exercises

12-When caring for a client with nephrotic syndrome which assessment

is most important for the nurse to obtain?

Daily Weight

13-A client who had a biliopancreatic diversion procedure (BOP) 3 months

ago is admitted with severe dehydration. Which assessment finding

warrants immediate intervention by the nurse?

Gastroccult positive emesis

14- A female client with possible acute renal failure (ARF) is admitted to the hospital

and mannitol (Osmitrol) is prescribed as a fluid challenge. Prior to carrying out this

prescription, what intervention should the nurse implement?

•No specific nursing action is required

•Instruct the client to empty the bladder

•Collect a clean catch urine specimen

•Obtain vital signs and breath sounds

ANS: D

15- The nurse positions a male client for a lumbar puncture by placing him in the side-

lying position with his knees flexed and pulled toward his trunk. What action should

the nurse implement next?

•Call another nurse to assist the healthcare provider

•Provide a small pillow for the client to curl around

•Instruct the client to perform a Valsalva maneuver

•Support the client’s head bent forward to the chest

ANS: D

16- When teaching a client with osteoporosis to increase weight-bearing exercise,

how should the nurse explain the purpose of this activity?

•Strengthen leg muscles

•Promote venous return

•Increase bone strength

•Restore range of motion

ANS: C

17- A male tells the clinic nurse that he is experiencing burning on urination, and

assessment that he had sexual intercourse four days ago with a woman he casually

met. Which action should the nurse implement?

•Observe the perineal area for a chancroid-like lesion

•Obtain a specimen of urethral drainage for culture

•Identify all sexual partners in the last four days

•Assess for perineal itching, erythemia, and excoriation

ANS: D

18- An older female client with long term type 2 diabetes mellitus (DM) is seen in the

doctor routine health assessment. To determine if the client is experiencing any long-

term complications of DM, which assessments should the nurse obtain? Select all that

apply:

•Visual acuity A

•Serum creatinine and blood urea nitrogen (BUN)——B

•Signs of respiratory tract infection————————C

•Sensation in feet and legs———————————–D

•Skin condition of lower extremities———————–E

ANS: A ; B ; D ; E .

19- Which laboratory test result is most important for the nurse to report to the

surgeon prior to a client’s scheduled abdominal surgery?

•Potassium level of 4 mEq/liter

•Blood glucose of 90 mg/dl

•Serum creatinine of 5 mg/dl (POSSIBLE ANSWER)

•Hemoglobin level of 13 grams

ANS: D

20- A client who has a history of long-standing back pain treated with methadone

(Dolophine), is admitted to the surgical unit following urological surgery. What

modifications in the plan of care should the nurse make for this client’s pain

management during the postoperative period?

•Use minimal parenteral opioids for surgical pain, in addition to

oral methadone

•Maintain client’s methadone, and medicate surgical pain based on pain rating

•Consult with surgeon about increasing methadone in lieu of parenteral opioids

•Make no changes in standard pain management for this surgery and

hold methadone.

ANS: B

21- The nurse applies an automatic external defibrillator (AED) to a client who

collapsed in an exam room at a community clinic. What action should the nurse take

next?

•Determine the defibrillator reading

•Assess the client’s oxygen saturation

•Bring a crash cart to the exam room

•Measure the client’s blood pressure

ANS: B

22- Which change in lab values would indicate to the nurse that treatment for gout

is successful?

•Decreased serum uric acid

•Decreased serum purine

•Increased serum uric acid

•Increased serum purine

ANS: A

23- The nurse reports that a client is at risk for a brain attack (stroke) finding?

•Jugular vein distention

•Palpable cervical lymph node

•Carotid bruit

•Nuchal rigidity

ANS: D

24- The nurse is assessing a group of older adults. What factor in a male client’s

history puts him at greatest risk for developing colon cancer?

•Is excessively exposed to sunlight

•Eats a high-fat diet

•Smokes cigars (POSSIBLE ANSWER)

•Has intestinal polyps

ANS: D

25- While taking routine vital signs at 0400 AM, the nurse notes that a client who had a

total knee replacement the previous day has a heart rate of 126 beats/minute. What

action should the nurse take first?

•Compare heart rate trends with blood pressure trends ( POSSIBLE ANSWER)

•Review the medical record for a history of cardiac disease

•Check surgical drainage system and bandage for bleeding

•Determine current pain level using a 10-point scale

ANS: C

26- A client who suffered an electrical injury on the left foot is admitted to the

burn include in this client’s plan of care? (incomplete)

•Assess lung sounds q4 hours

•Perform passive range of motion

•Evaluate level of consciousness

. Continuous cardiac monitoring

ANS: D

27- The nurse is taking a client’s blood pressure sphygmomanometer cuff is

inflated. What (incomplete)

•Administer a prescribed PRN antianxiety

•Assess the client’s recent serum calcium

•Notify the healthcare provider of the

•Prepare to implement seizure precautions

ESTA NO TIENE RESPUESTA

28- A client with eczema is using an over-the-counter (OTC) topical product with urea

10% OTC (Aqua Care Cream) to the affected skin areas. Which finding reflects the

expected therapeutic response?

•Decreased weeping of ulcerations in affected area

•Healing with a return to normal skin appearance

•Reduced pain in eczematous areas

•Hydration of affected dry skin areas

ANS: B

29- During an annual health check, the clinic nurse updates an adult female’s health

history. When discussing the woman’s history of lactose intolerance, the client reports

that it has been years since she last consumed dairy products. What dietary suggestions

should the nurse recommend to help ensure that the client receives an adequate intake

of calcium? Select all that apply:

•Increase intake of salmon, sardines, tofu, and leafy green vegetables

•Sip a half-cup of mil during a mid-day meal at least every other day

•Eat at least six servings of citrus fruits weekly

•Include 2 to 3 servings of yellow and green squash weekly

•Take a calcium supplement with vitamin D daily

ANS: A ; C ; E.

30- A healthcare worker with no known exposure to tuberculosis has received a

Mantoux tuberculosis skin test. The nurse’s assessment of the test after 72 hours

indicates 5mm of erythema without induration. What is the best initial nursing action?

•Review client’s history for possible exposure to TB

•Instruct the client to return for a repeat test in 1 week

•Refer client to a healthcare provider for isoniazid (INH) therapy

•Document negative results in the client’s medical record

ANS: A

31.A male client inskeletal traction tellsthe nurse thathe is frustratedbecause he

needs help repositioning himself in bed. Which intervention should the nurse

implement?

• Inform the client that it is the nurse’s responsibility to reposition

• Provide an overhead trapeze to the bed for the client to use

• Place a draw sheet under the client to assist with repositioning

• Administer an intravenous PRN anti-anxiety medication

ANS: B

32-In planning care for a client with pneumonia, which nursing problem should the nurse

identify as the priority?

• Impaired gas exchange related to the effects of alveolar-capillary

membrane changes

• Acute pain related to the effects of inflammation of the parietal pleura

• Deficient fluid volume related to fever, infection, and increased metabolic rate

• Disturbed sleep pattern related to pain, dyspnea, and hospitalization

ANS: A

33.A hospitalized client with chemotherapy-induced stomatitis complains of mouth

pain. What is the best initial nursing action?

• Encourage frequent mouth care

• Administer a topical analgesic per PRN protocol

• Cleanse the tongue and mouth with glycerin swabs

• Obtain a soft diet for the client

ANS: B

33- A client returns from surgery following a hiatal hernia repair via Nissen

fundoplication. Which position should the nurse implement for this

client?

• Right side-lying to promote stomach emptying

• Prone to apply external pressure to the suture

line Left side-lying to reduce stress on the suture

line 30 degree semi-Fowler’s to drop the

diaphragm

ANS: D

34- An adult woman with Grave’s disease is admitted with severe dehydration is

currently restless and refusing to eat. Which action is most important for the nurse

to implement?

• Keep room temperature cool

• Determine the client’s food preferences

• Maintain a patent intravenous site

• Teach the client relaxation techniques

ANS: C

34- The nurse admits a client who has a medical diagnosis of bacterial meningitis

to the unit. Which intervention has the highest priority in providing care for this

client?

• Administer initial dose of broad-spectrum antibiotic

• Instruct the client to force fluids hourly

• Obtain results of culture and sensitivity of CSF

• Assess the client for symptoms of hyponatremia

ANS: C

36- A client uses triamcinolone (Kenalog), a corticosteroid ointment, to manage

pruritis caused by a chronic skin rash. The client calls the clinic nurse to report

increased erythema with purulent exudate at the site. What action should the nurse

implement?

• Schedule an appointment for the client to see the healthcare provider

• Advise the client to apply plastic wrap over the ointment to promote healing

• Explain that the client needs to complete all prescribed doses of

the medication

• Instruct the client to continue the ointment until all erythema is relieved

ANS: A

37- During a paracentesis, two liters of fluid are removed from the abdomen of a

client with ascites. A drainage bag is placed, and 50 ml of clear, straw-colored fluid

drains within the first hour. What action should the nurse implement?

• Palpate for abdominal distention

• Clamp drainage tube for 5 minutes

• Continue to monitor the fluid output

• Send fluid to the lab for analysis

ANS: C

38- The nurse assesses the dressing of a client who has just returned from post-anesthesia

and finds that the dressing is wet with a moderate amount of bright red bloody drainage.

What action should the nurse take?

• Replace dressing with a new sterile dressing, and monitor the wound hourly

until bleeding is stopped

• Call surgery and request that the surgeon see the wound prior to leaving

the hospital

• Reinforce the dressing and document that a moderate amount of

sanguineous drainage was on the dressing

• Document that the dressing was saturated with serious drainage, and do

not change the dressing

ANS: B

39- While the home health nurse is making a home visit, a client with a history of

seizures demonstrates tonic-clonic seizure activity. What action should the nurse

implement first?

• Direct a family member to call emergency services

• Ascertain the trigger event

• Protect the client’s head with a pillow

• Observe the postictal breathing pattern

ANS: C

40- A client who weighs 176 pounds is admitted to the intensive care unit with a serum

glucose level of 600 mg/dl and a serum acetone level of 50 mg/dl. Regular insulin at a

rate of 0.1unit/kg/hour is prescribed. The pharmacy provides a solution of Regular

insulin 100 units/100 ml of normal saline. The nurse should set the infusion pump to

deliver how many ml/hour? (Enter numeric value only)

= 8ML/H

41- A client whose history includes IV drug abuse is admitted to the intensive care unit

(ICU) with Kaposi’s sarcoma associated with Acquired Immune Deficiency Syndrome

(AIDS). Which intervention is most important for the nurse to include in the client’s

plan of care?

• Observe for adverse medication reactions

• Assess for signs of AIDS dementia

• Identify signs of opportunistic infections

• Locate local HIV support groups

ANS: C

42-(Photo) The charge nurse observes a newly employed nurse gathering equipment to

obtain a venous blood sample from a client’s implanted port. The nurse has obtained

the equipment seen in the photo. What actions should the charge nurse take? (Select all

that apply)

• Guide the nurse in inserting the needle at a 45 degree angle

• Remind the nurse to wear sterile gloves for this procedure

• Instruct the nurse to obtain several red-topped tubes

• Determine if the nurse has ever performed this skill

• Assist in obtaining the correct needle to access the port

ANS: D; E

43- After a computer tomography (CT) scan with intravenous contrast medium, a

client returns to the room complaining of shortness of breath and itching. Which

intervention should the nurse implement?

A. Send another nurse for an emergency tracheotomy set

B. Call respiratory therapy to give a breathing treatment

C. Review the client’s complete list of allergies

D. Prepare a dose of Epinephrine (Adrenalin ) .

ANS: D

44- The nurse is reviewing blood pressure readings for a group of client’s on a

medical unit. Which client is at the highest risk for complications related to

hypertension?

A. Young adult Hispanic female who has a hemoglobin of 11 gm and drinks beer

every day

B. Middle-aged African-American male who has a serum creatinine level of 2.9 mg/dL

C. Older Asian male who eats a diet consisiting of smoked, cured, and pickled foods.

D. Post-menopausal Caucasian female who overeats and is 20% above ideal body weight

ANS: B

****PAQUETE QUE EMPIESA CON LEAVE OLD TIES-EL DE ORLIN-YADIRA

Y CONTINUACION DEL HESI MED SURGE PACKAGE QUE NO ESTAN EN

LAS DE ARRIBA.

1- Shingles— Teach the pt about phantom pain

2- Shingles Select all the apply

– pain

– ability

– skin integrity

3- PATIENT W/ EZCEMA APPLYING CREAM TTO IS WORKING:

HEALING WITH A RETURN SKIN TO NORMAL APPEARANCE.

4- PT WITH OBESITY HIGH GLUCOSE LEVEL IS AT RISK FOR?

CARDIOVASCULAR DISEASE

5- FOR ANEMIA WHAT DOESN’T HAVE IRON, WHICH FOODS ARE

NOT RICH IN IRON?

NO ORANGE

6- PT. W/ RISK OF DVT

PERFORM ROM EXERCISES ALSO LEGS EXERCISE CAN BE

OTHER WAY TO ANSWER

7- DISCHARGE FOR VENOUS ULCERS SELECT ALL APPLY?

– ELEVATE THE FEET WHEN LAYING DOWN

– CHECK BROWNISH SKIN AROUND THE ANKLES

– VITAMINS

HAY OTRA RESPUESTA POR AHI QUE ES SOCKS EN VEZ

DE VITAMINS

8- PT W/ SIADH:

HARD CANDY FOR THIRST.

9- PT ARRIVE TO PACU POSTOP MOANING WHAT TO DO:

CHECK PULSE, BP AND RESPIRATIONS.

10- Pt. DIAGNOSED RECENTLY W/ DM HAVE NOT BEEN ABLE TO CONTROL

GLUCOSE LEVEL DURING 3 MONTH WHAT SHOULD BE DONE:

I PUT CHECK FOR A1C LEVEL

(OTHER SAY ASSESS FOR WHAT SHE HAVE BEEN EATING 3 DAYS AGO).

11- WHEN BP IS HIGH

ADMINISTER (LASIX)

12- PATIENT W/ ESOPHAEGAL VARICES HAVE NOT BE BLEEDING FOR 3 DAYS:

PROVIDE LUKE WARM BROTH, ICE TEA AND LEMON POPSICLE.

13- CALCULO:

0.75

15- PT WITH OSTEOMALCIA

RISK FOR INJURY

16- SBAR—EXPLAIN SPECIFIC REASON FOR URGENT NOTIFICATON

TEMPERATURE

17- INTESTINAL BOWEL OBSTRUCTION

PLACE THE PT 90 DEGREES SITTING

18- OSTEOARTHRITIS

RISK FOR INJURY RELATED TO JOINT PAIN

19- BONE CANCER TYPE IV:

GIVE OPIODS- NON OPIODS ANALGESICS.

20- HYPOTHYROIDISM

RESTRICT SODIUM NA 122

21- PT ARRIVES TO CLINIC W/ NUCHAL RIGIDITY FEVER FOR 6 HOURS WHAT

TO DO:

PREPARE FOR ISOLATION PRECAUTIONS

( I PUT THIS ONE AND NO LUMBAR PUNCTURE)

22- INTERMITENT CLAUDICATION TEACHING

BANDAGE ELASTIC WRAPED AROUND LEGS

TAMBIEN PUEDE SALIR COMO PAIN TRACTION CAST NOTIFY MD (CAST NO

MORE THEN 4HR)

23- PREOPERATIVE NURSING CARE

ASSESS EMOTIONAL PREPAREDNESS

ALSO CAN BE CONCERNS AND ANXIETY FOR SURGERY DEPENDE LA QUE

PONGAN

24- TRACHESTOMY CARE:

LEAVE OLD TIES ON UNTIL NEW ONES BE ON PLACE OR SECURE.

25- STERNAL TRACTION COMPLAINS OF

PAIN ADMINISTER PRN MEDS

26- EXTERNAL FIXATION

ADMINISTER PRN

MEDS

27- MULTIPLE SCLEROSIS (MS)

ADMINISTER ANTIMEDICS/ PRN AS NEEDED

28- FEMALE PATIENT HOW HAVE EPIGASTRIC PAIN FOR 3 DAYS HAVE BEEN

TAKIN ANTACIDS AND NO RESOLVE ARRIVE TO HOSPITAL W/HR;128 BPM,

BP110/70 WHAT IS THE MOST IMPORTANT INTERVENTION FINDING IN

ASSESSMENT:

ASSES FOR RADIATING JAW PAIN.

29- Pt. W. RADIACTIVE THERAPY WHAT TO TEACH/ RECOMMEND TO

PROTECT THAT PART OF THE SKIN SPECIALLY FROM THE SUN

30- Pt WITH ALS WHAT TO DO TO PREVENT RESPIRATORY COMPLICATIONS:

TEACH BREATHING TECNIQUES, USES SPIROMETER, AUSCULTATE FOR BREATH

OR LUNG SOUNDS.

31- PT WITH LEFT LEF ULCER:

KEEP LEG ELEVATED AS MUCH AS HE CAN.

32- PT WITH AN EXTERNAL DEVICE COMPLAINING OF PAIN:

ASSESS FOR PHERIPHERAL PULSES.

33- CALCULATION 1G/0.4 G

= 2.5

34- EXAMPLES OF DASH DIET:

PEEL FRUITS AND VEGETABLES.

35- CHEST TUBE W/ A DRAINAGE CHANGING FROM CLEAR TO GREEN:

KEEP IV FLUIDS.

36- PT W/ OPEN ANGLE GLAUCOMA SELECT ALL THAT

APPLY: FREQUENT EYE EXAM TO ASSES FOR VISSION,

USE DROPS TO DIMINSH IOP,

AVOID EXTRENOUS EXERCICES LIKE JOGGING OR

RUNNING ( YO PUSE SOLO ESAS 3 RESPUESTAS).

37- PT W/ HYPERTHYROIDISM DEVELOPING EXOSPHTALMUS:

PRESCRIBE TEAR EYE DROPS.

38- PT VOMITING BLOOD LIKE THE PICTURE SAME AS HEMATENSIS:

CHECK VITAL SIGNS ( ASI ESTA EN TODOS LOS PAPELES)

AUSCULTATE LUNGS SOUNDS ( FUE LO QUE PUSO YADIRA)

39- PATIENT W/ ML FELL AND WHEN RECEIVING THE NURSE HE HAVE 2

PROJECTILE VOMITS WHAT SHE DO:

PROVIDE ANTIEMETICS PRN .

40- PT W/ RAYNAUD SYNDROME WHICH WORK AS A DATA ENTRY

CLERK: PROVIDE A SPACE TO WARM THE ENVIROMENT NEXT TO HER

( ALGO ASI ERA LA RESPUESTA). Y HAY OTRA RESPUESTA QUE SOLO

DICE KEEP MONITORING

41- PATIENT THAT HAVE THE K= 6.7 WHAT MEDICATION PROVIDE:

KAYELAXATE (TREATS HYPERKALEMIA).

42- COLON CANCER

PT KAYELAXATE

MED

43- RENAL INJURY

KAYELAXATE MED

44- PT WITH A BRONCHOSCOPY AND DRINK A GLASS OF JUICE

: DELAY THE PROCEDURE 6 HOURS

45- NEW PATIENT DIAGNOSES WITH DM TYPE IS RECEIVING TEACHING IN

WHICH GLUCOMETER WILL BE THE BEST:

ASSES FOR VISUAL ACUITY AND ABILITY TO READ OR SOMETHING LIKE THAT.

46- ABG (PH 7.25 PCO2 50 SODIUM 60 TACHY

AND CONFUSION/ RESPIRATORY

47- ACUTE AGN DIET:

RESTRICT NA INTAKE.

48- PT W/ A EXPRESSIVE APHASIA IS ANGER WHAT SHOULD DO

THE NURSE: CVA

COMMUNICATE W/ PICTURE BOARDS.

49- NURSE IS TEACHING THE WIFE IF A PATIENT DIAGNOSED W/

SEIZURE WHAT TO DO:

TEACH HER HOW TO POSITION HIM

50- PT AFTER TTO OF SOMETHING AND WANTS TO

EAT: NURSE ASSESS FOR BOWEL MOVEMENTS.

51- SLE:

ASSESS FOR HEMATURIA

52- PATIENT ALLERGIC TO BANANA (LATEX):

CALL TO MD AND OR STAFF TO BE CHANGE EVERYTHING

FOR SINTHETIC MATERIALS,

53- SUBCUT EMPHYSEMA- TORACOTOMY WAS A SELECT ALL

THAT APPLY:

ASSESS FOR LUNG SOUNDS,

NECK DISTENTION

I THINK IT WAS AND OTHER CHOICE THAT I NOT REMEMBER NOW.

54- RESTLESS LEG SYNDROME CON FEOSOL:

YO PUSE ASSESS FOR IRON AND

FERRITIN.

55- BNP

ADMINISTRATIVE FUROSEMIDE LASIX IV

56- PARKINSON PT WALKING

REASURE THAT STEPPING ON CRACKLES IS NOT HARMFUL

57- ADDISON DISEASE

TAKE CORTICOSTEROID MEDS

58- CARPO TONIC SYNDROME

WEAR BRACE IN BOTH

WRIST

59- PARKINSON AND ALZAIMERS PT

TATICARDIC AND CONFUSION

60- CHECK SHOES FOR DIABETIC PATIENTS

61- MID ABDOMEN BURNING

PAIN PEPTIC ULCER

62- ANTIBIOTICS

CLEAR DRAINAGE IMPROVE

63- ALLOPRINOL FOR

GOUT TAKE MEDS

ALWAYS

64- BLOOD TRANSFUSION HIGH TEMPERATURE

BACK PAIN AND HYPOTENSION

( ABO- LOW BACK PAIN AND HYPOTENSION)

65- CENTRAL FALL RISK

CARDIOVASCULAR DISEASE

66- RIGHT HIP

FRACTURE O2 SAT

LEVEL

67- RESPIRATORY ALKALOSIS ( MAIN CONCERN)

68- DESCRIBE PAIN NEUROPATHY

NERVOUS SYSTEM

69-

BETA 1.6 CALCULO

70- ACUTE ABDOMINAL PAIN, NASUA, PROJECTIBLE

VOMITING SEVERE HEADECHE AND PHOTO SENSITIVITI

71- UROLITHISIS O LITHOTRIPSY PROCEDURE—-RESTRICT PHYSICAL

ACTION

72- UAP ( DICE EL PACIENTE QUE TIENE ABD PAIN LARGE TARRY STOOL

TEST STOOL FOR OCCULT BLOOD

73- Insulin for a glucose level of 255 (Pte tmeblando despues que le

pusieron insulin.)…..

Obtain capillary glucose.

74- NGT proper tube procedure…. Elevate Head 60 to 90 degree….

75- RA (rheuma)…….Impaired peripheral mobility relate to join pain.

76- Finger stick glucose finding 50mg… LOC Level of conscious

77- BMI (una persona que pueden tener colon cancer) …. Large waits

circumference with central fat…..

Med surge Hesi:

1-Q: Parkinson patient walking……….Reassure that stepping on walks’ is not harmful.

2-Q: A male client, who is 24hours…… Auscultate bowel sounds in all four

quadrants. 3-Q: Beta 1.6 Math calculo

4-Q: Acute abdominal pain, nausea, projectile vomiting…. Severe headache and photo sensitivity

5- Q:StageIV cancer……Administrationopioids,noopioid

meds. 6-Q: Eczema…… Healing with a return to normal skin

appearance. 7-Q: Patient with DVT…. Perform leg exercise.

8-Q: Carpal tunnel…. Wear brace on bath

wrist. 9-Q Hypothyroidism Sodium 122 mcg/L

10-Q: Epigastric pain-female…… radiating pain to the

jaw 11-Q: OstemalaciaRisk for injury.

12-Q: Bronchoscopy (el pte 2 horas antes tomo jugo)… Delay procedure for 6 hours.

13-Q: Pictine (vomito dentro de un emesis)……Obtain vital signs.

14-Q:Cancercells Benignoffer theclientreassurancethatthisinformationindicatesthattheclientscancer

cells are benign.

15- Q: Cataract ……Light housekeeping is permitted, but avoid heavy

lifting. 16-Q:Esophagealvarices……Lukewarmbath,coldicetea,lemon

Popsicle. 17-Q: Urolithisi procedure…… Restrict physical action or Stain

urine????

18-Q: A client who took a camping vacation…… Jaundice

sclera. 19-Q: External fixation… Administrate PRN med.

20-Q: UAP, (dice el pte tiene abd pain, large tarry stool……Test stool for occult

blood. 21-Q: Pheocromocytoma Monitor BP.

22-Q: ABD a client experiences an ABD…… Low back pain and

hypotension 23-Q: SBAR …… Explain specific reason for urgent notification

24-Q Primary goal of nursing care for pte pre-op……assess emotional preparedness

25-Q: Intermittent claudication….wrap elastic bandage around legs.

26-Q TURP……Drink fluids 3L of water each day.

27-Q: Adult women difficulty time keeping…. .Hemoglobin A1C level.

TestBankWorld.org

28-………………………….Q: CVA: ……encourage use of

picture chart. (Boards) 29-Q: Diabetes KAPotassium of

2.5

TestBankWorld.org

30-Q: Shingles *select all that apply… Pain…Ability…Skin integrity

31- Q: Iron rich, food (pte with anemia cual NO se le va a dar)…..Orange

juice 32-Q: MS-multiple sclerosis…… Administration antiemetic.

33-Q: A client diagnoses with stable angina…. Call 911 if pain is

unrelieved. 34-Q: BNP…… administrative furosemide Lasix IV.

35-Q: DVT calculo: 9

36-Q: An older male client….. Palpate the bladder above the symphysis pubis.

37- Q: Insulin for a glucose level of 255 (Pte tmeblando despues que le pusieron insulin.)…..Obtain

capillary glucose.

38-Q: Pte con stiffness neck……Prepare for lumbar puncture.

39- Q: Shingles…..Check if client was vaccinated for shingles in the

past. 40-Q: SIADH: …… Hard candy

41-Q: Tele therapy radiation……Protect skin from sunlight exposure

42-Q: Tracheostomy….. Leave old ties in place, until new one …….

43-Q: PACU…. Pulse , BP, respiration.

44- Q: Venous legs ulcer (all that apply)…. Elevate the feet when lying down, Check brownish skin

around ankles, vitamins.

45- Q: NGT proper tube procedure…. Elevate dead 60 to 90

degree…. 46-Q: Xenograft……The xenograft is from non-human

……

47-Q: Clean water fluid…..Maintain IV fluid

48- Q: RA (rheuma)…….Impaired peripheral mobility relate to join

pain. 49-Q Hemodialysy…..Lower the head of the …. And elevate feet.

50-Q: A male adult come……….Obtain a sputum samples for culture.

51-Q Finger stick glucose finding 50mg… LOC Level of conscious

52-Q: BMI (una persona que pueden tener colon cancer) …. Large waits circumference with central fat…..

53-Q: When explaining dietary guidelines w/ Acute Glomeru……..Restrict sodium intake.

54-Q: Renal injury: Koyexalate 15 grams PO ???????

55-Q: ABG (PH:7.25-PCO2: 50, Sodium 60………..Tachy and confunsion.

Leave a Comment

Scroll to Top